Practice Questions (Exam 5-8)- Comps B

अब Quizwiz के साथ अपने होमवर्क और परीक्षाओं को एस करें!

Research using elicited imitation tasks (e.g., imitating a sequence of events such as removing a mitten from a puppet, shaking the mitten, and then replacing the mitten on the puppet) indicate that, for most infants, the onset of the ability to recall the past occurs: Select one: A. between 3 and 6 months of age. B. between 6 and 12 months of age. C. between 12 and 18 months of age. D. between 18 and 24 months of age.

between 6 and 12 months of age --In one study using the procedure described in this question, 75% of infants 6, 9, or 11 months of age imitated at least one action (e.g., removing the mitten) after a delay of 24 hours.

For a diagnosis of Bulimia Nervosa, an individual has to exhibit which of the following? Select one: A. purging following binge eating for at least one month B. binge eating for at least two months C. lack of control over eating plus either purging or excessive exercise for at least two months D. binge eating and inappropriate compensatory behavior for at least three months

binge eating and inappropriate compensatory behavior for at least three months

A mother is most likely to accurately recall which of the following about her child? Select one: A. birth weight B. ages when first crawled and walked alone C. height at certain ages D. age when read first word

birth weight

The optimal item difficulty (p) for a true-false test is: Select one: A. .25. B. .50. Incorrect C. .75. D. .95.

0.75 --The optimal item difficulty depends on several factors, including the likelihood that an examinee can choose the correct answer by guessing. The optimal item difficulty is halfway between 100% of examinees answering the item correctly and the likelihood of answering the item correctly by chance alone. For true-false items, the probability of choosing the correct answer by chance is .50, so the optimal difficulty level is .75.

The most commonly-prescribed drugs for Tourette's syndrome are drugs that: Select one: A. increase dopamine levels. B. block dopamine transmission. C. increase acetylcholine levels. D. block acetylcholine transmission.

block dopamine transmission --Haloperidol is an antipsychotic drug that exerts its effects by blocking the transmission of dopamine. It has been found effective for alleviating the symptoms of Tourette's syndrome in about 70% of patients.

For most children, holophrastic speech begins between the ages of: Select one: A. 8 to 12 months. B. 12 to 18 months. C. 18 to 24 months. D. 24 to 36 months.

12 to 18 months.

Children begin to express jealousy, embarrassment, and other self-conscious emotions between the ages of: Select one: A. 6 to 12 months. B. 18 to 24 months. C. 30 to 36 months. D. 42 to 48 months.

18 to 24 months --Children begin to exhibit self-conscious emotions at the time the sense of self emerges, which is between 18 and 24 months of age. By 24 months, jealousy, embarassment, and empathy are usually evident; and these emotions are followed by shame, guilt, and pride at 30 to 36 months.

Borderline Personality Disorder is most commonly diagnosed in individuals ages: Select one: A. 19 through 34. B. 29 through 44. C. 39 through 54. D. 49 through 64.

19 through 34 --The rates of BPD are highest during the young adult years. Although individuals with this disorder often continue to exhibit some symptoms throughout their life, by age 40, many no longer meet the diagnostic criteria for BPD.

For children with a learning disorder, the most frequent co-diagnosis is ADHD, with approximately _____% of children with a learning disorder also receiving the latter diagnosis. Select one: A. 5 to 10 B. 20 to 30 C. 45 to 50 D. 65 to 75

20 to 30 --The studies have found that the most common co-diagnosis is ADHD, with about 20 to 30% of children with a learning disorder also receiving a diagnosis of ADHD.

Studies investigating self-recognition in young children have found that by _______ months of age, over 60% of infants recognize their own image in a mirror. Select one: A. 21 B. 18 C. 15 D. 12

21 --n a frequently cited study investigating the emergence of self-recognition, J. Brooks-Gunn and M. Lewis had mothers wipe the noses of their babies, ages 9 to 24 months, with a red dye, and then place the babies in front of a mirror. Results indicated that about 30% of the 18-month-old babies recognized themselves in the mirror (i.e., they rubbed the red spot on their own noses when they saw their reflection), while 60% of the 21-month-old babies did so.

It is not until about _____ years of age that most children express more favorable attitudes toward members of their own ethnic or racial group than toward members of other groups. Select one: A. 4 B. 6 C. 8 D. 10

4 --A. Clark, D. Hocevar, and M. H. Dembo, for instance, found that, by 4 years of age, children typically express more favorable attitudes toward members of their own ethnic group than members of other ethnic groups

Studies comparing bilingual and monolingual children on measures of inhibitory control and cognitive flexibility have generally found that: Select one: A. monolingual children outperform bilingual children on both measures. B. bilingual children outperform monolingual children on both measures. C. monolingual children outperform bilingual children on a measure of inhibitory control but bilingual children outperform monolingual children on a measure of cognitive flexibility. D. bilingual children outperform monolingual children on a measure of inhibitory control but monolingual children outperform bilingual children on a measure of cognitive flexibility.

B. bilingual children outperform monolingual children on both measures.

Which of the following is most useful for explaining why so many people believe that their horoscopes accurately describe them? Select one: A. Barnum effect B. false consensus bias C. Zeigarnik effect D. illusory correlation

Barnum effect --Researchers have found that individuals are often willing to agree that vague descriptions of them - such as those presented in horoscopes - accurately describe their personal characteristics. The Barnum effect (also known as the Forer effect) predicts that people tend to accept vague or general descriptions of themselves as accurate. The Barnum effect has been attributed to a number of factors including gullibility, wishful thinking, and a confirmation bias.

Why does the most pervasive modern form of supervision tend to be the triangular model? Select one: A. Because organizational policies and professional knowledge form the foundation, while the relationship with the supervisor forms the core. B. Because the relationship with the supervisor and professional experience form the foundation while care of clients forms the core. C. Because the relationship with the clients and counseling theory form the foundation and the organizational policies form the core. D. Because the relationship with the organization and the supervisor literature form the foundation and personal experience form the core.

Because organizational policies and professional knowledge form the foundation, while the relationship with the supervisor forms the core. --One of the common modern forms of supervision is a triangular model where the organizational policies and professional knowledge form the foundation, the supervisor relationship is at the core, and the ultimate emphasis on providing service to the clients.

A psychologist investigating short-term memory shows the letter "T" to each research subject for a brief period and, after removing the letter, asks the subject what he or she has seen. The psychologist finds that many subjects report seeing the wrong letter, with the most common error of recall being which of the following letters? Select one: A. L B. D C. S D. F

D --Studies similar to the one described in this question have found that errors in short-term memory usually involve a confusion of letters that sound alike, thereby supporting the theory that information is stored acoustically in short-term memory. Of the letters listed in the responses, only "D" sounds like "T".

A child with Autism Spectrum Disorder would most likely obtain a higher score than a "normal" peer on which of the following? Select one: A. Embedded Figures Test B. Wisconsin Card Sorting Test C. Tower of London Test D. WISC-V Arithmetic subtest

Embedded Figures Test --Children with autism often show superior performance on the Embedded Figures Test, which measures field dependence/independence.

Which of the following is true about the standard error of the mean? Select one: A. It increases as the standard deviation decreases. B. It is not affected by the size of the standard deviation. C. It decreases as sample size increases. D. It decreases as the number of samples increases.

It decreases as sample size increases.

According to Helms's White Identity Development Model, the reintegration stage is characterized by: Select one: A. a realization that whites have a responsibility for racism. B. adoption of a "culture-blind" perspective. C. embracing white identity while rejecting racist views of minorities. D. adopting a belief in white superiority and minority inferiority.

adopting a belief in white superiority and minority inferiority. --The disintegration phase, which is marked by considerable conflict, may be resolved by adopting traditional racist views, which in turn characterizes the reintegration phase.

The 2013 National Survey on Drug Use and Health found that the largest percentage of respondents ages 12 through 17 used which of the following during the past month? Select one: A. hallucinogenics B. marijuana C. tobacco products D. alcohol

alcohol --In this survey, 11.6% of respondents ages 12 to 17 reported using alcohol in the past month.

A therapist who believes the therapeutic change process should focus on altering the client's environment is adopting which of the following approaches? Select one: A. alloplastic B. autoplastic C. emic D. etic

alloplastic --Alloplastic refers to changing or adapting to the environment by effecting changes in the environment.

Which of the following drugs would be most useful for reducing neuropathic pain? Select one: A. amitriptyline B. fluoxetine C. clozapine D. tacrine hydrochloride

amitriptyline --Neuropathic pain is chronic pain that is due to a nervous system injury or dysfunction. The first-line treatments for neuropathic pain are analgesics, which include certain antidepressants, anticonvulsants, opioids, and local anesthetics. Antidepressant drugs that increase levels of both serotonin and norepinephrine not only reduce the depression that often accompanies neuropathic pain but also have analgesic properties. The tricyclic amitriptyline is one of the oldest and most widely used tricyclic drug for neuropathic pain.

Bilateral lesions in which of the following areas of the brain is most likely to result in a loss of the fear response without loss of other emotional responses? Select one: A. amygdala B. hippocampus C. suprachiasmatic nucleus D. thalamus

amygdala --The case of "patient SM" has confirmed that the amygdala is responsible for the experience of fear in response to frightening stimuli. SM has focal bilateral amygdala lesions as the result of a rare disease. While she is able to experience other emotions, she does not experience fear.

Research on sensory compensation in deaf children has found that: Select one: A. children who are deaf outperform peers without hearing deficits on most visual tasks and this difference is apparent by four to six years of age. B. children who are deaf outperform peers without hearing deficits on most visual tasks but this difference is not apparent until preadolescence. C. children who are deaf perform similarly to peers without hearing deficits on most visual tasks. D. children who are deaf do more poorly than peers without hearing deficits on most visual tasks.

children who are deaf perform similarly to peers without hearing deficits on most visual tasks. --Contrary to the predictions of the sensory compensation hypothesis, deafness is not generally associated with superior visual abilities. One exception is that some deaf children and adults who use sign language exhibit superior visual processing skills on mental rotation, face recognition, and visual attention tasks, but this advantage appears to be due to the use of sign language rather than to deafness itself.

When using Atkinson, Morten, and Sue's (1993) Racial/Cultural Identity Development Model, a person's stage of identity development is determined by: Select one: A. identifying his/her degree of acceptance of the minority and majority cultures. B. assessing the amount of contact he/she normally has with members of the minority and majority cultures. C. assessing his/her attitudes toward the minority and majority cultures. D. evaluating his/her reaction to racial oppression.

assessing his/her attitudes toward the minority and majority cultures. --Each stage in this model is characterized by a different combination of attitudes toward one's own minority group, other minority groups, and the majority (dominant) group.

A distinguishing feature of the Montessori method is its: Select one: A. emphasis on cooperative learning and group achievement. B. use of art and music to teach academic subjects. C. assumption that all learning stems from sense perception. D. placement of children in classes on the basis of developmental level rather than chronological age.

assumption that all learning stems from sense perception --The Montessori method is characterized by the use of special materials designed to teach sense discrimination. For example, to facilitate the teaching of reading, large letters of the alphabet are covered with sandpaper.

Which of the following is categorized as a "negative symptom" of Schizophrenia? Select one: A. avolition B. loosening of associations C. catatonia Incorrect D. perceptual disturbances

avolition --Avolition is an inability to initiate and persist in action and is

On Holland's Self-Directed Search, the frequency with which an examinee's code occurred in the standardization sample is referred to as: Select one: A. congruence. B. commonness. C. consistency. D. coherence.

commonness --The score report for the SDS provides information on congruence, consistency, coherence, differentiation, and commonness. Commonness indicates the degree to which an examinee's code occurred in the standardization sample (i.e., the percent of people in the sample who obtained the same code).

Sherif et al.'s (1961) "Robber's Cave" study is considered important because of the information it provided on: Select one: A. social facilitation and inhibition. B. competition and cooperation. C. bystander apathy. D. conformity to group norms.

competition and cooperation --Sherif and his colleagues found that introducing a superordinate goal that could be accomplished only when members of two groups cooperated was the most effective way for reducing intergroup competition and hostility.

A psychologist is asked to evaluate a prisoner for the purpose of making a recommendation about the prisoner's readiness for parole. To be consistent with ethical guidelines, the psychologist should: Select one: A. refuse to do the evaluation since making a recommendation in this situation is prohibited. B. conduct the evaluation but base his recommendation on the results of standardized tests only. C. conduct the evaluation as long as the prisoner has been told the purpose of the evaluation and has been warned about the limits on confidentiality. D. conduct the evaluation after obtaining a consent from the prisoner's legal representative and report the results without making a recommendation.

conduct the evaluation as long as the prisoner has been told the purpose of the evaluation and has been warned about the limits on confidentiality.

Polly Pooch teaches her son to feed the dog by first showing him how to put the dog food into the dog dish. Once the boy has mastered that task, she teaches him to open a can of dog food and then put it into the dish. Finally, she teaches her son to open the cupboard, take out a can of dog food, open it, and put the dog food in the dog dish. The procedure that Mrs. Pooch has used is best described as: Select one: A. backward chaining. B. forward chaining. C. stimulus control training. D. sequential training.

backward chaining --Polly Pooch's son has been taught a complex behavior by teaching him the individual responses in the "behavior chain." Since Mrs. Pooch has started with the last behavior and "worked backward" from there, this is best described as an example of backward chaining.

Cognitive dissonance theory and ________ share in common the assumption that an uncomfortable experience or sensation is the primary cause of attitude change. Select one: A. the sleeper effect B. attitude inoculation C. balance theory D. the elaboration likelihood model

balance theory --Like cognitive dissonance theory, balance theory (Heider, 1958) is based on the principle of cognitive consistency, which proposes that having inconsistent attitudes produces a state of discomfort that leads to attitude change.

Dr. Prattle is asked by a local radio station if she would be interested in doing a "talk show" on Saturday mornings. On the show, she will respond to people who call the station. To be consistent with the provisions of the Ethics Code, Dr. Prattle should: Select one: A. refuse to appear on the show. B. avoid giving direct advice of a psychological nature to callers. C. be sure that callers understand that their conversations with her do not constitute a therapeutic relationship. D. be sure that she has adequate malpractice insurance.

be sure that callers understand that their conversations with her do not constitute a therapeutic relationship --As noted by M. B. Canter, et al. (1994), in their discussion of the Ethics Code, the ethical requirements with regard to media psychology are "still evolving." However, since this question refers specifically to the Code, the best answer is the one that best resembles the Code's language. Of the responses, this one comes closest to the language of Standard 5.04 (Media Presentations). Moreover, Canter, et al. note that on radio talk shows, the psychologist should be sure that he/she clearly distinguishes between "media psychology" and "therapy"

The endoplasmic reticulum is involved in: Select one: A. processing incoming visual stimuli. B. processing incoming auditory stimuli. C. transporting proteins and fats. D. storing neurotransmitters

transporting proteins and fats --The rough portion of the endoplasmic reticulum plays a role in the synthesis and transportation of proteins, while the smooth portion is involved in the synthesis and transportation of fats.

The minimum and maximum values of the standard error of estimate are: Select one: A. -1 and +1. B. 0 and 1. C. 0 and the standard deviation of the criterion. D. 0 and the standard deviation of the predictor.

0 and the standard deviation of the criterion. --The standard error of estimate equals the standard deviation of Y (the criterion) times the square root of 1 minus the validity coefficient squared. When the validity coefficient is 1, the standard error equals 0 (there is no error), and when the validity coefficient is 0, the standard error equals the standard deviation of the criterion scores.

According to the DSM-5, the lifetime prevalence of Schizophrenia in the general population is approximately: Select one: A. .01%. B. 0.5%. C. 5%. D. 10%.

0.5% --A lifetime prevalence rate of 0.3% and 0.7% is the rate reported in the DSM-5. Since 0.5% falls within this range, this is the best answer.

A research study examined the relationship between television viewing habits and academic achievement test scores. The results showed that children who watched more television generally had lower test scores. The Pearson-product moment correlation coefficient was -.81. What is the proportion of variance of achievement test scores that is accounted for by television viewing? Select one: A. 0.9 B. 0.66 C. -0.81 D. -0.9

0.66 --The value of the Pearson-product moment correlation coefficient ranges from -1.0 to +1.0, and the corresponding proportion of variance (which is referred to as the coefficient of determination) is computed by squaring the correlation coefficient. The correlation coefficient for television viewing and achievement scores is -.81. The proportion of variance is calculated by squaring -.81 (i.e., -.812), which is approximately .66.

A researcher obtains a correlation coefficient of .85 when he correlates the raw scores participants received on her study's predictor and criterion. If the researcher converts each participant's raw scores to non-normalized T scores and calculates a correlation coefficient, this coefficient will be: Select one: A. larger than .85. B. smaller than .85. C. .85. D. -0.85

0.85 --Converting raw scores to non-normalized T scores is a linear transformation, and a linear transformation of scores does not have an effect on the correlation coefficient. A linear transformation of raw scores affects a distribution's mean and standard deviation but does not change is shape. (Note that raw scores can also be converted to normalized T scores when scores obtained by a sample are not normally distributed but there is reason to believe that the distribution is normal in the population. In this case, the correlation coefficient would be affected by the transformation.)

Arthur Jensen's (1998) research on sources of variability in IQ indicated that the average IQ difference within families (as measured by the difference in IQs of full siblings living together) is _____ points. Select one: A. 4 B. 11 C. 18 D. 26

11 --Jensen found that the average IQ difference between siblings in the same family was 11 points, while the average difference in IQs of the same person tested on two occasions a week apart was 4 points.

According to Piaget, the ability to think abstractly is first evident at about ____ years of age. Select one: A. 6 B. 8 C. 11 D. 15

11 --The formal operational stage is the final stage in Piaget's stage model of cognitive development. It emerges at around age 11.

A researcher compares the effectiveness of two different relaxation techniques for relieving test anxiety by randomly assigning 30 students who have high levels of anxiety to one of the treatments so that each group includes 15 students. If the researcher uses the t-test for independent (unrelated) samples to analyze the data she obtains, the degrees of freedom will be: Select one: A. 13 B. 14 C. 28 D. 29

28 --The formula for the degrees of freedom for the t-test for independent samples is N-2, where N is the total number of participants. The total number of participants is 30: 30 - 2 = 28.

Studies on information processing during the first two years of life indicate that infants first exhibit recognition memory for up to 24 hours following presentation of a stimulus at about ____ months of age. Select one: A. 3 B. 5 C. 7 D. 9

3 --Researchers often use habituation to assess information processing skills in infants. Studies using this technique have found that recognition memory skills are apparent at a very early age. By three months of age, infants habituate to visual stimuli - i.e., show less response to the second presentation of a stimulus for periods up to 24 hours. This is interpreted as indicating that the infant recognizes the stimulus.

Children between the ages of approximately _______ years are in the phallic stage of psychosexual development according to Freud and in the initiative vs. guilt stage of psychosocial development according to Erikson. Select one: A. 1 to 3 B. 3 to 6 C. 6 to 12 D. 12 to 15

3 to 6 --Children between the ages of 3 and 6 are in the third stage of psychosexual development and psychosocial development. Freud referred to the third stage as the phallic stage and Erikson identified the conflict of this stage as involving initiative vs. guilt.

For most children, expressions of pride, shame, and guilt are first evident by ________ months of age. Select one: A. 12 B. 20 C. 30 D. 42

30 --Self-conscious emotions become apparent during the second and third years of life. Research on the ages at which the self-conscious emotions are first evident has provided somewhat inconsistent results. However, there is evidence that empathy, jealousy, and embarrassment emerge at about 18 to 24 months of age and are followed by pride, shame, and guilt at about 30 months.

Kochanska and Knaack (2003) conclude that effortful (inhibitory) control is an important contributor to the early development of conscience. Their research found that effortful control becomes a stable, coherent trait by ________ months of age. Select one: A. 8 to 12 B. 12 to 18 C. 18 to 30 D. 33 to 45

33 to 45 G. --Kochanska and A. Knaack found that effortful control is modestly coherent across tasks at age 22 months but does not become highly coherent (trait-like) until 33 months of age. They also found that children with higher effortful control at 22 to 45 months had stronger consciences at 56 months of age

According to Margaret Mahler's object relations theory, the development of a sense of self is the result of a separation-individuation process that begins at about _____ months of age. Select one: A. 2 to 3 B. 4 to 5 C. 8 to 10 D. 12 to 14

4 to 5 --Separation-individuation is triggered by the child's ability to separate from his/her primary caregiver. The process begins at about 4 to 5 months of age when a child who is being held by his/her caregiver is able to lean away to scan the environment.

The mean age of onset of motor tics in Tourette's Disorder is: Select one: A. 1 to 3 years. B. 5 to 7 years. C. 9 to 11 years. D. 12 to 14 years.

5 to 7 years --The reported mean age of onset of motor tics varies, but a commonly reported range is between 5 and 7 years. The onset of vocal tics is usually later, with a mean age of 11 years.

REM (rapid eye movement) sleep accounts for about ___ of a newborn's total sleep period and ___ of an adult's total sleep period. Select one: A. 75%; 10% B. 50%; 20% C. 30%; 40% D. 10%; 50%

50%; 20% --Infants not only begin a sleep period with REM sleep but also spend more time in REM sleep, with REM sleep representing about 50% of their total sleep period. In contrast, adults begin a sleep period with non-REM sleep and spend less time (about 20%) in REM sleep.

An examinee obtains a score of 70 on a test that has a mean of 80, a standard deviation of 15, and a standard error of measurement of 5. The 95% confidence interval for the examinee's score is: Select one: A. 65 to 75 B. 60 to 80 C. 55 to 85 D. 50 to 90

60 to 80 --The 95% confidence interval for an obtained test score is constructed by multiplying the standard error of measurement by 1.96 and adding and subtracting the result from the examinee's score. This interval is closest to the 95% confidence interval and was obtained by multiplying the standard error by 2.0 (instead of 1.96) and adding and subtracting the result from the examinee's score of 70.

Research conducted in a number of countries has confirmed that, when men are shown pictures of women of various shapes and sizes, they tend to prefer women whose waist circumference is about ___ percent of their hip circumference. Select one: A. 30 B. 50 C. 70 D. 90

70 --Evolutionary psychologists propose that a woman's waist-to-hip ratio provides potential mates with information about her reproductive potential. Studies in the United States, Britain, and other countries have found that men generally prefer a low waist-to-hip ratio of around .7. According to D. Singh, this is because this ratio is associated with greater fertility and health.

A test has a test-retest reliability coefficient of .70. This means that: Select one: A. 70% of variability in test scores is true score variability. B. 49% of variability in test scores is true score variability. C. 70% of variability in test scores is attributable to measurement error. D. 49% of variability in test scores is attributable to measurement error.

70% of variability in test scores is true score variability. --A reliability coefficient of .70 indicates that 70% of variability in test scores is due to true score variability, while the remaining 30% is due to measurement error.

Which of the following MMPI-2 two-point codes is not accurately matched with its interpretation? Select one: A. 78/87: bipolar symptoms with substance abuse B. 49/94: acting out behaviors with low frustration tolerance C. 23/32: depressed with mild physical symptoms D. 34/43: passive-aggressive and angry with poor insight

78/87: bipolar symptoms with substance abuse --Scale 7 is the psychasthenia scale, and Scale 8 is the schizophrenia scale. Highest scores on these two scales is associated with significant emotional turmoil, depression, pessimism, OCD, and psychosis. (Bipolar disorder and possible substance abuse are suggested by a two-point code of 29/92.)

About 10 to 15% of new mothers develop postpartum major depression in the first few days or weeks following delivery. In addition, up to _____% of new mothers exhibit "baby blues," a less severe condition that involves mood swings and sleep disturbances. Select one: A. 30 B. 40 C. 50 D. 80

80

Which of the following is an example of higher-order conditioning? Select one: A. A CS is paired with an US until a CR is established; subsequently, the CR occurs in the presence of stimuli similar to the US. B. A CS is paired with an US until a CR is established; subsequently, the CS elicits a variety of responses including the CR. C. A CS is paired with an US until a CR is established; subsequently, the CS is paired with a second US until a second CR is established. D. A CS is paired with an US until a CR is established; subsequently, a second CS is paired with the first CS until it also elicits a CR.

A CS is paired with an US until a CR is established; subsequently, a second CS is paired with the first CS until it also elicits a CR. --Higher-order conditioning involves using a CS as an US in order to establish a CR for a second CS.

A researcher for a major drug company is going to test a new anti-depressant for market viability, why would the researcher choose a Practical Clinical Trial (PCT) over another method? Select one: A. A PCT is intended to answer a single, clearly formulated research question by concentrating on a major outcome of direct clinical significance. B. A PCT provides multiple answers which gives it a complex depth for which a researcher may analyze for more effective results. C. A PCT tends to provide a clear, formulated answer that the researcher can utilize to demonstrate cause and effect between the three major variables being studied. D. A PCT is a rich and complex system of analysis for which a researcher may utilize to test multiple interventions.

A PCT is intended to answer a single, clearly formulated research question by concentrating on a major outcome of direct clinical significance. --PCTs tend to be large, simple trials with many hundreds, if not thousands, of patients. PCTs frequently provide relevant information on common conditions, such as depression or anxiety, or the availability of a variety of treatments, both pharmacological and psychosocial.

From the perspective of object-relations theory, projective identification is illustrated by which of the following? Select one: A. A woman who doesn't want to own her feelings of love and hate manipulates another person into experiencing them. B. A child kicks and pushes his mother away from him even though he actually longs for her attention. C. A man continuously interprets his wife's behavior as an attempt to "mother" him. D. A therapist reacts to lower-SES clients in ways consistent with stereotypes she formed during her internship.

A woman who doesn't want to own her feelings of love and hate manipulates another person into experiencing them. --Projective identification underlies a number of pathological conditions and is responsible for severe difficulties in establishing one's own identity and in feeling secure enough to establish relationships with others.

The Publication Manual of the American Psychological Association states that some abbreviations do not require explanation in text. These include all of the following except: Select one: A. IQ. B. HIV. C. APA. D. ESP

APA --You may have been able to recognize this as the correct answer as long as you recalled that "APA" can stand for several organizations including the American Psychological Association and the American Psychiatric Association. Therefore, you would need an explanation the first time that it's used. The abbreviations listed in the other answers are provided in the Publication Manual as examples of abbreviations that do not require an explanation.

The word HEART is projected onto a screen so that HE is projected to the subject's left visual field only while ART is projected to the subject's right visual field only. If the subject in the experiment is a "split brain" patient, you would expect him/her to report seeing: Select one: A. HE only. B. ART only. C. ART HE. D. HE ART.

ART only --This is the result reported by Sperry (1970). Because ART was projected to the patient's right visual field, it was processed by the left hemisphere which is specialized for language in most people. Thus, a split-brain patient could report seeing ART.

Research on relapse following smoking cessation has found which of the following to be true? Select one: A. On the average, smokers gain 17.5 pounds during the first year after quitting smoking. B. Although many smokers want to quit, only about 9% successfully quit each year. C. Smoking cessation interventions are significantly more effective for women than for men. D. About two-thirds of smokers who quit on their own or with the assistance of an intervention relapse within three months of quitting.

About two-thirds of smokers who quit on their own or with the assistance of an intervention relapse within three months of quitting.

Your new client is a 20-year-old college student who says she's failing her classes because of her anxiety. She states that she has to force herself to leave her dorm room to go to class, and when she does, she avoids the "quad" and other large open spaces because she's afraid she'll have a panic attack. The client tells you that, when in large open spaces, she can't breathe and her heart races, she feels like she's going to die, and she's afraid that no one will help her if she has an attack. She says that she experiences the same problem when she's part of a crowd and avoids going anywhere where there's likely to be a "lot of people," which has made it hard for her to have a social life. Based on this information, the most likely DSM-5 diagnosis for this client is: Select one: A. Panic Disorder with Agoraphobia B. Agoraphobia C. Agoraphobia without History of Panic Disorder D. Specific Phobia, situational type

Agoraphobia --A DSM-5 diagnosis of Agoraphobia requires the presence of marked fear of or anxiety about at least two of five situations (using public transportation, being in open spaces, being in enclosed spaces, standing in line or being part of a crowd, and being outside the home alone). The individual fears or avoids these situations due to a concern that escape might be difficult or help will be unavailable in case he/she develops incapacitating or embarrassing symptoms; and the situations nearly always provoke fear or anxiety and are actively avoided, require the presence of a companion, or are endured with intense fear or anxiety.

The ApoE4 gene on chromosome 19 has been linked to an increased risk for: Select one: A. Alzheimer's disease. B. Huntington's disease. C. diabetes mellitus. D. multiple sclerosis.

Alzheimer's disease --Research has identified a link between Alzheimer's disease and specific genes on several chromosomes including chromosome 19. The ApoE4 gene on chromosome 19 has been associated with an increased risk for both heart disease and late-onset Alzheimer's disease. Note that, even if you were not familiar with the ApoE4 gene, knowing that Alzheimer's disease has been linked to abnormalities on chromosome 19 would have helped you identify this as the correct answer.

Surveys have found that women in which of the following groups report the highest lifetime rate of victimization by an intimate partner? Select one: A. White Americans B. African Americans C. Asians/Pacific Islanders D. American Indians/Alaska Natives

American Indians/Alaska Natives --Tjaden and Thoennes report a lifetime rate for American Indians/Alaska Natives of 37.5%. They note that this higher rate for American Indian/Alaska Native women is consistent with previous research but state that it is not clear whether the rate is actually due to more violent experiences or an increased willingness to report them.

The research suggests that a person with Alcohol Use Disorder is most likely to meet the diagnostic criteria for which of the following? Select one: A. Borderline Personality Disorder B. Paranoid Personality Disorder C. Antisocial Personality Disorder D. Avoidant Personality Disorder

Antisocial Personality Disorder --Most studies have found that Antisocial and Borderline Personality Disorders are the personality disorders that co-occur most frequently with Alcohol Use Disorder, with Antisocial Personality Disorder being more common than Borderline Personality Disorder.

Tom, a 35-year-old television repairman, becomes extremely upset whenever his boss and co-workers say something unfavorable about his work and is very uncomfortable whenever he feels he is being watched at work. He reports having only one close friend who he says he is very afraid of losing. Tom rarely leaves the house except to go to work, and he tells you that one of the reasons is because he is not a very good driver and he fears he'll get into a car accident. Based on this information, the best diagnosis for Tom is: Select one: A. Schizoid Personality Disorder. B. Schizotypal Personality Disorder. C. Avoidant Personality Disorder. D. Dependent Personality Disorder.

Avoidance Personality Disorder --Based on the information given, the best diagnosis is Avoidant Personality Disorder, which entails sensitivity to criticism, avoidance of social activities, embarrassment, and distress at the inability to form close personal relationships.

During the first few sessions with a male client, you learn that he has trouble maintaining friendships and is disturbed by this since he doesn't like being alone, has frequent mood shifts, gets very angry about even minor irritations, isn't sure what he wants to do with his life, and has had three different jobs in the past two years. Based on this information, the best diagnosis is: Select one: A. Borderline Personality Disorder. B. Narcissistic Personality Disorder. C. Histrionic Personality Disorder. D. Schizoid Personality Disorder.

Borderline Personality Disorder --The pattern of instability in mood, relationships, and occupation is characteristic of Borderline Personality Disorder.

Tetraplegia is most likely to be the outcome of spinal cord injury at the ________ level. Select one: A. T9-T12 B. L2-L5 C. S3-S5 D. C1-C7

C1-C7 --To identify this as the correct answer, you would have to know that tetraplegia is now the more commonly used term for quadriplegia and that it is the result of injury at the cervical level.

_________ is most often used to initially diagnose a cerebral contusion because it is usually available in medical settings, is cost-effective, and can be performed quickly. Select one: A. MRI B. CT C. SPECT D. PET

CT --A cerebral contusion is bruising of the brain that is caused by a head injury. A CT scan is most commonly used to initially detect a contusion for the reasons stated in this question.

A child with Autism Spectrum Disorder without language impairment would be expected to obtain the lowest score on which of the following WISC-V subtests? Select one: A. Block Design B. Matrix Reasoning C. Similarities D. Coding

Coding --This is a difficult question, but knowing that children with Autism Spectrum Disorder without language impairment tend to obtain the lowest scores on tests involving processing speed would have helped you identify the correct answer. Children with Autism Spectrum Disorder without language impairment tend to obtain the lowest scores on the Coding and Symbol Search subtests of the WISC-V which are two of the three subtests that contribute to the Processing Speed Index

To use two or more categorical variables to predict status on a single categorical variable, you would use which of the following? Select one: A. path analysis B. logit analysis C. multiple regression analysis D. canonical correlation analysis

D. logit analysis --Logit analysis is the appropriate multivariate technique in this situation. If you're familiar with the multivariate techniques described in the Statistics and Research Design chapter, you may have been able to identify the correct answer to this question through the process of elimination.

Your client is convinced that, for the past three months, the government has been tapping his phone, reading his emails, and having someone follow him. He states that he believes that, at one point, he was drugged by "an agent" and was "out for a day or two." The client works as a computer programmer for a medical records company, has been married for three years, and has one child. He says that, despite the government's interference in his life, everything else seems to be okay. What is the most likely DSM-5 diagnosis for this client? Select one: A. Paranoid Personality Disorder B. Brief Psychotic Disorder C. Delusional Disorder D. Schizophrenia

Delusional Disorder

Augustine, age 5, is in foster care and enrolled in a therapeutic nursery school. He and his older sister were removed from their home after they were found wandering in the park asking people for food and their parents were both found to have a drug problem. While observing Augustine at school, you notice that he impulsively approaches strangers, tends to be overly familiar with people he has just met, and quickly becomes attached to any adult who pays attention to him. Before assigning a DSM-5 diagnosis of ______________ to Augustine, you will want to confirm that his symptoms are attributable to ______________. Select one: A. Reactive Attachment Disorder, disinhibited type; developmental delays B. Disinhibited Social Engagement Disorder; developmental delays C. Reactive Attachment Disorder, disinhibited type; early neglect or deprivation D. Disinhibited Social Engagement Disorder; early neglect or deprivation

Disinhibited Social Engagement Disorder; early neglect or deprivation --Disinhibited Social Engagement Disorder involves a pattern of culturally inappropriate and overly familiar behavior with unfamiliar people with evidence that the behavior is related to the experience of extremely insufficient care.

Jacob, age 14, is being seen by a school clinician for repeated angry outbursts at school. He is persistently irritable, constantly argues with adults, has a history of behavioral referrals, and recently changed schools after being expelled for threatening a teacher. Jacob's parents also report outbursts at home. He is failing in school, and his parents are thinking of sending him to a wilderness camp where he can "get some sense knocked into him." The most likely DSM-5 diagnosis for Jacob is which of the following? Select one: A. Intermittent Explosive Disorder B. Oppositional Defiant Disorder C. Bipolar II Disorder D. Disruptive Mood Dysregulation Disorder

Disruptive Mood Dysregulation Disorder --Disruptive Mood Dysregulation Disorder involves severe, recurrent temper outbursts with a persistent irritable or angry mood between outbursts on most days. Oppositional Defiant Disorder differs from Disruptive Mood Dysregulation Disorder primarily in terms of the severity, frequency, and chronicity of the temper outbursts, with Disruptive Mood Dysregulation Disorder having more severe, frequent, and chronic symptoms. When the criteria for both disorders are met, a diagnosis of Disruptive Mood Dysregulation Disorder is assigned.

To assess the general intelligence of a six-year old child who is deaf, you would use which of the following tests? Select one: A. Halstead-Reitan B. Kaufman Assessment Battery for Children C. Fagan Test D. Hiskey-Nebraska

Hiskey-Nebraska --The Hiskey-Nebraska Test of Learning Aptitude was developed for use with deaf and hard-of-hearing children aged 3 to 17.

Which of the following is true about the use of hypnotically induced testimony in legal proceedings? Select one: A. Hypnotically induced testimony is permitted in all jurisdictions as long as the hypnotherapist is licensed or certified. B. Hypnotically induced testimony is permitted in all jurisdictions only when there is no contradictory evidence. C. Hypnotically induced testimony is permitted in some jurisdictions under specific conditions (e.g., when corroborating evidence is available). D. Hypnotically induced testimony is not permitted in any jurisdiction under any conditions.

Hypnotically induced testimony is permitted in some jurisdictions under specific conditions (e.g., when corroborating evidence is available). --Of the answers given, this one best describes the current status of the admissibility of hypnotically induced testimony in legal proceedings. Many jurisdictions have adopted a "per se exclusion" that excludes testimony about memories that first occurred during or after hypnosis. In other jurisdictions, a "totality of circumstances test" is used, which requires that certain guidelines or conditions be met so that it is unlikely that hypnosis impacted the individual's memory.

Use of which of the following designs involves administering each level of the independent variable to participants so that it appears the same number of times in each position (first, second, etc.)? Select one: A. factorial B. time-series C. Latin square D. Solomon

Latin Square --The Latin square design is a type of counterbalanced design that assures that each treatment (level of the IV) appears an equal number of times in each ordinal position.

Which of the following drugs are most effective for individuals who have received a diagnosis of Major Depressive Disorder with Atypical Features? Select one: A. MAOIs or SSRIs B. MAOIs or tricyclics C. SSRIs or tricyclics Incorrect D. tricyclics or neuroleptics

MAOIs or SSRIs --The choice of the best antidepressant for a particular individual depends on several factors including the nature of his/her symptoms. Atypical depression includes reversed vegetative symptoms (increase sleep and appetite), marked mood reactivity, phobic symptoms, and/or a sense of severe fatigue. The MAOIs and SSRIs have been found most effective for these symptoms.

A 33-year-old man tells his therapist that, since his involvement in a serious car accident two months ago, he has been feeling sad every day; has had no appetite; feels tired all of the time and sleeps for 10 to 12 hours each night; isn't interested in spending time with his friends and family; and feels guilty about the accident, even though it wasn't his fault. He also says that, because he's having trouble concentrating at work and getting to work on time, he's afraid he's going to be fired. Based on these symptoms, the most likely diagnosis is: Select one: A. Bipolar II Disorder. B. Major Depressive Disorder. C. Adjustment Disorder. D. Acute Stress Disorder.

Major Depressive Disorder --The nature, number, and duration of the man's symptoms are consistent with a diagnosis of Major Depressive Disorder: He is experiencing a depressed mood and has at least five other characteristic symptoms of this disorder; his symptoms have lasted for more than two weeks; and he is experiencing impaired occupational functioning as the result of his symptoms.

A therapist instructs a client who suffers from insomnia to polish his hardwood floors for at least two hours whenever he wakes up during the night. Apparently this therapist is familiar with the work of: Select one: A. Luigi Boscolo. B. Milton Erickson. C. Salvador Minuchin. D. Marquis de Sade.

Milton Erickson

Which of the following best describes the relationship between REM sleep and dreaming? Select one: A. All dreaming occurs during REM sleep. B. Most dreaming occurs during REM sleep and dreams during non-REM sleep are less vivid and elaborate and more realistic. C. Most dreaming occurs during REM sleep but dreams during non-REM sleep are more vivid, elaborate, and bizarre in nature. D. Most dreaming occurs during REM sleep and dreams during non-REM sleep occur only when the individual has been sleep-deprived.

Most dreaming occurs during REM sleep and dreams during non-REM sleep are less vivid and elaborate and more realistic. --Although some dreaming occurs during non-REM (Stages 1 through 4) sleep, these dreams consist of fleeting images and are less vivid, detailed, and storylike than the dreams that occur during REM sleep.

Which of the following are related to the psychology of oppression? Select one: A. Strength, learning, knowledge, and cultural individuality. B. Perceptions, willpower, self-surrender, and power status. C. Apathy, individualism, loss of identity, and group think. D. Motivation, emotions, ambitions, and ideals.

Motivation, emotions, ambitions, and ideals. --The psychology of oppression consists of: motivation, agency, perception, emotions, ambitions, ideals, reasoning, memory, aesthetics, and morals that accept the oppressive social system, desire it, identify with it, take it for granted as normal and even as ideal, take pleasure in it, defend it, and reject alternatives to it. Answers A, B and C are incorrect as none of these are related to the psychology of oppression other than perception.

Whenever a mother yells at her child when he is whining, the boy stops whining for a short period. Over time, the mother notices that she's having to yell at her son more and more often to keep the boy from whining. The boy is controlling his mother's behavior through: Select one: A. positive reinforcement. B. negative reinforcement. C. positive punishment. D. negative punishment.

Negative reinforcement --Note that this question is asking about the mother's behavior (yelling), not the boy's behavior. To determine what is occurring, you have to determine (1) whether the mother's yelling is increasing or decreasing and (2) what occurs after the mother yells: The mother's yelling is increasing, which indicates that it is being reinforced. And the consequence following her yelling is removed (i.e., the boy temporarily stops whining). This indicates that the reinforcement is negative. This situation is an example of negative reinforcement. The mother's behavior is increasing because a stimulus is removed after the behavior occurs.

What is a unique challenge that comes from using Evidence-Based Therapies in public mental health services settings? Select one: A. No universal guidelines provided by the government. B. No standardized supervision or training literature. C. No procedures for selecting supervisors or trainers. D. No testing applications to verify licensing authenticity.

No standardized supervision or training literature. --A significant portion of the training and supervision literature within the field of clinical psychology does not often address the unique challenges that arise in training in the public mental health service settings. A comprehensive model for providing training in EBTs that can apprise local and larger scale implementation and facilitate training partnerships with publicly funded or community-based agencies and practitioners of EBTs has been demonstrated to be most effective.

PET scans of people with _______________ show that these individuals often have increased activity levels in the orbitofrontal cortex, cingulate cortex, and caudate nucleus. Select one: A. Posttraumatic Stress Disorder B. Narcolepsy C. Obsessive-Compulsive Disorder D. Schizophrenia

Obsessive-Compulsive Disorder --Knowing that the caudate nucleus is one component of the basal ganglia and that the basal ganglia are involved in voluntary movement and that the orbitofrontal cortex and cingulate cortex mediate emotional reactions may have helped you identify the correct response to this question. Activity in these areas of the brain is higher in individuals with OCD, especially during provocation of symptoms. In contrast, drug and behavioral treatments for OCD reduce activity in these areas.

Which of the following is NOT true about the effects of crowding? Select one: A. Men are more negatively affected than women by crowded conditions. B. Increasing age is associated with an increasing negative impact of crowding. C. People are more willing to discuss intimate details in crowded (versus uncrowded) conditions. D. Crowded (versus uncrowded) conditions are associated with a higher risk for health-related problems.

People are more willing to discuss intimate details in crowded (versus uncrowded) conditions. --This is the opposite of what is true -- people are less willing to discuss intimate topics and to seek social support in crowded situations.

George Kelly is associated with which of the following? Select one: A. Reality Therapy B. Personal Construct Therapy C. Existential Therapy D. Solution Focused Therapy

Personal Construct Therapy --Personal Construct Therapy (also known as "constructive alternativism") is based on the premise that people construe (construct) their own experiences. His approach was very influential in the development of narrative-constructivist approaches to therapy.

During her first session, Jane, age 43, tells the clinician that she needs to work on anxiety and relationship issues. She says she divorced her second husband 11 months ago and has been dating a man she met at church for a month but thinks he's going to "dump" her because of her physical problems. Jane has Type 2 diabetes, which is well-managed with medication. However, she complains of frequent stomach problems, severe headaches, and back pain and says she spends a lot of time researching diseases on the Internet to try to figure out what's wrong with her besides diabetes. Jane has been evaluated by a physician who told her that, other than diabetes, she does not have any medical problems. What DSM-5 diagnosis should the clinician consider first? Select one: A. Somatic Symptom Disorder B. Factitious Disorder C. Illness Anxiety Disorder D. Psychological Factors Affecting Other Medical Condition.

Somatic Symptom Disorder --Jane's symptoms are consistent with Somatic Symptom Disorder, which is included in the DSM-5 with Somatic Symptom and Related Disorders. The essential feature of this disorder is the presence of multiple somatic symptoms that cause distress or disruption in one's daily life.

Implosive therapy would be most useful as a treatment for which of the following? Select one: A. Specific Phobia B. Major Depressive Disorder C. Hypoactive Sexual Desire Disorder D. Reactive Attachment Disorder

Specific Phobia --Knowing that implosive therapy is used to extinguish a phobic or other anxiety response would have enabled you to identify the correct answer to this question. Implosive therapy combines classical extinction with psychodynamic interpretation and is used to treat anxiety reactions.

A school psychologist wants to determine if there is a significant difference in reading readiness scores between male and female students in the school's preschool program. She obtains scores on a standardized reading readiness test for 17 girls and 13 boys. Which statistical test will be most appropriate for determining if there is a significant difference between the scores obtained by boys and girls? Select one: A. two-way ANOVA B. Student's t-test C. Kolmogorov test D. chi-square test for contingency tables

Student's t-test --A t-test (a.k.a. Student's t-test) is used to compare the mean scores obtained by two groups.

An adult with traumatic brain injury is likely to obtain the lowest scores on which of the following WAIS-IV subtests? Select one: A. Symbol Search, Coding, and Cancellation B. Block Design, Visual Puzzles, and Picture Completion C. Digit Span, Arithmetic, and Letter-Number Sequencing D. Vocabulary, Similarities, and Information

Symbol Search, Coding, and Cancellation --WAIS-IV Manual indicates that individuals in several clinical groups -- e.g., traumatic brain injury, mild Alzheimer's dementia, and ADHD -- obtained the lowest score on the Processing Speed Index, which consists of the Symbol Search, Coding, and Cancellation subtests.

Bobby B. obtains a test score of 110. The test has a mean of 120 and a standard deviation of 10 and test scores have a range of 100 and are normally distributed. If Bobby's teacher converts all of the students' test scores to T-scores or z-scores, Bobby's score will be which of the following? Select one: A. T = 60; z = +1.0 B. T = 90; z = -1.0 C. T = 40; z = -2.0 D. T = 40; z = -1.0

T = 40; z = -1.0 --Knowing that the T-score distribution has a mean of 50 and standard deviation of 10 and that the z-score distribution has a mean of 0 and standard deviation of 1 would have helped you identify the correct answer to this question. Since Bobby's raw score is one standard deviation below the mean, his T-score would be 40 and his z-score would be -1.0. (In the T-score distribution, a score of 40 is one standard deviation below the mean and, in the z-score distribution, a z-score of -1.0 is one standard deviation below the mean.)

When using the DSM-5, level of severity of Intellectual Disability is based on: Select one: A. The individual's score on a standardized intelligence test. B. The individual's adaptive functioning in conceptual, social, and practical domains. C. The degree of discrepancy between the individual's cognitive and adaptive functioning. D. The degree of discrepancy between the individual's IQ and academic achievement.

The individual's adaptive functioning in conceptual, social, and practical domains. --The DSM-5 distinguishes between four levels of severity for Intellectual Disability - mild, moderate, severe, and profound - and bases them on level of adaptive functioning in conceptual, social, and practical domains.

Increasing age is LEAST likely to have which of the following effects on a man's sexual response cycle? Select one: A. The time to achieve an erection will increase. B. Complete penile erection may not occur until just prior to orgasm. C. The resolution phase and refractory period will both increase. D. All of the above occur.

The resolution phase and refractory period will both increase. --although the refractory period begins to increase in the 30s or 40s (and may last for several hours by age 60), the resolution phase actually decreases, with loss of erection following orgasm being quite rapid.

Which of the following statements is most consistent with the current understanding of postconcussional syndrome (PCS)? Select one: A. The more severe the head trauma, the more severe and the longer the duration of the symptoms of PCS. B. Most patients who develop PCS never fully recover from the motor and other physical symptoms of the disorder. C. The symptoms of PCS are often due to a combination of organic and psychological factors. D. In most cases, PCS is purely psychological in nature and motivated by a desire for financial compensation

The symptoms of PCS are often due to a combination of organic and psychological factors. --Although the etiology of PCS continues to be debated, the experts generally agree that it has both physiological and psychological origins. Psychological factors that contribute to PCS include the individual's subjective interpretation of the injury, premorbid personality characteristics, desire for secondary gain, and access to social support.

Difficulty in repeating words just spoken by another person and recalling the name of a familiar object are characteristic symptoms of: Select one: A. Wernicke's aphasia only. B. Wernicke's and conduction aphasia only. C. Wernicke's and Broca's aphasia only. D. Wernicke's, Broca's, and conduction aphasia.

Wernicke's, Broca's, and conduction aphasia. --All three types of aphasia involve the two symptoms listed in the question (difficulty in repeating words and recalling the names of familiar objects). Note that conduction aphasia is due to damage to nerve fibers that connect Broca's area to Wernicke's area.

A woman is having trouble with her six-month old son who often "fusses" at bedtime and wakes up at least once or twice during the night. She does an Internet search on the problem and finds several articles about Dr. Richard Ferber. She learns that his method for getting babies to sleep ("Ferberizing") is supported by many experts and involves: Select one: A. a "progressive-waiting" approach. B. therapeutic touch. C. a "cold turkey" approach. D. co-sleeping.

a "progressive-waiting" approach. --Ferber's method is similar to that promoted by a number of other experts. When using Ferber's method, the parent puts the child to bed. If he cries, the parent returns and reassures the child and pats him on the back but does not pick him up. If the child cries again later, the parent waits a little longer before returning. On subsequent nights, the wait period is gradually increased, which is why the method is known as the progressive waiting method.

A movie viewer is MOST likely to say he/she feels uncomfortable in a crowded movie theater when the movie is: Select one: A. a violent action film. B. a sexually arousing film. Incorrect C. a boring documentary. D. an amusing comedy.

a boring documentary

A partial seizure is characterized by: Select one: A. a focal onset in one hemisphere of the brain with or without a loss of consciousness. B. a focal onset in one hemisphere of the brain without a loss of consciousness. C. a focal onset in one hemisphere of the brain with a loss of consciousness. D. a simultaneous onset in both hemispheres of the brain without a loss of consciousness.

a focal onset in one hemisphere of the brain with or without a loss of consciousness. --Partial seizures begin in one hemisphere and affect movement and sensations beginning on one side of the body. Simple partial seizures do not affect consciousness, while complex partial seizures do alter consciousness.

: In Albert Ellis's (1985) A-B-C model, B refers to: Select one: A. the environmental barriers to rational thought. B. the biological bases for human behavior. C. a person's belief about an activating event. D. a person's behavioral reaction to an activating event.

a person's belief about an activating event. --Ellis's A-B-C model is central to the theory and practice of his Rational-Emotive Behavior Therapy. Ellis described emotions and behaviors in terms of a chain of events -- an external activating event (A), the person's belief about the activating event (B), and the emotional or behavioral consequence of that belief (C).

Fritz Heider's (1958) balance theory proposes that the relationship between three entities can be either balanced or unbalanced. These entities are: Select one: A. a person, another person, and an attitude object. B. the intrapersonal self, the interpersonal self, and the agent self. C. attitudes, cognitions, and behaviors. D. the person's past, present, and future goals.

a person, another person, and an attitude object --Balance theory is a cognitive consistency theory that predicts that a person is motivated to change an attitude when he/she experiences inconsistencies in attitudes. Balance theory is also known as P-O-X theory, where p is the person, O is another person, and X is the attitude object.

An existential therapist is most likely to describe "existential anxiety" as: Select one: A. the result of unresolved intrapsychic conflicts. B. a potential stimulus for growth. C. the result of a boundary disturbance. D. a manifestation of incongruence between self and experience.

a potential stimulus for growth. --Existential therapists view anxiety as an inevitable condition of life and distinguish between normal and neurotic anxiety. For existential therapists, neurotic anxiety is out of proportion to the situation and can be immobilizing. In contrast, normal (existential) anxiety can serve as a source of motivation to change.

Malnutrition during fetal development is likely to have which effect on the brain? Select one: A. scarring due to the death of brain cells B. synaptic enlargement C. malformation of the brain cells D. a reduced number of brain cells

a reduced number of brain cells --The most consistent finding of the research is that malnutrition during fetal development results in a reduced number of brain cells. There is also some evidence that there are problems in myelination.

Patients who have recently started taking naltrexone (ReVia) as a treatment for alcohol dependence are most likely to experience which of the following drug side effects? Select one: A. dizziness, ataxia, visual disturbances, nausea, and rash B. blurred vision, sexual dysfunction, weight gain, edema, and tremor C. shortness of breath, increased dreaming, nausea, diarrhea, and bradycardia D. abdominal cramping, nausea, insomnia, nervousness, and headache

abdominal cramping, nausea, insomnia, nervousness, and headache --These are potential side effects of naltrexone. Note, however, that most people experience few side effects when taking naltrexone as a treatment for alcohol dependence.

Dr. Bill sets his clients' fees on the basis of a "sliding scale" that is based on their current income. This practice is: Select one: A. acceptable but not explicitly mentioned in the Ethics Code. B. unacceptable but not explicitly mentioned in the Ethics Code. C. explicitly recommended in the Ethics Code. D. explicitly prohibited in the Ethics Code.

acceptable but not explicitly mentioned in the Ethics Code.

As defined in the APA's Ethics Code, uninvited in-person solicitation of business by a psychologist is: Select one: A. acceptable as long as the solicitation is not coercive. B. acceptable when it involves providing disaster or community outreach services. C. acceptable as long as the person's "best interests" are of paramount concern. D. never acceptable.

acceptable when it involves providing disaster or community outreach services. --Standard 5.06 generally prohibits psychologists from engaging in uninvited in-person solicitation of business. However, it also notes that the prohibition does NOT preclude psychologists from "providing disaster or community outreach services."

With regard to power in the client-therapist relationship, a feminist therapist would most likely: Select one: A. recognize that her power stems primarily from her ability to serve as a role-model. B. acknowledge the power differential inherent in the relationship but view it as a temporary condition. C. initially foster her position of power so as to maximize her influence. D. regard power a "non-issue" because feminist therapists consider the therapeutic relationship to be egalitarian.

acknowledge the power differential inherent in the relationship but view it as a temporary condition. --According to Douglas, the power differential inherent in the therapeutic relationship should, first of all, be acknowledged. It should also be viewed as temporary since the goal of therapy will be to terminate the relationship and/or to readjust the relationship so that it is more egalitarian. Douglas stresses the importance of minimizing the power differential inherent in the therapeutic relationship.

Coping strategies are an important contributor to adaptation to chronic pain, and research on coping strategies suggests that: Select one: A. active and passive coping strategies are equally effective for improving the physical and psychological functioning of those with chronic pain. B. active coping strategies are more effective than passive strategies for improving the physical and psychological functioning of those with chronic pain. C. passive coping strategies are more effective than active strategies for improving the physical and psychological functioning of those with chronic pain. D. active coping strategies are more effective for improving the physical functioning of those with chronic pain, while passive coping strategies are more effective for improving their psychological functioning.

active coping strategies are more effective than passive strategies for improving the physical and psychological functioning of those with chronic pain --The research is not entirely consistent, but has generally found that active coping strategies are associated with greater improvement in psychological and physical functioning. A. L. Snow-Turek, N. P. Norris, and G. Tan, for example, found that passive strategies were strongly associated with depression and other forms of psychological distress, while active strategies were inversely related to psychological distress

Longitudinal studies of individuals who receive a diagnosis of Borderline Personality Disorder in adolescence or early adulthood indicate that these individuals often exhibit a reduction or remission in symptoms over time. However, recovery varies for type of symptom, with _________ symptoms showing the least amount of improvement with increasing age. Select one: A. impulsive B. interpersonal C. affective D. cognitive

affective --Longitudinal studies have found that, by middle age or sooner, most individuals with BPD no longer meet the diagnostic criteria for the disorder. The resolution of symptoms varies, however: For example, in a six-year prospective study of 290 patients with BPD, Zanarini et al. found that impulsive symptoms resolved most quickly, affective symptoms were the most chronic, and cognitive and interpersonal symptoms were intermediate in terms of resolution

Klinefelter's syndrome is a genetic disorder that: Select one: A. affects males and is caused by the presence of an extra X chromosome. B. affects males and is caused by the presence of an extra Y chromosome. C. affects females and is caused by the presence of an extra X chromosome. D. affects females and is caused by the presence of an extra Y chromosome.

affects males and is caused by the presence of an extra X chromosome. --A male with Klinefelter's syndrome has two or more X (female) chromosomes in addition to a single Y chromosome. Symptoms include small testicles, infertility, enlarged breast tissue, scant facial/body hair, and a decreased libido.

Schaie's (1996) Seattle Longitudinal Study examined the relationship between cognitive abilities and: Select one: A. socioeconomic status. B. speed of processing C. age. D. gender.

age --Schaie's Seattle Longitudinal Study investigated age-related changes in cognitive abilities. In contrast to earlier cross-sectional studies which indicated that increasing age in adulthood is associated with declines in many cognitive abilities, Schaie's study found that IQ remains relatively stable during adulthood and that substantial age-related declines prior to age 70 are apparent for only two abilities - perceptual speed and numerical ability.

Dr. E. Dict has been seeing Jay Bird, a prisoner at the local jail, in therapy for the past five months. The parole board contacts Dr. Dict to request that he evaluate Bird for an upcoming parole hearing. Dr. Dict should: Select one: A. agree to evaluate Bird since he is best qualified to evaluate him fairly. B. agree to evaluate Bird only if Bird agrees to the arrangement. C. agree to evaluate Bird only if he feels that Bird is eligible for parole. D. refuse to evaluate Bird for the parole hearing.

agree to evaluate Bird only if he feels that Bird is eligible for parole. --This type of situation is addressed in the Specialty Guidelines for Forensic Psychology and the Ethics Code, which require psychologists to avoid dual relationships whenever objectivity, exploitation, and other issues related to effectiveness are of concern, which would be the case in this situation. The two roles -- therapist and forensic evaluator -- clearly conflict and should be avoided.

According to Sternberg's (1998) triarchic theory, intelligence consists of which of the following three factors? Select one: A. general, broad, and specialized B. analytical, creative, and practical C. attention, processing, and planning D. general, fluid, and crystallized

analytical, creative, and practical --Sternberg's theory distinguishes between the three facets of intelligence listed in this answer. According to Sternberg, analytical intelligence is the componential facet of intelligence and consists of metacomponents, performance components, and knowledge-acquisition components; creative intelligence is the experiential facet; and practical intelligence is the contextual facet.

The DSM-5 diagnoses of Bipolar I and Bipolar II Disorder both require that the person has had at least one episode involving an abnormally persistent elevated, expansive, or irritable mood: Select one: A. and persistently increased goal-directed activity or energy. B. with a lack of awareness of his/her illness and the need for treatment. C. that is sufficiently severe to cause a marked impairment in functioning or require hospitalization. D. and at least one episode involving a depressed mood or a loss of interest or pleasure in all or most activities.

and persistently increased goal-directed activity or energy. --The DSM-5 diagnostic criteria for Bipolar I and Bipolar II disorder include at least one episode involving an abnormally persistent elevated, expansive, or irritable mood with the presence of persistently increased goal-directed activity or energy.

Research on the role of hormones on sexual arousal suggests that: Select one: A. estrogen is responsible for arousal in females, while androgen is responsible for arousal in males. B. estrogen is responsible for arousal in males, while androgen is responsible for arousal in females. C. estrogen is responsible for arousal in both males and females. D. androgen is responsible for arousal in both females and males.

androgen is responsible for arousal in both females and males --The research suggests that estrogen plays an insignificant role in female sexual motivation and arousal and that androgen, which is produced by the adrenal cortex, plays an important role in the sexual functioning of both males and females.

Freud argued that the "work of the mental apparatus is directed toward keeping the quantity of excitation low." If the mental apparatus is unsuccessful in doing so, the result is likely to be which of the following? Select one: A. anxiety B. ego decompensation C. psychosis D. object-cathexis

anxiety --The function of the ego's defense mechanisms, for example, is to keep the conflicts that produce anxiety out of consciousness. Excessive excitation by the id's impulses, according to Freud, leads to anxiety.

Dr. Tartuffe is conducting a survey to obtain the information needed to compare therapists who have and have not had a sexual relationship with a client. The results of his survey will most likely indicate that therapists who say they have been sexually involved with a client: Select one: A. are also more likely to have been sexually involved with their own supervisor. B. are also more likely to have had non-sexual dual relationships with their clients. C. are also more likely to make use of therapeutic touch in therapy. D. are not distinguishable from those who have not been sexually involved with a client in terms of sex with their own supervisor, non-sexual dual relationships, or the use of therapeutic touch

are also more likely to have had non-sexual dual relationships with their clients. --Lamb and Catanzaro found that psychologists who had sex with their clients were no more likely to have had sex with their supervisors than psychologists who did not have sex with their clients. However, psychologists reporting sexual relationships with their clients also reported more nonsexual dual relationships. (The issue of therapeutic touch was not examined in the survey, and there is no evidence to support response c.)

In comparison to tricyclic antidepressants, fluoxetine and other SSRIs: Select one: A. are more cardiotoxic. B. are less likely to negatively affect sleep and anxiety symptoms. C. are less likely to cause cognitive impairments. D. are more effective for positive symptoms.

are less likely to cause cognitive impairments --Another benefit of the SSRIs is that they are not associated with cognitive impairments (the tricyclics are).

When attempting to expand your private practice, it is important to keep in mind that in-person solicitations: Select one: A. are always unethical. B. are unethical only when they target individuals receiving similar services from another professional. C. are unethical only when they include deceptive or misleading information. D. are unethical when they include deceptive information or when the person solicited is susceptible to undue influence.

are unethical only when they include deceptive or misleading information. --This is most consistent with Standard 5.06 of the Ethic Code: Solicitations are unethical when they include deceptive or false information or when their targets are individuals who are vulnerable to undue influence.

Melanie Klein considered the actions of children during play therapy to be equivalent to an adult client's free associations. Anna Freud, in commenting on Klein's conclusions: Select one: A. agreed that play therapy could be used as a substitute for free association. B. argued that the actions of children during play therapy are better considered a form of "acting out." C. argued that the actions of children during play therapy are better interpreted as a form of resistance. D. concluded that play therapy is counterproductive in the analysis of children.

argued that the actions of children during play therapy are better considered a form of "acting out." --In her paper "Acting Out" (The Writings of Anna Freud, Volume IX), Freud notes that, in pre-latency children, motor actions are the legitimate media of expression and communication of impulses; and, consequently, during that period, play is a form of "acting out."

The Strong Interest Inventory would probably be LEAST useful: Select one: A. as a job selection test to predict future performance. B. as a guide for choosing a college major. C. as a guide for choosing a career. D. as a tool for predicting job turnover.

as a job selection test to predict future performance --Research has found interest tests to be good predictors of academic and job choice and job persistence and satisfaction which are listed as the other alternatives. However, they are not accurate predictors of job performance.

During the first session with an American Indian family consisting of a grandmother, mother, father, and three children, an Anglo-American therapist would be best advised to: Select one: A. refer the family to an American Indian therapist. B. ensure that she establishes a "value-free" environment. C. assess each family member's level of acculturation before developing a treatment plan. D. treat the family like any other family unless there is a clear reason to do otherwise.

assess each family member's level of acculturation before developing a treatment plan. --An initial step when working with clients from culturally diverse groups is to identify the client's stage of racial/ethnic identity development, degree of acculturation, and worldview (beliefs, values, attitudes, etc.).

Dr. Burgher is the only psychologist in a small town. His daughter's girl scout leader wants to begin seeing Dr. Burgher to work through some family-related problems. As an ethical psychologist, Dr. Burgher should: Select one: A. refuse to see the woman because to do so would constitute a "multiple relationship." B. see the woman since he is the only psychologist in town and her problems are unrelated to her role as girl scout leader. C. determine the seriousness of the woman's problems and base his decision about seeing the woman on the outcome of that evaluation. D. assess the potential for exploitation and loss of objectivity and base his decision about seeing the woman on the outcome of that assessment.

assess the potential for exploitation and loss of objectivity and base his decision about seeing the woman on the outcome of that assessment. --This is the best answer of those given because it reiterates the actual language of the Ethics Code.

In response to questions included in the Adult Attachment Interview, a 34-year old mother describes her childhood relationships with her own parents as very positive, but she is unable to come up with specific examples of pleasant experiences with them. Although she says this inability is due to poor memory, she is able to recall other events that occurred during early childhood. Most likely, this woman's 13-month old baby will exhibit which of the following attachment patterns in the Strange Situation? Select one: A. secure B. avoidant C. resistant D. disorganized/disoriented

avoidant --Parents categorized as "dismissing" describe their relations with parents in positive terms but are unable to recall specific examples that support their descriptions. They most often have children who exhibit avoidant attachment in the Strange Situation.

Research using brain imaging techniques has confirmed that repeated exposure to alcohol during prenatal development is most likely to negatively impact the: Select one: A. hypothalamus, thalamus, and parietal lobes. B. medulla, thalamus, and temporal lobes. C. basal ganglia, hippocampus, and frontal lobes. D. brainstem, amygdala, and frontal lobes.

basal ganglia, hippocampus, and frontal lobes. --FAS is characterized by a combination of behavioral and cognitive symptoms including an intellectual disability, impaired attention and memory, hyperactivity, impaired motor coordination, impulsivity, and poor judgment. The areas of the brain listed in this answer mediate some of these functions and are often adversely affected by prenatal exposure to alcohol. Other areas that are commonly affected include the corpus callosum, cerebellum, and hypothalamus.

The ___________ refers to the tendency to ignore relevant statistical data when making a probability judgment and to rely, instead, on irrelevant information. Select one: A. Von Restorff effect B. Forer effect C. availability heuristic D. base rate fallacy

base rate fallacy --This question accurately describes the base rate fallacy, which is also known as base rate neglect. In one study on this phenomenon, participants were given data about the distribution of GPA in the population and then asked to estimate the GPA of hypothetical students. Rather than relying on the statistical information, participants usually used irrelevant descriptive information about each hypothetical student to estimate the student's GPA.

If a psychology licensure candidate is believed to have cheated on the psychology licensing exam, he/she will: Select one: A. be allowed to re-take the exam but will be monitored during the process. B. be barred from re-taking the exam for a period of no less than three years. C. be given an opportunity to provide evidence to the licensing board why his/her test score should not be cancelled. D. be required by the Ethics Committee to appear at a formal hearing on the matter within six months.

be given an opportunity to provide evidence to the licensing board why his/her test score should not be cancelled.

An undergraduate student is asked by a professor to assist in a new research project. The student will help with the literature review and data analysis. According to the Ethics Code, when the research is published, the student should, at a minimum: Select one: A. be listed as a co-author. B. be mentioned in an introductory statement or a footnote. C. be mentioned in the abstract. D. receive a letter of recommendation from the professor.

be mentioned in an introductory statement or a footnote --At a minimum, the student should be acknowledged for his/her assistance in a footnote or introductory statement.

You have been seeing a couple in therapy for six months. The wife calls you and says she wants to start seeing you for individual sessions also. You should: Select one: A. bring up this possibility in the next couples session. B. tell her you cannot see her individually until couples therapy is over. C. refer her to another therapist for individual therapy. D. make an appointment with her to determine if her problem warrants individual therapy.

bring up this possibility in the next couples session. --Standard 10.02 of the Ethics Code makes it clear that, in couples therapy, psychologists must avoid potentially conflicting roles. One way to ensure that conflicts do not occur is to clarify one's policies and responsibilities. It may or may not be the psychologist's policy to provide individual therapy while also providing conjoint therapy, and this should have been clarified at the onset of therapy. Also, a referral may be appropriate, but the issue should first be discussed in conjoint therapy.

To assess the relationship between the linear combinations of two or more X variables and two or more Y variables, you would use which of the following? Select one: A. multivariate analysis of variance B. analysis of covariance C. discriminant function analysis D. canonical correlation

canonical correlation --To identify the answer to this question, you need to determine (1) whether you are interested in the relationship (correlation) between variables or the effects of one or more variables on another variable or variables; and (2) how many variables there are. If you were just guessing, this would have been the best guess (and the right answer) since it is the only one that contains the word "correlation" and the question states that you are interested in the RELATIONSHIP between variables.

Sherif's (1965) social judgment theory implies that, before trying to persuade someone to your point of view, you should consider his/her: Select one: A. categories of judgment. B. locus of control. C. category-based expectancies. D. level of self-monitoring.

categories of judgment --Sherif's theory distinguishes between three categories of judgment -- latitude of acceptance, latitude of non-commitment, and latitude of rejection. A person is most likely to be persuaded when a message is within his/her latitude of acceptance.

Research on Baddeley's (2000) multi-component model of working memory suggests that the ____________ plays an essential role in mental arithmetic and is responsible for accessing and executing computational algorithms and heuristics. Select one: A. sensory register B. central executive C. phonological loop D. visuo-spatial sketchpad

central executive --A. D. Baddeley's multi-component model of working memory describes working memory as being comprised of a central executive that controls and regulates three specialized "slave" systems - the phonological loop, the visuo-spatial sketchpad, and the episodic buffer. The central executive serves a number of supervisory functions including focusing and switching attention, controlling encoding and retrieval strategies, and mentally manipulating information held in the slave systems. For example, while performing mental arithmetic, the central executive is responsible for selecting and executing calculation heuristics.

Classically conditioned reflexes are an example of implicit memory and are mediated primarily by the: Select one: A. suprachiasmatic nucleus B. hippocampus C. arcuate fasciculus D. cerebellum

cerebellum --Conditioned reflexes rely primarily on the cerebellum. For example, the cerebellum played a key role in Pavlov's research in which dogs were conditioned to respond to a bell with salivation. Other areas of the brain involved in implicit memory include the amygdala (which mediates emotional associations) and the striatum (which is responsible for memories of skills and habits).

A psychologist who views learning as the result of operant conditioning would likely attribute the acquisition of complex behaviors to which of the following? Select one: A. chaining B. stimulus control C. higher-order conditioning D. stimulus generalization

chaining --With chaining, each response in the chain of responses required to learn and perform a complex behavior serves as a reinforcement for the previous response in the chain.

A behaviorist would most likely argue that the use of spanking as a behavioral management technique is not effective because: Select one: A. it leads to physical child abuse. B. it fosters a sense of learned helplessness in the child. C. changes in behavior aren't likely to persist over time. D. changes in behavior won't occur at all.

changes in behavior aren't likely to persist over time. --Over time, punishment alone is not likely to be effective, since it only results in a temporary suppression of behavior. In fact, as soon as the punishment is not present, the behavior is likely to increase.

During the course of a mental status exam, a 36-year old man takes a great deal of time to answer the examiner's questions because he often focuses on unnecessary details and makes parenthetical remarks. When he is not interrupted, the man usually does get around to answering the examiner's question. The man's speech illustrates which of the following? Select one: A. circumstantiality B. loosening of associations C. confabulation D. derailment

circumstantiality --Circumstantiality refers to speech that is indirect and delayed in reaching the point because of unnecessary, tedious details and parenthetical remarks.

The APA's Ethical Principles of Psychologists and Code of Conduct: Select one: A. clearly prohibits a psychologist from having sexual relationships with students and supervisees under any circumstances. B. clearly prohibits a psychologist from having sexual relationships with a student or supervisee when he/she is in a position of authority over that person. C. prohibits a psychologist from having sexual relationships with students and supervisees indirectly through the "dual relationship" clause. D. does not address the issue of sexual relationships with students and supervisees.

clearly prohibits a psychologist from having sexual relationships with a student or supervisee when he/she is in a position of authority over that person. --Standard 7.07 states that, "Psychologists do not engage in sexual relationships with students or supervisees who are in their department, agency, or training center or over whom psychologists have or are likely to have evaluative authority."

Which of the following is used to determine the amount of variability in one variable that is predictable from another variable? Select one: A. coefficient of determination B. Cronbachs alpha C. coefficient of concordance D. eta

coefficient of determination --The coefficient of determination is calculated by squaring the correlation coefficient and provides a measure of shared variability, or the amount of variability in one variable that can be explained by variability in another variable (or, put another way, the amount of variability in one variable that is predictable from another variable).

When working with a client belonging to an ethnic minority group, a White therapist interprets the client's reluctance to disclose personal information as a sign of paranoia. According to Ridley (2005), this therapist is exhibiting which of the following? Select one: A. marginalization B. overidentification C. color blindness D. cultural ambivalence

color blindness --As defined by Ridley, color blindness refers to a therapist's "illusion that minority clients are no different than non-minority clients" (p.67). He notes that "color blind" therapists tend to overlook the effects of racism and discrimination and, consequently, view deviations from White middle-class norms as pathological.

When working with Native American/Alaskan Native clients, an effective intervention would involve coordination with traditional healers (if appropriate), incorporation of members of the extended family, and a(n): Select one: A. insight-oriented approach. B. reality therapy or a solution-focused approach. C. combination of client-centered and behavioral approaches. D. combination of Adlerian and rational-emotive approaches.

combination of client-centered and behavioral approaches. --There is some disagreement among the experts about the optimal therapeutic approach for members of this population; however, it depends on several factors including the client's level of acculturation. D. W. Sue and D. Sue recommend a combination of client-centered and behavioral approaches.

You are working in a correctional facility and are asked to evaluate a prisoner to determine his eligibility for parole. In this situation, you should: Select one: A. conduct the evaluation as requested since it is part of your job. B. conduct the evaluation only if you believe it will serve a useful dispositional function. C. conduct the evaluation after reminding the prisoner that anything he says can be shared with prison authorities. D. refuse to conduct the evaluation unless it has been court-ordered.

conduct the evaluation only if you believe it will serve a useful dispositional function. --Relatively little has been written about ethical considerations in correctional facilities, so few guidelines are available. However, APA published recommendations in a 1978 issue of the American Psychologist that apply to this situation. Recommendation 3 of the above-cited document states that "Other than for legitimate research purposes, psychological assessments of offenders should be performed only when the psychologist has a reasonable expectation that such assessments will serve a useful therapeutic or dispositional function."

Results of Pope's (1992) survey of APA members indicated that which of the following issues was cited most frequently as being ethically troubling or challenging? Select one: A. fees B. sexual matters C. confidentiality D. the conduct of colleagues

confidentiality --Confidentiality was listed by 18% of respondents as an ethically troubling issue.

If you believe that, during a full moon, people tend to act "crazier," you will be likely to notice incidents of bizarre behavior during a full moon and ignore similar incidents when the moon is not full. This tendency is referred to as the: Select one: A. confirmation bias. B. saliency bias. C. self-fulfilling prophecy bias. D. correspondence bias.

confirmation bias

Of the various types of test validity, ________ validity has been described as the broadest category of validity because it overlaps and encompasses all other types. Select one: A. content B. construct C. criterion-related D. contextual

construct --Due to changes in the conceptualization of validity, methods for evaluating validity, and methods of interpreting test scores, construct validity is now described by many experts as a unifying concept of validity. The classical (tripartite) view of validity distinguishes between three major types -- content, criterion-related, and construct. From this perspective, construct validity refers to the degree to which a test measures the construct it was designed to measure.

Dr. Able, a psychologist, has been seeing a client for five months and feels that the client may not be benefitting from treatment. When she mentions this to him, the client says he is quite satisfied with the progress of therapy. Based on this information, Dr. Able's best course of action would be to: Select one: A. continue seeing the client in therapy since the client is satisfied. B. tell the client that she is ethically required to refer him to another therapist. C. obtain supervision to ensure that her treatment is effective. D. consult with another psychologist to discuss the situation.

consult with another psychologist to discuss the situation. --Although referral (response b) might be acceptable, it is not necessary as a first course of action in this case given the circumstances. Instead, consultation would be a better action since it would help the therapist determine if his evaluation of the situation is correct.

An educational psychologist with expertise in the areas of intellectual and learning disabilities is hired by a school district to assist a teacher who is having difficulty working effectively with newly "mainstreamed" students. The psychologist will work directly with the teacher to help him acquire the skills he needs to work with these students but will have little or no direct contact with the students. This is best described as a type of: Select one: A. client-centered case consultation. B. consultee-centered case consultation. C. program-centered administrative consultation. D. consultee-centered administrative consultation.

consultee-centered case consultation --In consultee-centered case consultation, the focus is on the consultee, and the consultant helps the consultee acquire the knowledge or skills needed to work effectively with a particular type of clients or students.

You receive a voicemail from a well-respected licensed psychologist in your community who is currently seeing one of your former clients. She says that she has obtained the client's consent and wants you to forward the client's record to her. You should: Select one: A. forward a photocopy (not the original) of the record to her. B. forward only information that you feel is relevant and not obsolete. C. contact the client to obtain a release directly from him. D. wait until you hear directly from the client before taking any action.

contact the client to obtain a release directly from him.

Penny Pallid is an Anglo American teacher and Cameron Chroma, her student, is African American. According to Helms's interaction model, Ms. Pallid will be LEAST effective in teaching Cameron if she is in the __________ stage of racial identity development and Cameron is in the __________ stage. Select one: A. reintegration; pre-encounter B. pseudo-independent; pre-encounter C. contact; immersion D. pseudo-independent; encounter

contact; immersion --This is an example of a crossed relationship. It is the most conflicted type and is characterized by disharmony, fear, and "warfare." It is least likely to promote the student's growth.

Identifying alternative behaviors to replace targeted undesirable behaviors is an integral part of which of the following behavioral techniques? Select one: A. response cost B. implosive therapy C. contingency management D. negative practice

contingency management --"Contingency" refers to the consequence of a behavior, and contingency management involves identifying and applying punishments for undesirable behaviors and identifying and applying reinforcements for desirable ones.

Papez's circuit is involved in which of the following? Select one: A. control of emotions B. control of circadian rhythms C. emergence of the secondary sex characteristics D. transfer of visual input to the visual cortex

control of emotions --Knowing that the Papez circuit is one of the major pathways of the limbic system and that the limbic system is referred to as the "emotional brain" would have helped you identify the correct answer to this question. Papez (1937) was among the first researchers to propose the existence of a neuroanatomical circuit that mediates emotion. As described by Papez, this circuit includes the hippocampus, mammillary bodies, cingulate gyrus, and anterior nuclei of the thalamus. In addition to controlling the experience and expression of emotions, it plays a role in the acquisition of new memories.

The best way to keep two sets of six digits in working memory is to: Select one: A. visualize the digits in a string. B. convert the 12 digits to "chunks." C. connect each digit with a visual image. D. repeat the digits first forward and then backward several times.

convert the 12 digits to "chunks" --The number of digits exceeds the number of units that can be held in short-term memory. Therefore, it would be necessary to "chunk" the digits into a smaller number of units; e.g., four chunks of three digits each.

The Introduction and Applicability section of the APA's Ethics Code notes that the APA may take action against a member after his/her: Select one: A. conviction of a felony. B. conviction of a felony or misdemeanor. C. conviction of a felony or misdemeanor only when the offense is clearly related to the psychologists professional role. D. conviction of a felony only when the offense resulted in a loss or suspension of licensure.

conviction of a felony --The Introduction and Applicability section states that the APA "may take action against a member after his or her conviction of a felony, expulsion or suspension from an affiliated state psychological association, or suspension or loss of licensure." It does not place qualifications on these exceptions and it does not apply to misdemeanors.

Which of the following brain structures is responsible for interhemispheric communication? Select one: A. precentral gyrus B. arcuate fasciculus C. cingulate gyrus D. corpus callosum

corpus callosum --Interhemispheric communication refers to communication between the right and left hemispheres of the brain. The corpus callosum is the primary bundle of nerve fibers connecting the left and right hemispheres. Its role in transferring information between the hemispheres was identified in the 1950s by Myers and Sperry.

A client taking Parnate, an MAO-inhibitor, should be warned against eating all of the following except: Select one: A. yogurt. B. soy sauce. C. cottage cheese. D. over-ripe avocados.

cottage cheese --Although cheese should generally be avoided, it is aged and ripe cheeses that are a problem. Cottage cheese and cream cheese are usually acceptable. (The other foods listed contain tyramine and must be avoided.)

The World Health Organization's International Pilot Study of Schizophrenia (IPSS) and Determinants of Outcome Study have compared patients with Schizophrenia from non-Western developing countries to those from Western industrialized countries. This research has found that patients from developing and industrialized countries tend to differ in terms of: Select one: A. age and gender ratios. B. course and outcomes. C. symptom profiles. D. gender and prognosis.

course and outcomes --The IPSS found course and outcome differences in patients from developing and industrialized countries; patients from developing countries were much more likely to exhibit an a

Which of the following would a managed care organization consider most useful for reducing its risk for legal liability? Select one: A. capitation B. behavioral health "carve outs" C. credentialing of providers D. utilization review

credentialing of providers --Credentialing providers helps ensure that providers are competent to provide services and thereby reduces the risk for legal liability (i.e., it contributes to risk management).

A test developer is asked to develop a test to evaluate the effects of a training program on the sales success of inexperienced sales employees. When establishing the validity of the test, the test developer will probably be MOST interested in which of the following types of test validity? Select one: A. discriminant B. content C. criterion-related D. construct

criterion-related --Criterion-related validity refers to the relationship between test scores and a criterion measure. In this case, the test developer will be interested in the degree to which training outcome correlates with the criterion of job performance. Thus, the test developer will be interested in assessing the criterion-related validity of the test.

In the Seattle Longitudinal Study, K. W. Schaie (1996) used which of the following research designs to investigate the relationship between age and intelligence? Select one: A. counterbalanced B. crossover C. cross-sectional D. cross-sequential

cross-sequential --To reduce problems associated with longitudinal and cross-sectional research (e.g., dropouts, cohort effects), Schaie used a cross-sequential design that combined the two strategies. The results of his research identified predictable changes (increases or decreases) in specific cognitive abilities during adulthood and indicated that the range of change for each ability is fairly narrow until age 60 or later.

Research suggests that the underutilization of mental health services by Asian Americans results from: Select one: A. cultural values that encourage resolution of problems within the family. B. a lower incidence of mental health problems. C. cultural values that stress personal independence. D. a distaste for the formality involved in therapy relationships.

cultural values that encourage resolution of problems within the family. --Asian Americans often adhere to traditional cultural values that encourage the resolution of problems within the family. Reasons for preferring assistance from family members include the stigma associated with mental illness and seeking outside help and a lack of culturally competent mental health services.

Which of the following is NOT characteristic of Piaget's preoperational stage of development? Select one: A. magical thinking B. insight learning C. finalism D. decentration

decentration --In the preoperational stage, children exhibit centration, or an inability to mentally hold two dimensions at the same time. Decentration occurs in the concrete operational stage and contributes to the ability to conserve.

During an experiment, the investigator asks participants about their beliefs regarding the study's purpose and how they were expected to perform. When analyzing the data, she finds that participants' actual performance is consistent with their beliefs and expectations. This suggests that the study's results may be confounded by: Select one: A. carryover effects. B. the halo bias. C. the Hawthorne effect. D. demand characteristics.

demand characteristics --In this situation, the participants may have acted in ways consistent with their expectations rather than simply in response to the experimental manipulation. Demand characteristics are unintentional cues in the experimental environment or manipulation that affect participants' beliefs or expectations and thereby may account for the results of the study.

In the 1979 case of Larry P. v. Riles, the judge ruled that IQ tests could no longer be used to: Select one: A. make hiring and other employment decisions. B. make college admission decisions. C. determine if older African American adults should be assigned a legal guardian. D. determine if African American children should be placed in special education classes.

determine if African American children should be placed in special education classes. --Judge Peckham concluded that standard IQ tests should not be used to assess African American children because their use resulted in too many of these children being placed in "dead-end" (special education) classes.

Smith, Glass, and Miller's (1980) use of meta-analysis involved: Select one: A. counting the number of psychotherapy outcome studies that found a statistically significant difference between treatment and no-treatment groups. B. statistically comparing the number of psychotherapy outcomes studies that did and did not find significant treatment effects. C. determining the average magnitude of the outcome of psychotherapy across a large number of outcome studies. D. using initial symptom severity and type and duration of treatment to predict average treatment outcome.

determining the average magnitude of the outcome of psychotherapy across a large number of outcome studies. --When using meta-analysis, the outcomes of each study are converted to a common metric - i.e., an effect size - and an average effect size is calculated.

The most common precursor of Bulimia Nervosa during adolescence is: Select one: A. academic failure. B. dieting. C. sexual abuse. D. peer rejection.

dieting --L. K. G. Hsu notes that Bulimia and other eating disorders almost always begin with dieting. He proposes that "dieting provides the entree into an eating disorder" (p. 76), especially when dieting is combined with other high-risk factors such as depression or social anxiety, low self-concept, and poor identity formation.

The slope of an item characteristic curve provides information on which of the following? Select one: A. reliability B. difficulty level C. discrimination D. probability of guessing correctly

discrimination --The slope of the curve indicates the item's ability to discriminate between low and high achievers. The "flatter" the slope, the less the item's discrimination.

A client you have been seeing for eight months says he wants to quit therapy. He feels that the original problems he came to therapy for have all been resolved. You disagree and feel that there is good reason for the man to continue seeing you. You should: Select one: A. get the client to agree to a few more sessions. B. discuss his reasons for wanting to terminate. C. discuss his reasons for wanting to terminate and your reasons for thinking he should continue. D. let him terminate but let him know he can come back if he desires.

discuss his reasons for wanting to terminate and your reasons for thinking he should continue.

As defined in Atkinson, Morten, and Sue's (1993) Racial/Cultural Identity Development Model, a person who has contradictory appreciating and depreciating attitudes toward both his/her own culture and the dominant (majority) culture is in which of the following stages? Select one: A. resistance B. dissonance C. conformity D. disintegration

dissonance --The dissonance stage is characterized by conflicting attitudes toward one's own group and the majority group.

A former therapy client calls you and expresses an interest in becoming friends. The former client suggests that you meet at a local restaurant for lunch. You remember that the client shared many of your interests, and that, at the time of therapy, you were aware that he/she was someone you would like to know better. As an ethical psychologist, you: Select one: A. meet the former client for lunch only if the therapeutic relationship was terminated more than two years ago. B. meet the former client for lunch only if the therapeutic relationship was terminated more than two years ago and you believe that a relationship is not clinically contraindicated and is not likely to be exploitative. C. meet the client for lunch to clarify and discuss any potential conflicts and then proceed with caution. D. do not meet the client for lunch and explain why a friendship would not be in the client's best interests.

do not meet the client for lunch and explain why a friendship would not be in the client's best interest --Most experts agree that friendships with former clients should be avoided since this sets up an unequal situation and precludes the possibility that the former client can return for therapy in the future.

Haloperidol (Haldol) exerts its effects primarily by blocking _________ receptors in certain areas of the brain. Select one: A. GABA B. norepinephrine C. dopamine D. glutamate

dopamine --Like other conventional antipsychotics, haloperidol is a dopamine antagonist and exerts its therapeutic effects primarily by blocking dopamine receptors. However, it also has some effect on acetylcholine and serotonin levels.

As the result of a traumatic head injury sustained in a motor vehicle accident, a middle-aged man has mild impairments in judgment, insight, and planning and reduced sexual interest. Damage to which of the following areas of the brain is most likely responsible for these deficits? Select one: A. dorsolateral prefrontal region B. orbitofrontal region C. medial temporal region D. posterior parietal region

dorsolateral prefrontal region --Although traumatic brain injury can produce diffuse brain injury, the frontal and temporal lobes are the areas of the cortex that are usually most adversely affected. The dorsolateral prefrontal region mediates executive cognitive functions. Damage to this area is associated with deficits in attention, planning, problem-solving, and other higher-order cognitive abilities as well as reduced sexual interest and apathy.

Ho (1987) and others have recommended the use of a(n) __________ approach when working with African American therapy clients. Select one: A. ecostructural B. solution-focused C. narrative D. ethnographic

ecostructural

Freud's psychosexual theory of development is based on the premise that, in each stage: Select one: A. efforts to obtain pleasure center on a different part of the body. B. a different psychological need is "prepotent." C. the source of anxiety is related to a different psychosocial conflict. D. the source of anxiety is related to a different psychic need.

efforts to obtain pleasure center on a different part of the body. --Freud's psychosexual theory emphasizes the impact of the sex drive (libido) on development. In each stage of development, a different part of the body is most sensitive to erotic stimulation.

The ring and little fingers of the hand represent one dermatome and are innervated by which of the following? Select one: A. second cervical nerve (C4) B. eighth cervical nerve (C8) C. second thoracic nerve (T2) D. tenth thoracic nerve (T10)

eighth cervical nerve (C8) --The skin on the medial side of the arm and forearm and the ring and little fingers is innervated by C8.

Berscheid's (1989) "emotion-in-relationships" model predicts that strong emotions in close relationships are: Select one: A. related to the degree to which prepotent needs are mutually satisfied. B. a function of each partners level of differentiation. C. moderated by personal beliefs about the acceptability of emotional expression. D. elicited by unexpected interruptions in routine interactions.

elicited by unexpected interruptions in routine interactions --According to E. Berscheid (1989), a partner is most likely to experience a high degree of emotion in an intimate relationship when his/her partner's behavior disrupts an organized sequence of behavior. Berscheid's emotion-in-relationships model predicts that disruptions in organized sequences or patterns of behavior (established interactions between the partners) produce strong emotions in close relationships, with the nature of the emotion (positive or negative) depending on the consequence of the disruption.

A therapy client says, "I feel useless and incompetent and, therefore, I must be a worthless person." From the perspective of cognitive therapy, this is an example of: Select one: A. all-or-none thinking. B. overgeneralization. Incorrect C. personalization. D. emotional reasoning.

emotional reasoning --The logic underlying emotional reasoning is "I feel, therefore I am."

Longitudinal research conducted by Strauss and Carpenter (1991) has found that, for patients with Schizophrenia, prognosis with regard to future employment is best predicted by: Select one: A. frequency of social contacts. B. symptom severity. C. employment history. D. duration of hospitalization.

employment history --This issue was investigated by W. Strauss and J. Carpenter in their longitudinal study of individuals with Schizophrenia. A key finding of their study was that the various domains of functioning are relatively independent - previous employment is the best predictor of current employment, past symptom severity is the best predictor of current symptom severity, etc.

During a group therapy session, a few group members complain that some of the members have not been disclosing information about themselves. Other members become defensive and say that they don't feel like they should be required to talk about themselves. A therapist relying on the approach advocated by Irvin Yalom (1985) would: Select one: A. regard the conflict as a normal stage of group development and not intervene. B. interpret the conflict as resistance. C. encourage members to discuss the meaning of the conflict for them. D. clarify the function of self-disclosure in therapy.

encourage members to discuss the meaning of the conflict for them. --This is most consistent with Yalom's belief that group therapy provides several "curative factors" including interpersonal learning and self-understanding.

A drive for order that involves the testing of patterns and structures against the real world is referred to as: Select one: A. elaboration. B. deduction. C. equilibration. D. accommodation.

equilibration --As defined by Piaget equilibration involves a combination of assimilation and accommodation and is motivated by a drive for balance or order.

The Buckley Amendment: Select one: A. establishes the right of the school psychologist to determine who should have access to information included in school records. B. establishes the right of parents to inspect the school records of their children. C. establishes the right of patients or their legal representatives to have access to their hospital records. D. requires that parental consent be obtained before a student can be subjected to a psychological or psychiatric examination

establishes the right of parents to inspect the school records of their children. --Among other things, the Buckley Amendment states that any school district may be denied federal funds if parents of students (or other legal guardians) or students who have reached the age of majority are not given access to their school records. It also prohibits disclosure of school records to unauthorized individuals without parental consent.

Which of the following would probably be LEAST useful when a therapist is working with an Asian or Asian American client? Select one: A. identifying therapy goals during the initial session B. establishing an egalitarian relationship with the client early in therapy C. making use of "relationship questions" that elicit information about the client's interactions with others D. asking the client to try to identify exceptions to the presenting problem

establishing an egalitarian relationship with the client early in therapy --An egalitarian relationship is usually contraindicated because of "the Asian norms of deference to authority and modesty in the presence of superiors

A psychologist in private practice who has been sued once in the past by a client for malpractice and who fears being sued again decides to limit his practice to clients whose problems are not highly associated with higher rates of malpractice claims. This is: Select one: A. unethical if he is competent to treat these clients. B. ethical only if he is willing to see these clients in emergencies and other unusual circumstances. C. ethical as long as he makes appropriate referrals. D. ethical as long as he makes his policy clear at the outset of treatment.

ethical as long as he makes appropriate referrals --This issue is addressed by R. I. Simon in Clinical psychiatry and the law (American Psychiatric Press, Washington, DC, 1992). Although Simon is discussing the obligations of psychiatrists, the same principles apply to psychologists: Self-employed mental health professionals do not have a legal or ethical duty to enter into professional relationships. While the psychologist's decision may seem unfair, he is free to serve the clients he chooses. He should, however, make appropriate referrals. (Note that the Ethics Code does require psychologists not to discriminate on the basis of race, gender, or any other basis proscribed by law. Litigious clients do not fall into this category, however.)

Dr. A., a psychologist, provides office space and secretarial services to Dr. B., a newly-licensed psychologist, in exchange for 25% of the fee that Dr. B. collects from his clients. Dr. A. begins referring clients to Dr. B. and charges him 35% for the referred clients. This is: Select one: A. unethical because the arrangement violates APA's prohibition against "fee splitting." B. unethical because the percentage that Dr. A. is charging Dr. B. is exploitative. C. ethical only if the larger fee for referred clients reflects increased costs to Dr. A. for making referrals to Dr. B. D. ethical because the arrangement does not violate the provisions of the Ethics Code regarding referral fees.

ethical only if the larger fee for referred clients reflects increased costs to Dr. A. for making referrals to Dr. B. --It is ethical to pay a fee for a referral when the fee reflects the person's expenses for making the referral. However, charging more for referred patients than non-referred patients when there are no additional costs would be unacceptable.

A research psychologist designs a study that involves hospitalized psychiatric patients who have legal guardians. Before beginning the study, he obtains signed consents from the legal guardians. The psychologist has acted: Select one: A. unethically since he has breached the patients' confidentiality. B. ethically as long as the patients are allowed to withdraw from the study at any time. C. ethically as long as he has also obtained assent from the patients. D. ethically, since all a psychologist must obtain in this situation is the informed consent of legal guardians.

ethically as long as he has also obtained assent from the patients. --This answer is consistent with the requirements of the Ethics Code -- i.e., when research participants are not legally capable of giving informed consent (either because they are mentally incompetent or minors), consent must be obtained from their legal guardians, and the participants should be provided with understandable information and their asent to participate should be obtained.

A practitioner of Rogers's client-centered therapy would use the Q-sort technique to: Select one: A. facilitate identifying a client's diagnosis. B. help the client identify specific therapy goals. C. help identify solutions to a client's presenting problems. D. evaluate a clients progress in therapy.

evaluate a clients progress in therapy --Rogers used the Q-sort technique to evaluate therapy progress by evaluating a client's degree of congruence between his/her self and ideal self. This involved having the client first sort the cards in terms of how well they describe the client's current self and then in terms of his/her ideal self. The more similar the two sorts, the greater the congruence between the self and ideal self.

An African American therapy client exhibiting the "split-self syndrome" as a response to racial oppression will most likely: Select one: A. exhibit a high degree of detachment from his family and neighborhood. B. avoid contacts with Whites. C. evaluate himself against White standards. D. exhibit a high degree of "functional paranoia."

evaluate himself against White standards. --According to Landrum-Brown, the split-self syndrome "is a result of accepting a polarized, hierarchical manner of thinking and accepting negative racial messages." For African Americans, one manifestation of this syndrome is evaluating oneself against White standards.

Longitudinal research by Tizard and colleagues (e.g., Hodges and Tizard, 1989) indicates that infants initially raised in institutions are subsequently able to develop a close bond with their adoptive parents: Select one: A. only if they are adopted by 12 months of age. Incorrect B. only if they are adopted by 24 months of age. C. even if they are not adopted until 4 to 6 years of age. D. only if they had established a strong bond with a biological parent prior to being institutionalized.

even if they are not adopted until 4 to 6 years of age. --Tizard et al.'s research indicates that "late adoptees" (children placed in adoptive homes after age 4) are able to form strong bonds when the adoption occurs by six years of age. Although these researchers found that children adopted between the ages of 4 and 6 did develop strong bonds with their adoptive parents, their data also revealed that these children had higher-than-normal rates of emotional and social problems in childhood and adolescence, including an excessive need for adult attention and "overfriendliness" toward unfamiliar adults

Research (e.g., Davidson and Parker, 2001) evaluating the mechanisms that contribute to the effectiveness of EMDR (eye movement desensitization and reprocessing) has generally found that its beneficial effects are attributable to: Select one: A. rapid eye movements. B. exposure to the feared stimulus. C. a placebo effect. D. counterconditioning

exposure to the feared stimulus --Based on the results of their meta-analysis, these investigators conclude that "eye movements are unnecessary and that EMDR may be viewed as an imaginal exposure technique"

A person who experiences cataplexy as part of his narcoleptic attacks will most likely try to avoid: Select one: A. afternoon naps. B. expressing strong emotions. C. fluctuations in salt intake D. eye strain.

expressing strong emotions --Cataplexy involves bilateral loss of muscle tone. Cataplexy is usually triggered by intense emotions. Consequently, people with cataplexy often attempt to control their emotional expressiveness.

Which of the following approaches in family therapy encourages the therapist to develop a "therapeutic triangle" with family members? Select one: A. extended family systems therapy B. communication/interaction family therapy C. adaptive family therapy D. strategic family therapy

extended family systems therapy --Bowen's extended family systems therapy encourages the therapist to become a member in a therapeutic triangle with two family members (usually the spouses/partners).

An African American client feels that the primary obstacle in his life is racism and that there's nothing he can do to change the problems he is experiencing. As defined by Sue (1978), this individual has an: Select one: A. internal locus of control and internal locus of responsibility. B. external locus of control and internal locus of responsibility. C. internal locus of control and external locus of responsibility. D. external locus of control and external locus of responsibility.

external locus of control and external locus of responsibility. --This individual has an external locus of responsibility (he can't do anything about his problems) and an external locus of control (he believes his problems are due to racism).

According to the overjustification hypothesis: Select one: A. external rewards increase intrinsic interest. B. intrinsic interest decreases the valuation of external rewards. C. external rewards decrease intrinsic interest. D. intrinsic interest increases the valuation of external rewards.

external rewards decrease intrinsic interest. --Results of research demonstrating this phenomenon have been interpreted in terms of Bem's self-perception theory, which proposes that people make attributions about their own attitudes and feelings by observing their own behaviors. This theory implies that a person observing herself doing something for an external reward would conclude that the external reward, and not intrinsic interest, is the cause of her behavior.

Obtaining research participants for a study on the sexual practices of single young adults from computer dating services, "singles' clubs," and activities for singles sponsored by a variety of organizations would help increase: Select one: A. statistical power. B. statistical significance. C. internal validity. D. external validity.

external validity --In this situation, the researcher is obtaining participants from a variety of "natural" settings. External validity refers to the generalizability of research results. When participants are selected from natural settings (i.e., from settings where singles are likely to go to meet other singles), this will increase the generalizability of the results.

Dr. Agape uses a dismantling strategy to identify the "active ingredient" of systematic desensitization. He is most likely to find that which of the following is the critical component? Select one: A. stimulus control B. reciprocal inhibition C. extinction D. behavior rehearsal

extinction --Systematic desensitization was originally developed as an application of reciprocal inhibition. However, studies using the dismantling strategy have found that its effects are due to repeated exposure to the CS without the US (i.e., to classical extinction).

If an alpha error is equivalent to a false positive, then a beta error is equivalent to a: Select one: A. true positive. B. true negative C. false positive. D. false negative

false negative --In decision theory, a false positive occurs when a predictor indicates an effect (e.g., a diagnosis) when, in fact, there is none - e.g., the predictor tells you that you have a disease when you don't have it. Therefore, a false positive is similar to an alpha (Type I) error in inferential statistics. In contrast, a false negative occurs when a predictor indicates no effect (e.g., an absence of a diagnosis) when, in fact, there is an effect - e.g., the predictor says you do not have a disease when you do have it. Therefore, a false negative is similar to a beta (Type II) error in inferential statistics.

Self-in-relation therapy is an approach to: Select one: A. family therapy that emphasizes the multiple transferences that affect current relationships among family members. B. group therapy that focuses on interpersonal interpretations and misinterpretations that affect individual functioning. C. feminist therapy that emphasizes the role of the mother-son versus mother-daughter relationship in creating gender differences in behavior. D. biopsychosocial therapy that recognizes the interacting impact of biological and sociocultural factors on male and female development.

feminist therapy that emphasizes the role of the mother-son versus mother-daughter relationship in creating gender differences in behavior. --Object relations theory was an important influence on self-in-relation theory. However, self-in-relation theory extends the object relations approach by considering the impact of same versus opposite gender in caregiver-infant relationships on development, especially on development of the relational self.

Agranulocytosis is a potential side effect of clozapine, carbamazepine, and a number of other psychiatric drugs. Early symptoms of this disorder include: Select one: A. fever, sore throat, mouth ulcers, and lethargy. B. nausea, diarrhea, vomiting, and a metallic taste in the mouth. C. sweating, palpitations, headache, tremulousness, and cardiac arrhythmia. D. constricted pupils, decreased visual acuity, sweating, constipation, and nausea.

fever, sore throat, mouth ulcers, and lethargy. --Agranulocytosis is caused by a failure of the bone marrow to produce a sufficient number of certain white blood cells, which increases the body's susceptibility to infection. Fever, sore throat, chills, mouth ulcers, lethargy, and weakness are early signs of agranulocytosis.

Two weeks before the election for senior class president, the four candidates are given the opportunity to deliver a 10-minute speech during a school assembly. Sally Smart is familiar with research on the recency and primacy effects and, therefore, asks if she can speak: Select one: A. first. B. second. C. third. D. fourth.

first --The primacy effect is usually stronger than the recency effect, although the advantage of going first or last is affected by several factors. For example, there is more likely to be a primacy effect when the messages are presented back-to-back and there is a delay before a decision must be made.

The presence of which of the following symptoms would suggest a diagnosis of Conduct Disorder rather than a diagnosis of Oppositional Defiant Disorder? Select one: A. frequent lying and running away from home B. low frustration tolerance and temper outbursts C. drug use D. onset of symptoms after age 12

frequent lying and running away from home --Lying and running away are characteristic of Conduct Disorder. Behaviors associated with Oppositional Defiant Disorder are less severe and include negativism, defiance, and hostility.

Adults with ADHD are most likely to: Select one: A. be excessively perfectionistic in their work. B. prefer jobs that involve routine, repetitive tasks. C. avoid relationships, especially sexual relationships. D. frequently change jobs and intimate partners.

frequently change jobs and intimate partners --The research has found that adults with ADHD change jobs and intimate partners more often than do individuals without the disorder.

It is generally believed that, of the lobes of the cerebral cortex, the __________ lobes are the last to fully develop and myelinate. Select one: A. frontal B. temporal C. parietal D. occipital

frontal --Development of the cortex corresponds to the emergence of various capabilities. The frontal lobes mediate a number of functions including higher-order cognitive skills, which do not fully develop until early adulthood.

Research comparing the use of mental health services by heterosexual adults and gay/lesbian adults has generally found that: Select one: A. gay men are more likely than heterosexual men to use mental health services, but lesbian women are less likely than heterosexual women to do so. B. lesbian women are more likely than heterosexual women to use mental health services, but gay men are less likely than heterosexual men to do so. C. gay men and lesbian women are more likely than their heterosexual counterparts to use mental health services. D. gay men and lesbian women are less likely than their heterosexual counterparts to use mental health services.

gay men and lesbian women are more likely than their heterosexual counterparts to use mental health services.

Research on the serial position effect suggests, that when "cramming" for a test, it is important to: Select one: A. get a good night's sleep before taking the test. B. engage in a distracting activity after studying and prior to taking the test. C. give extra attention to material studied during the middle of the study session. D. give extra attention to material studied at the beginning of the study session.

give extra attention to material studied during the middle of the study session. --Studies on memory and forgetting have shown that people tend to remember information presented in the beginning and, in some cases, end of a list better than that presented in the middle. If this finding is extended to a study session, it predicts that a student should pay most attention to the information reviewed in the middle of the study session since that is the information that is most likely to be forgotten.

Which of the following is not an empirically supported treatment for primary insomnia? Select one: A. stimulus control therapy B. sleep restriction C. cognitive therapy D. habit reversal training

habit reversal training --Habit reversal training is used to eliminate habits and other undesirable repetitive behaviors including tics, trichotillomania, and stuttering.

The gradual decline in the intensity, frequency, or duration of a response to the repeated presentation of the same punishment or other stimulus is referred to as: Select one: A. satiation. B. inhibition. C. fading. D. habituation.

habituation

When using punishment to reduce or eliminate an undesirable behavior, it is important to keep in mind that gradually increasing the intensity or severity of a punishment will likely reduce its effectiveness. This reduction in the effectiveness of punishment is referred to as: Select one: A. satiation. B. habituation. C. sensitization D. extinction.

habituation --The question describes habituation which, in the context of punishment, occurs when the punisher loses its effectiveness. Gradually increasing the intensity of a punishment over time is usually ineffective because it leads to habituation.

As described in the DSM-5, the five core symptoms of the Schizophrenia Spectrum Disorders are: Select one: A. delusions, hallucinations, negative symptoms, impaired cognition, and disorganized speech. B. disorganized thinking, delusions, hallucinations, negative symptoms, and depersonalization or derealization. C. hallucinations, negative symptoms, grossly disorganized or abnormal motor behavior, delusions, and disorganized thinking. D. grossly disorganized or abnormal motor behavior, hallucinations, negative symptoms, affective instability, and delusions.

hallucinations, negative symptoms, grossly disorganized or abnormal motor behavior, delusions, and disorganized thinking.

A white middle-aged man is at highest risk for suicide if he: Select one: A. has been divorced for 10 years, overtly expresses aggression and hostility, and is unwilling to talk about his suicidal feelings. B. has been divorced for 6 years, exhibits a constriction of affect, and has a peptic ulcer. C. has been divorced for 3 years, overtly expresses aggression and hostility, and is unwilling to talk about his suicidal feelings. D. has been divorced for 1 year, exhibits a constriction of affect, and has a peptic ulcer.

has been divorced for 1 year, exhibits a constriction of affect, and has a peptic ulcer. --The more recent the divorce, the greater the risk for suicide. A higher risk in adulthood is also associated with constriction of affect (versus overt expression) and the presence of certain illnesses including a peptic ulcer

Research investigating the effects of Head Start and similar compensatory education programs has found that these programs: Select one: A. have few benefits in terms of academic achievement or other outcomes. B. have some short-term benefits in terms of academic achievement only. C. have benefits in terms of academic outcomes, social adjustment, and employment. D. have few benefits in terms of academic outcomes but some benefits in terms of of social adjustment and employment.

have benefits in terms of academic outcomes, social adjustment, and employment. --Longitudinal studies have found that the immediate effects of compensatory education programs on IQ are often not maintained but that these programs have other positive academic, social, and occupational consequences. Answer C is the best option. The answer summarizes the results of longitudinal studies which have found that improvements in IQ test scores are often not maintained but that children who attend these programs often obtain higher achievement test scores, are less likely to drop out of school, and have lower rates of delinquency and unemployment.

Congenital cytomegalovirus (CMV) is the second-most common cause of an intellectual disability after Down Syndrome. In additional to cognitive impairments, it is most likely to cause which of the following? Select one: A. facial deformities B. heart and lung abnormalities C. genital, urinary, and intestinal abnormalities D. hearing and visual impairments

hearing and visual impairments --CMV is a type of herpes virus. Although infection with CMV during adulthood usually produces no or mild symptoms, infection during prenatal development or birth can be life-threatening. Approximately 10% of infants infected with CMV show symptoms at birth, and about 10 to 15% of infants who are asymptomatic at birth develop symptoms during the first few years of life. Common symptoms include some degree of intellectual disability and hearing and visual impairments.

In the context of Freudian psychoanalysis, the goal of confrontation is to: Select one: A. reduce the client's resistance. B. clarify a client's statements. C. help the client see a behavior in a new way. D. foster the development of transference.

help the client see a behavior in a new way. --When using confrontation, the therapist makes a statement or asks a question that connects the client's behavior to an unconscious motivation or conflict.

You are developing an educational program to discourage high school students from using drugs. You are considering using "fear arousal" as one method for persuading the students but should keep in mind that the research has shown that: Select one: A. lower levels of fear arousal are associated with the greatest amount of opinion change. B. the greater the amount of fear aroused, the greater the opinion change. C. high levels of fear arousal are associated with the greatest amount of opinion change as long as certain conditions are met. D. fear arousal is not a good method for inducing opinion change.

high levels of fear arousal are associated with the greatest amount of opinion change as long as certain conditions are met. --Although some early research suggested that high fear arousal results in less attitude change than low fear arousal, subsequent studies have demonstrated that high levels of fear arousal are the most effective for inducing attitude change as long as the fear-arousing message is accompanied by information about what actions to take to reduce or avert dangerous consequences.

Research on the comorbidity of the eating disorders suggests that the rates of OCD, Social Phobia, and Specific Phobia are: Select one: A. higher in individuals with Anorexia Nervosa and Bulimia Nervosa than in other individuals and that the onset of the anxiety disorder is most often prior to the onset of the eating disorder. B. higher in individuals with Anorexia Nervosa or Bulimia Nervosa than in other individuals and the onset of the anxiety disorder is most often simultaneous with the onset of the eating disorder. C. higher in individuals with Anorexia Nervosa or Bulimia Nervosa than in other individuals and the onset of the anxiety disorder is most often after the onset of the eating disorder. D. similar in individuals with Anorexia Nervosa or Bulimia Nervosa as the rates for other individuals and the onset is most often simultaneous with the onset of the eating disorder.

higher in individuals with Anorexia Nervosa and Bulimia Nervosa than in other individuals and that the onset of the anxiety disorder is most often prior to the onset of the eating disorder. --The majority of studies have found that the onset of the anxiety disorder precedes the onset of the eating disorder.

Chronically high levels of cortisol due to chronic stress can lead to significant damage to certain areas of the brain, especially to the: Select one: A. hippocampus. B. thalamus. C. suprachiasmatic nucleus. D. cerebellum.

hippocampus --Increased cortisol levels provide a temporary increase in energy which enables us to better respond to stress. However, when stress becomes chronic, increased cortisol levels can have a detrimental effect. The hippocampus is very sensitive to cortisol, and chronically high levels of this hormone can destroy neurons in this structure resulting in significant impairments in memory.

A 12-month old baby who has been classified as "insecure/ambivalent" is reunited with his mother in Ainsworth's "Strange Situation." Most likely, this baby will: Select one: A. hit or push his mother when she approaches and continue to cry after she picks him up. B. run away from his mother when she approaches but then cling to her when she picks him up. C. greet his mother but try to avoid her when she attempts to pick him up. D. ignore his mother intially but welcome her attempts to pick him up.

hit or push his mother when she approaches and continue to cry after she picks him up.

Lewin's field theory predicts that: Select one: A. human behavior is due more to physical than to psychological factors. B. a leader's power is directly affected by the characteristics of the task and environment. C. human behavior is a function of both the person and his/her environment. D. a group can be no more effective than its least effective member.

human behavior is a function of both the person and his/her environment. --According to Lewin's field theory, behavior is a function of the person and the environment. He expressed this relationship with the following formula: B = f(P, E).

Precocious puberty, or the development of secondary sex characteristics prior to age 8 in girls and 8.5 in boys, has been linked to premature awakening of the: Select one: A. hypothalamic-pituitary axis. B. hypothalamic-pituitary-adrenal axis. C. hippocampal-anterior thalamic axis. D. mesocorticolimbic-dopaminergic axis.

hypothalamic-pituitary axis. --The hypothalamic-pituitary axis is also known as the hypothalamic-pituitary-gonadal axis and is involved in sexual maturation.

A structural family therapist would use a family map to: Select one: A. facilitate interactions between family members during a therapy session. B. help family members recognize differences in their perceptions. C. identify family values, life themes, and significant life events. D. clarify the nature of the boundaries between family subsystems.

identify family values, life themes, and significant life events. --A family map is used to help make a structural diagnosis of a family by clarifying its boundaries, coalitions, alignments, etc.

A female client says she frequently misperceives things and this has made her anxious about leaving home. For example, she often thinks she sees small animals at work when, in fact, the "animals" are actually inanimate objects such as books, coffee mugs, and office supplies. The woman's misperceptions are best described as: Select one: A. illusions. B. delusions. C. hallucinations. D. depersonalization.

illusions --An illusion is a misperception of reality (e.g., misperceiving a coffee mug as a rodent).

Global aphasia involves: Select one: A. intact comprehension, nonfluent speech, and deficits in naming and repetition. B. impaired comprehension, nonfluent speech, and deficits in naming and repetition. C. impaired comprehension, fluent speech, and paraphasia. D. intact comprehension, fluent speech, and paraphasia.

impaired comprehension, nonfluent speech, and deficits in naming and repetition. --As its name suggests, global aphasia is caused by widespread brain injury and is characterized by extensive disruption in the ability to produce and understand language. Impaired comprehension, impaired speech, anomia (inability to name a common or familiar object, attribute, or action), and paraphasia (substitution of a word with a sound or incorrect word) are symptoms of this type of aphasia.

Ataxia is a common symptom of advanced multiple sclerosis and is characterized by: Select one: A. a complete or almost complete loss of movement. B. an uncomfortable sense of restlessness or agitation. C. impaired coordination and balance. D. slow writhing involuntary movements of the extremities.

impaired coordination and balance --Ataxia involves a loss of balance and coordination. It is one of the symptoms of multiple sclerosis and can be caused by damage to the cerebellum, dorsal spinal cord, or vestibular system. Although its symptoms depend on the area affected, they often include unsteady gait, difficulty with fine motor tasks, abnormal eye movements, slurred speech, and difficulty swallowing.

Expressive aphasia is to Broca's area as ____________ aphasia is to Wernicke's area. Select one: A. global B. conduction C. impressive D. transcortical

impressive --Wernicke's aphasia has several names including sensory, receptive, and impressive aphasia.

Research on sexual and nonsexual dual relationships between therapists and their clients has found that: Select one: A. male therapists engage in more sexual dual relationships, but female therapists engage in more nonsexual dual relationships. B. the rates of sexual dual relationships are higher among those with less professional experience. C. in a sexual dual relationship, the client is, on the average, 10 years younger than the therapist. D. therapists who undergo personal therapy are less likely to become sexually involved with their clients.

in a sexual dual relationship, the client is, on the average, 10 years younger than the therapist. --The average age of therapists who become involved in sexual dual relationships is 42 or 43 (depending on which study you read), while the average age of the client is 32 or 33.

Berscheid's (1983) emotion-in-relationship model proposes that strong positive emotions are more likely to be elicited in the initial stages of a relationship than in later stages because: Select one: A. in the early stages, the partners have limited information about each other's undesirable qualities. B. in the early stages, each partner is more likely to do something that is both positive and unexpected. C. in the early stages, the partners work harder to be sensitive to each other's needs. D. over time, people become "emotionally immune" to the behaviors of others.

in the early stages, each partner is more likely to do something that is both positive and unexpected. --According to Berscheid, it is more difficult to do something pleasant and unexpected in the later stages of a relationship. Therefore, it is more difficult to elicit positive emotions from one's partner in the later stages of a relationship than in the initial stages.

When deciding on an intervention plan for a client with Agoraphobia, it is important to keep in mind that which of the following seems to be the key element in treating this disorder? Select one: A. counterconditioning B. progressive relaxation C. in vivo exposure D. cognitive restructuring

in vivo exposure --In vivo exposure with response prevention has been consistently found to be an effective intervention for Agoraphobia.

Tulving has divided memory into three types: procedural, episodic, and semantic. According to Tulving, semantic memory: Select one: A. consists of internal representations of stimulus-response connections. B. is more affected by amnesia than procedural memory and episodic memory. C. includes rules for manipulating words and symbols. D. is coded temporally.

includes rules for manipulating words and symbols.

A menopausal woman who is considering hormone replacement therapy (HRT) should be advised that this treatment may do all of the following except: Select one: A. reduce the risk for osteoporosis. B. reduce or eliminate "hot flashes." C. increase sex drive. D. reduce mood symptoms.

increase sex drive --HRT continues to be controversial because it is not only associated with a number of benefits but also with some negative side effects. The results of research investigating the effects of HRT on libido are inconsistent, and the best conclusion that can be drawn at this time is that it does not increase sex drive.

A distribution of scores has a mean of 110 and a standard deviation of 10. Adding 12 points to each score in the distribution will: Select one: A. increase the mean by 12 but have no effect on the standard deviation. B. increase the mean by 12 and the standard deviation by the square root of 12. C. increase the mean and the standard deviation by 12. D. increase the standard deviation by the square root of 12 but have no effect on the mean.

increase the mean by 12 but have no effect on the standard deviation. --Adding 12 points will increase the mean and other measures of central tendency by 12 points but will have no effect on the standard deviation or other measures of variability.

Alcohol-Induced Sleep Disorder most often involves which of the following symptoms? Select one: A. increased wakefulness, restless sleep, and vivid dreams B. abnormal physiological and behavioral symptoms during sleep C. excessively long nocturnal sleep and excessive sleepiness during the day D. difficulty falling and staying asleep with a reduction in REM (dream) sleep

increased wakefulness, restless sleep, and vivid dreams --Although alcohol initially produces sleepiness, this is followed by insomnia, restless sleep, and increased REM sleep, often with vivid, anxiety-laden dreams.

Amitriptyline is often used as a treatment for neuropathic pain, and it exerts its effects by: Select one: A. increasing norepinephrine and serotonin levels. B. increasing norepinephrine levels and decreasing serotonin levels. C. decreasing serotonin and dopamine levels. D. increasing serotonin levels and decreasing dopamine levels.

increasing norepinephrine and serotonin levels --Amitriptyline increases the levels of both norepinephrine and serotonin by inhibiting their reuptake.

Which of the following would NOT be useful for increasing a test's reliability coefficient? Select one: A. increasing the sample variance B. ensuring that the average p value is close to .50 C. increasing the heterogeneity of the content domain D. increasing the number of items from 50 to 100

increasing the heterogeneity of the content domain --A test's reliability coefficient, like any other correlation coefficient, is affected by a number of factors. This would have the effect of decreasing the reliability coefficient, especially the coefficient of internal consistency.

The "storm and stress theory" of adolescent mental health proposes that most adolescents experience extensive physical, social, and psychological turmoil. Research conducted in the past three or four decades on this topic: Select one: A. confirms that most adolescents do experience substantial "storm and stress." B. demonstrates that "storm and stress" is characteristic of adolescents in westernized cultures only. C. indicates that, for the large majority of individuals, adolescence is not a time of significant "storm and stress." D. shows that "storm and stress" is largely due to increases in family conflicts during the adolescent period.

indicates that, for the large majority of individuals, adolescence is not a time of significant "storm and stress." --Studies suggest that, while adolescents may exhibit greater emotional lability, only about 10 to 20% of adolescents exhibit some type of severe emotional turmoil, which is approximately the same percent as found in the adult population.

Phallic is to initiative vs. guilt as latency is to: Select one: A. autonomy vs. shame and doubt. B. identity vs. role confusion. C. industry vs. inferiority. D. generativity vs. stagnation.

industry vs. inferiority --Freud's phallic and Erikson's initiative vs. guilt stages occur at about ages 3 to 6, while Freud's latency and Erikson's industry vs. inferiority stages occur at about ages 6 to 12.

Wolfgang Kohler's research on animal learning and animal cognition led to his conclusion that learning is: Select one: A. biologically based. B. insightful. C. the result of trial-and-error. D. the result of reinforcement and punishment.

insightful --Kohler's research revealed that like humans, animals seem to experience an "aha" experience ("insight") while solving problems.

As defined by Carl Jung, individuation refers to: Select one: A. being able to separate one's intellectual and emotional functioning. B. differentiating between self and object. C. experiencing a state of anonymity. D. integrating parts of the self to create a unique identity.

integrating parts of the self to create a unique identity. --As defined by Jung, individuation is the goal of psychological development and involves integrating the conscious and the unconscious elements of the psyche.

An African American individual in the immersion substage of the immersion-emersion stage of Cross's (1991) Black Racial Identity Development Model will exhibit which of the following? Select one: A. a lack of interest in race or racial issues B. identification with the White (dominant) culture C. functional paranoia D. intense involvement in African American culture

intense involvement in African American culture --An African American individual in the immersion substage is immersed in African American culture, which is evident in his/her interests, speech, style of dress, actions, choice of friends, etc. This individual also feels a great deal of rage toward Whites. During the emersion substage, the person's intense emotions subside but he/she continues to reject White culture and begins to develop a more sophisticated African American identity.

Egan and Perry's (2001) multidimensional model of gender identity consists of five components - membership knowledge, gender typicality, gender contentedness, felt pressure, and: Select one: A. intergroup bias. B. gender stability. C. integrative awareness. Incorrect D. gender constancy.

intergroup bias --This answer lists the fifth component identified by S. K. Egan and D. G. Perry. As defined by these authors, membership knowledge refers to the individual's knowledge about his/her own gender; gender typicality is the degree to which the individual perceives his/her characteristics to be similar to those of individuals of the same gender; gender contentedness is the extent to which the individual is satisfied with his/her own gender; felt pressure is the degree to which the individual feels pressure to conform to gender group norms; and intergroup bias refers to the belief that one's same-sex group is superior to the other sex group.

Members of an outpatient therapy group are likely to rank which of Yalom's therapeutic factors as most important? Select one: A. interpersonal input, catharsis, cohesiveness, and self-understanding B. self-understanding, universality, identification, and instillation of hope C. family re-enactment, catharsis, altruism, and interpersonal input D. interpersonal input, interpersonal output, universality, and identification

interpersonal input, catharsis, cohesiveness, and self-understanding

In discussing a depressed client, a therapist says the client's problems are due to the fact that she did not experience normal grief following the death of her mother three years ago. He says that the focus of treatment will be on helping the client go through the grieving process and restore her interests and friendships so that she can overcome her loss. Apparently, this therapist is a practitioner of: Select one: A. rational-emotive therapy. B. interpersonal therapy. C. object-relations therapy. D. reality therapy.

interpersonal therapy --Interpersonal therapy (IPT) defines four key problem areas presented by depressed patients: grief, interpersonal role disputes, role transition, and interpersonal deficits. These areas are the primary targets of therapy.

StartofPracticeExam6Questions: Fairburn's (2008) CBT-E (cognitive-behavioral therapy - enhanced) adds which of the following to traditional cognitive-behavioral therapy for individuals with Bulimia Nervosa? Select one: A. interventions aimed at boundary disturbances B. interventions aimed at interpersonal relationships C. interventions that foster insight into the cause of the disorder D. interventions derived from Rogerian therapy

interventions aimed at interpersonal relationships --C. G. Fairburn's CBT-E incorporates interventions that target perfectionism, low self-esteem, interpersonal factors, and emotional factors

An individual at Level 8 on the Rancho Scale of Cognitive Functioning Revised: Select one: A. is nonresponsive to sounds or light and appears to be in a state of deep sleep. B. is confused, agitated, and incoherent, may exhibit bizarre behavior, and is unable to care for him/herself. C. is alert and oriented and can remember and integrate remote and recent events but may have some impairments in judgment, planning, and abstract reasoning. D. is functioning at an intellectual level that is superior for his/her age, education, and demographic background.

is alert and oriented and can remember and integrate remote and recent events but may have some impairments in judgment, planning, and abstract reasoning.

Bell and Ainsworth (1972) conducted one of the first longitudinal studies on the impact of maternal responsiveness on infant crying. The results of their research indicated that a mother's prompt and consistent response to her infant's crying during the first few months of the infant's life: Select one: A. has no effect on the amount of infant crying in subsequent months. B. is associated with an increase in the amount of infant crying in subsequent months. C. is associated with a decrease in the amount of infant crying in subsequent months. D. is associated with a decrease in the amount of infant crying in subsequent months only for infants high in irritability.

is associated with a decrease in the amount of infant crying in subsequent months. --Bell and Ainsworth (1972) found that consistent and prompt maternal response to infant crying during the first three months of the infant's life was associated with a reduced frequency and shorter duration of crying and fussing in subsequent months. Note, however, that their study was criticized on methodological grounds and that subsequent studies have provided both confirming and disconfirming results.

A patient who has been taking antipsychotic medication for a lengthy period of time is being slowly taken off the drug due to the development of tardive dyskinesia. Soon after the drug is completely stopped, the patient exhibits an increase in the symptoms of tardive dyskinesia. This: Select one: A. means that the patient should never have been given antipsychotic medication in the first place. B. is common, and the initial increase in symptoms will likely be followed by a reduction. C. suggests that the symptoms are not due to the antipsychotic medication but, instead, to unrelated brain pathology. D. indicates that the patient will not benefit from withdrawal of the drug.

is common, and the initial increase in symptoms will likely be followed by a reduction. --Many patients show a temporary increase in symptoms of tardive dyskinesia when an antipsychotic drug is withdrawn. However, over time, the symptoms gradually reduce in severity.

In multiple regression, an inverse correlation between a predictor and the criterion: Select one: A. indicates that a mistake has been made. B. suggests that the predictor should be eliminated from the analysis. C. is indicated by a low regression coefficient. D. is indicated by a negative regression coefficient.

is indicated by a negative regression coefficient --In a multiple regression equation, the direction of the correlation between a predictor and criterion is indicated by the sign of the regression coefficient. When there is an inverse (negative) correlation between the variables, the sign is negative.

The nocturnal-only subtype of Enuresis: Select one: A. is more common in girls and occurs most often during REM sleep. B. is more common in girls and occurs most often during non-REM sleep. C. is more common in boys and occurs most often during REM sleep. D. is more common in boys and occurs most often during non-REM sleep.

is more common in boys and occurs most often during non-REM sleep. --The nocturnal-only subtype occurs most often during the first third of the night during non-REM, slow-wave sleep and is more common in boys than girls.

Childhood-onset Obsessive-Compulsive Disorder: Select one: A. is about equally common in boys and girls. B. is four times more common in girls than boys. C. is two times more common in girls than boys. D. is more common in boys than girls.

is more common in boys than girls. --While OCD is about equally common in male and female adults, because of its earlier onset in males, it is more common in male children than in female children.

Solitary (non-social) play: Select one: A. is the predominant form of play for children between one and three years of age. B. is useful as an indicator of cognitive development during the preschool years. C. may be indicative of developmental delays if it constitutes a significant portion of the child's play after age three. D. is not necessarily less mature than group play and, in some circumstances, may be associated with a high level of competence.

is not necessarily less mature than group play and, in some circumstances, may be associated with a high level of competence. --The studies have generally shown that nonsocial play is not necessarily more immature than group play. In fact, because it often consists of goal-directed or educational activities or activities involving large-motor movements, it is often associated with high levels of competence.

The relationship between authoritative parenting and children's academic achievement: Select one: A. is stronger for children who had an easy temperament as babies. B. is observed for children but not for adolescents whose school performance is more highly affected by peers. C. is observed among children as well as among adolescents. D. is stronger among children who have lower levels of intelligence.

is observed among children as well as among adolescents.

In children, Generalized Anxiety Disorder: Select one: A. usually first develops after a recent loss such as parental divorce or death of a family member. B. is usually manifested as autonomic hyperarousal and other prominent physical symptoms. C. is often manifested as excessive concern about performance or competence at school. D. typically involves crying, tantrums, freezing, and clinging to family members, especially among those under 7 years of age.

is often manifested as excessive concern about performance or competence at school. --Children with GAD often worry excessively about their competence or the quality of their performance and this worry is frequently related to schoolwork and performance in sports activities.

When using structural equation modeling, the "fit" between the proposed causal model and the obtained data can be evaluated using the chi-square test for goodness-of-fit. However, one problem with using the chi-square test for this purpose is that: Select one: A. it is very sensitive to the sample size. B. it is very insensitive to the sample size. C. it is difficult to interpret when there are more than three predictors. D. it is difficult to interpret when there are more than three criteria.

it is very sensitive to the sample size. --The chi-square test is very sensitive to sample size, with large samples producing statistically significant results even when there is a small difference between the proposed model and the obtained data. Because the goal in using the chi-square test in this situation is to NOT obtain a significant difference (i.e., you want the data to fit the model), a large sample can be problematic.

A potential problem with the Premack Principle is that: Select one: A. it is often difficult to identify an alternative behavior that serves the same function as the target behavior. B. it may not be possible to apply the reinforcer immediately following the target behavior. C. for some people, secondary reinforcers have little value. D. it is difficult to extinguish a behavior without simultaneously reinforcing an alternative one.

it may not be possible to apply the reinforcer immediately following the target behavior. --The Premack Principle entails reinforcing a low-frequency behavior with a high-frequency behavior or activity. The high-frequency behavior or activity may not be immediately available, which can reduce the effectiveness of the intervention since positive reinforcement is most effective when the reinforcer can be delivered immediately following the target behavior.

One criticism of anger management training for children is that its positive effects are limited because: Select one: A. children are unable to develop control over angry feelings and behaviors. B. children tend to blame others for their anger. C. it places too much emphasis on behavior (versus cognition). D. it places too much emphasis on the individual (versus social) factors.

it places too much emphasis on the individual (versus social) factors. --One problem with most anger management training programs is that they focus on the individual and ignore the role of interpersonal or systemic factors. The importance of broadening the focus is supported by research showing that targeting individual, family, peer, and community relationships increases the effectiveness of anger management programs

When conducting a functional behavioral assessment, a behavior's function is described in terms of: Select one: A. the skills required to perform it. B. the criteria used to measure it. C. its antecedents and consequences. D. its physical characteristics.

its antecedents and consequences. --A functional behavioral assessment involves determining the environmental factors (antecedents and consequences) that contribute to its occurrence.

Prosopagnosia is believed to be due to lesions in the: Select one: A. junction of the occipital, temporal, and parietal lobes. B. junction of the frontal and occipital lobes. C. precentral gyrus. D. central sulcus.

junction of the occipital, temporal, and parietal lobes. --Knowing that prosopagnosia is a type of visual agnosia and that the occipital lobe mediates visual processing would have helped you identify the correct answer to this question. The occipital lobe is involved in visual processing, while the temporal lobe mediates long-term memory; and damage to these areas plus the parietal lobe may produce an inability to recognize familiar faces.

When treating a patient with Delirium, a priority is to identify and then treat or remove its cause. In addition, it is important to: Select one: A. make sure that the patient is provided with adequate stimulation. B. keep the patient in a quiet room where he/she can be monitored by a family or staff member. C. administer a neuroleptic to reduce the patient's disorientation and agitation. D. make sure family members and friends visit regularly.

keep the patient in a quiet room where he/she can be monitored by a family or staff member. --Delirium is characterized by disorientation and confusion, so an important goal is provide an environment that decreases disorientation. For example, it is important to reduce distractions and to provide a quiet, well-lit room and constant monitoring.

Jenny and Craig are engaging in intercontinental dating via video chatting and other electronic means since they initially met through a dating website. She lives in Hong Kong, and he lives in Belfast. What does research indicate could be a potential problem as Jenny and Craig try to grow their relationship? Select one: A. The long distance. B. Lack of quality and depth in their relationship. C. Their cultural differences. D. None of the above.

lack of quality and depth in their relationship --The use of the internet for interpersonal communication is not the same as traditional face-to-face communication in enriching the quality of life. Online communication does have the potential to have an adverse effect on peoples perceived life quality. Although the relative lack of strong ties or in-depth quality that can result in internet communication cannot be the sole reason for the negative effect of online communication for quality of life. An absence of nonverbal cues: lack of warmth, and less demand for engagement in internet communication results in impersonality, shallow interactions, and difficulty in building social support. These are reasons why there is a negative contribution of online communication to perceived quality of life.

Which of the following is listed in the DSM as a diagnostic (versus associated) feature for Antisocial Personality Disorder? Select one: A. inflated sense of self B. lack of remorse C. lack of empathy D. superficial charm

lack of remorse --For this diagnosis, the DSM requires the presence of at least three of seven characteristic behaviors. One of the seven is a lack of remorse.

Latent class analysis (LCA) is most similar to which of the following techniques? Select one: A. multitrait-multimethod matrix B. cluster sampling C. latent trait analysis D. ANCOVA

latent trait analysis --LCA is used to identify the underlying latent structure of a set of observed data. a. Incorrect The multitrait-multimethod matrix is used to assess construct (convergent and divergent) validity. Latent trait analysis (LTA) is also used to identify the underlying latent structure of a set of observed data. A primary difference between the two techniques is that, in LCA, the latent variable that determines the structure is nominal; while, in LTA, the latent variable is continuous.

Kochanska's (1997) research suggests that, during the toddler years, the optimal parental behavior for ensuring the development of conscience in a child depends on the child's: Select one: A. activity level. B. level of fearfulness. C. intelligence. D. attachment.

level of fearfulness --A number of researchers have found that fearful children score higher on measures of conscience, and Kochanska extended this finding by showing that the relationship between fearfulness and conscience development is affected by caregivers' socialization practices. Specifically, she found that, among fearful toddlers, conscience development is fostered when the mother uses "gentle discipline." In contrast, among fearless toddlers, conscience development depends more on a secure mother-child attachment and maternal responsiveness.

Dr. Billings, a psychologist, asks a graduate student to help him with a research project. The student does most of the initial library research, administers the tests and interviews to subjects, and writes some of the article that Dr. Billings will submit to a professional journal. When the article is published, Dr. Billings should make sure that the graduate student is: Select one: A. listed as the first author. B. listed as a co-author. C. acknowledged in a footnote. D. acknowledged in an introductory statement.

listed as a co-author. --The graduate student has written some of the article and, therefore, should be listed as a co-author.

As the result of head trauma, a young man experiences damage to the primary motor cortex in the left cerebral hemisphere. This is most likely to have which of the following effects? Select one: A. loss of voluntary movement in his right hand and fingers B. loss of voluntary movement in his left hand and fingers C. loss of involuntary movement on the right side of his body D. loss of involuntary movement on the left side of his body

loss of voluntary movement in his right hand and fingers --For most functions, the right side of the body is controlled by the left hemisphere and vice-versa. The motor cortex is involved in the regulation of voluntary movements, and damage to this area is most likely to affect voluntary movements in distal parts (e.g., hands and fingers) of the opposite side of the body.

On the MMPI-2, an attempt to "fake good" is suggested by which of the following? Select one: A. elevated F scale score with low L and K scale scores. B. elevated F and L scale scores with a low K scale score. C. low F scale score with elevated L and K scale scores. D. low K scale score with a moderately high L scale score and a very high F scale score.

low F scale score with elevated L and K scale scores. --A low score on the F (infrequency) scale indicates an absence of psychopathology, social conformity, or an attempt to "fake good"; a high score on the L (lie) scale suggests an attempt to "fake good," conformity, or denial; and a high score on the K (correction) scale suggests an attempt to "fake good," denial, or a lack of insight.

When a test has high sensitivity, this means that there is a: Select one: A. low chance of false negatives and high chance of false positives. B. low chance of false negatives and low chance of false positives. C. high chance of false negatives and high chance of false positives. D. high chance of false negatives and low chance of false positives.

low chance of false negatives and high chance of false positives --Sensitivity refers to the proportion of people with the condition who are correctly identified by the test and is calculated by dividing the true positives by the true positives plus false negatives. When the sensitivity is high, this means that most of the people with the disorder will be identified as having the disorder by the test (i.e., there will be few false negatives) but that there will be some people without the disorder who will also be identified as having the disorder (i.e., there will be some false positives).

Research on the impact of cultural identification on substance abuse among Native American youth has found that the risk for abuse is: Select one: A. lowest for those who are acculturated into the non-Native American (mainstream) culture. B. lowest for those who can adapt to both Native American and non-Native American cultures. C. highest for those who strongly identify with the Native American culture and reject the non-Native American culture. D. highest for those who do not strongly identify with either the Native American or non-Native American culture.

lowest for those who can adapt to both Native American and non-Native American cultures. --The studies have shown that either strong identification with one's own Native American tribe or a bicultural identity is associated with a lower risk for substance use than a strong identification with the mainstream culture only.

Studies investigating dissociation in children have found that: Select one: A. maltreated children and nonmaltreated children exhibit similar levels of dissociation. B. maltreated children exhibit more dissociative symptoms than nonmaltreated children do and this difference is apparent during the preschool years. C. maltreated children exhibit more dissociative symptoms than nonmaltreated children do but this difference is not apparent until the elementary school years. D. maltreated children exhibit more dissociative symptoms than nonmaltreated children do but this difference is not apparent until the middle-school years.

maltreated children exhibit more dissociative symptoms than nonmaltreated children do and this difference is apparent during the preschool years. --The studies have shown that, when compared to their nonmaltreated peers, maltreated preschool children exhibit more dissociation than nonmaltreated children do.

The area of the brain referred to as the "midbrain" contains all of the following structures except: Select one: A. inferior colliculus. B. superior colliculus. C. substantia nigra. D. mammillary bodies.

mammillary bodies --The mammillary bodies are part of the hypothalamus and are located in the diencephalon which, along with the telencephalon, make up the forebrain.

Research on the impact of parents' gender stereotypes about math ability on their children's interest in math has found that: Select one: A. maternal and paternal stereotypes have an impact on the math interest of boys but not girls. B. maternal and paternal stereotypes have an impact on the math interest of both boys and girls. C. maternal stereotypes have an impact on the math interest of boys but not girls. D. paternal stereotypes have an impact on the math interest of girls but not boys.

maternal and paternal stereotypes have an impact on the math interest of both boys and girls. --Studies have found that, while the gender discrepancy in math achievement has declined in recent years, girls continue to be less likely to choose a math-related major in college or career in mathematics. In addition, the research has confirmed that maternal and paternal gender-related attitudes, beliefs, and behaviors are a key contributor to the disparity in interest in math and perceptions about math ability. Interestingly, a longitudinal study by Bleeker et al. suggests that the father's gender stereotypes may have a different impact than the stereotypes of the mother - i.e., while father's stereotypes are associated with an increased interest in math among boys and a decreased interest among girls, the mother's stereotypes are associated with a decreased interest for both boys and girls (J. E. Bleeker et al., "I can, but I don't want to":

To encourage college students to participate in her research study, Dr. LaPlace offers students free admittance to a popular four-hour workshop ("Finding a Meaningful Relationship and Keeping It") that is offered by a colleage of hers. This policy: Select one: A. is unacceptable since it is coercive and, therefore, violates the provisions of the Ethics Code. B. is unacceptable since it represents a multiple relationship and, therefore, violates the provisions of the Ethics Code. C. is unacceptable since it represents a "conflict of interest" and, therefore, violates the provisions of the Ethics Code. D. may be acceptable as long as Dr. LaPlace is careful to clarify any risks, obligations, and so on with the students.

may be acceptable as long as Dr. LaPlace is careful to clarify any risks, obligations, and so on with the students.

For retention of information in long-term memory, which of the following seems to be the most critical? Select one: A. visual encoding B. repetition C. time D. meaningful organization

meaningful organization --encoding semantically--that is, in terms of meaning--helps ensure that information will be transferred to long-term memory and that the information is more easily retrieved.

Research on primacy and recency effects has shown that: Select one: A. memory for the first item in a list decreases while memory for the last item increases as the delay between study and recall increases. B. memory for the first item in a list increases while memory for the last item decreases as the delay between study and recall increases. C. memory for the first and the last item decreases as the delay between study and recall increases. D. memory for the first and the last item increases as the delay between study and recall increases.

memory for the first item in a list increases while memory for the last item decreases as the delay between study and recall increases. --The recency-primacy shift contradicts, to some extent, the axiom that "memory tends to fade over time." Specifically, the research has shown that, as the delay between study and recall increases, memory for the last item in a list decreases but memory for the first item increases -- i.e., there is a shift from a recency effect to a primacy effect.

As defined by Piaget, the ability to engage in deferred imitation depends on the development of: Select one: A. transductive reasoning. B. mental representation. C. animistic thinking. D. self-recognition.

mental representation --Mental representation is also referred to as representational or symbolic thought and is the ability to hold an image in one's mind for a period beyond one's immediate experience. It develops at about 18 to 24 months of age and is responsible for the ability to engage in deferred imitation, use language, and participate in symbolic play.

Bartlett Barmicide says he's continued having hallucinations ("flashbacks") several months after he stopped using a drug. He also states that he realizes that this is probably due to the drug. Bartlett most likely used which of the following? Select one: A. an inhalant B. methamphetamine C. cocaine D. mescaline

mescaline --The DSM diagnosis of Hallucinogen Persisting Perception Disorder is characterized by re-experiencing perceptual disturbances reminiscent of those experienced during earlier Hallucinogen Intoxication. Of the substances listed in the answers, only mescaline is a type of hallucinogen.

A therapist relying on a solution-focused family therapy approach would be most interested in which of the following? Select one: A. miracle and scaling questions B. boundary disturbances C. joining and mimesis D. schism and skew

miracle and scaling questions --Solution-focused therapists use various types of questions to identify solutions to a client's problems. The miracle question involves asking what the individual would notice first if, as the result of a miracle, his/her problem is suddenly gone. Scaling questions are used to determine how a client currently feels.

According to Gyorgy Gergely (1994), visual feature representation is a necessary precondition for: Select one: A. gender identity development. B. mirror self-recognition. C. attachment. D. social referencing.

mirror self-recognition --According to G. Gergely, the ability to recognize oneself in the mirror (which occurs between 1-1/2 and 2 years in humans) requires a certain level of cognitive development. One cognitive prerequisite is the construction of "a visual feature representation of the typical physical appearance of the not-directly-visible parts of ... [the] body" (p. 55)

In a study examining affiliation, experimental subjects (introductory psychology students) are told that they will be receiving a series of electric shocks. Subjects are then given the opportunity to either wait alone, wait with other students who will be participating in the same experiment, or wait with other students who are merely waiting to see their faculty advisors. Results of the study will most likely confirm the hypothesis that: Select one: A. misery loves company B. misery loves miserable company. C. misery loves unmiserable company. D. misery loves no company.

misery loves miserable company --Schachter (1959) found that high anxiety subjects not only prefer to affiliate, but prefer to affiliate with others in similar circumstances in other words, "misery loves miserable company." Research on affiliation has found that highly anxious subjects tend to prefer to wait for an experiment to begin with other highly anxious subjects.

A researcher wants to compare the immediate and long-term effects of three different treatments for PTSD. Consequently, she randomly assigns individuals who were recently exposed to the same trauma to either the EMDR, stress inoculation, or in vivo exposure group and then assesses their symptoms one week, one month, and three months after the end of treatment. The researcher is using which of the following research designs? Select one: A. between groups B. within subjects C. counterbalanced D. mixed

mixed --A design is referred to as "mixed" when it involves at least one between-groups variable and one within-subjects variable.

A test developer uses a multitrait-multimethod matrix to organize the data she has collected in a validation study of her newly developed self-report measure of self-esteem. The matrix indicates that the correlation between her self-report measure of self-esteem and an established (previously validated) teacher rating of self-esteem is .91. This correlation coefficient is referred to as a: Select one: A. monotrait-heteromethod coefficient. B. monotrait-monomethod coefficient. C. heterotrait-heteromethod coefficient. D. heterotrait-monomethod coefficient.

monotrait-heteromethod coefficient --The monotrait-heteromethod coefficient indicates the correlation between two different measures of the same trait - e.g., two different measures of self-esteem.

Research by Baker and Green (2005) found that, when compared to older adults with chronic pain, younger adults with chronic pain tend to report: Select one: A. more pain intensity but fewer symptoms of depression. B. less pain intensity and fewer symptoms of depression. C. more pain intensity and more symptoms of depression. D. less pain intensity but more symptoms of depression.

more pain intensity and more symptoms of depression. --Baker and Green found that, among White and Black adults with chronic pain, adults under 50 reported more depressive symptoms, greater pain intensity, and more problems coping with pain than did those age 50 and over.

Which of the following techniques would be appropriate for using scores on measures of self-esteem, hopelessness, and perceived social support to classify adolescents as either non-depressed, vulnerable to depression, or depressed? Select one: A. linear regression analysis B. multiple regression analysis C. multiple discriminant function analysis D. principal components analysis

multiple discriminant function analysis --Multiple discriminant function analysis is an extension of discriminant function analysis when multiple predictors will be used to classify individuals into one of three or more criterion groups.

The psychoanalyst Adolph Stern provided the first organized clinical description of the borderline patient. Of the ten basic characteristics Stern delineated, which of the following did he consider to be the most primary? Select one: A. inhibited aggression B. fixation C. emotional dysregulation D. narcissism

narcissism --Stern considered the difficulties experienced by the borderline patient to be secondary to narcissism and viewed narcissism as arising from a serious disturbance in the early mother-child relationship. (Note that other psychoanalytically-oriented theorists have identified other factors as being primary: Kernberg, for example, emphasizes the role of excessive aggression.)

A 42-year-old woman who has just begun taking lithium carbonate as a treatment for Bipolar Disorder will most likely experience which of the following side effects? Select one: A. constipation, abdominal pain, and excessive salivation B. nausea, vomiting, diarrhea, and abdominal pain C. increased appetite, carbohydrate craving, and weight gain D. nausea, vomiting, constipation, and salt craving

nausea, vomiting, diarrhea, and abdominal pain --Nausea, vomiting, diarrhea, abdominal pain, anorexia, and dry mouth are common early gastrointestinal side effects of lithium. These symptoms usually appear within a few weeks and, if they occur late in therapy, may be a sign of lithium toxicity.

Scoring and interpretation of the Thematic Apperception Test (TAT) usually involves considering which of the following? Select one: A. form quality and content B. emotional control/lability C. needs and press D. general and specific attitudes

needs and press --Most scoring and interpretation systems for the TAT rely on Murray's concepts of needs (internal determinants of behavior), press (environmental determinants of behavior), and thema (interactions between needs and press).

The kappa statistic is used as a measure of reliability when data are: Select one: A. nominal or ordinal (discontinuous). B. interval or ratio (continuous). C. nonlinear. D. metric.

nominal or ordinal (discontinuous) --The kappa statistic is used to evaluate inter-rater reliability. The kappa statistic is used to measure the consistency of ratings assigned by two raters when data are nominal or ordinal. (Note that some authors use the term "discontinuous" to refer to nominal and other discrete data - i.e., data that represents noncontinuous categories.)

To determine how well an examinee did on a test compared to other examinees, you would use: Select one: A. criterion-referenced interpretation. B. domain-referenced interpretation. C. objectives-referenced interpretation. D. norm-referenced interpretation.

norm-referenced interpretation --When using norm-referenced interpretation, an examinee's test performance is compared to the performance of members of the norm group (other people who have taken the test).

Al A., a lonely, single, middle-aged psychologist and college professor, is invited to a play by a graduate student, Betty B., who Al finds sexually attractive. Betty has returned to school after a 15-year absence and has been divorced for three years. If Al accepts Betty's invitation, he will: Select one: A. be acting unethically because the Ethics Code prohibits professors from dating students. B. not be acting unethically as long as he doesn't become sexually involved with Betty. C. not be acting unethically as long as Betty is a graduate student in a department other than the one Al teaches in. D. not be acting unethically because this situation is not covered by the Ethics Code.

not be acting unethically as long as Betty is a graduate student in a department other than the one Al teaches in. --If Betty is a student in another department, it would likely be acceptable for Al to accompany her to the play and become involved with her.

Administering Ritalin to a child to confirm or rule out a diagnosis of ADHD is: Select one: A. contraindicated since Ritalin increases activity levels and reduces attention in normal children. B. not useful because only a minority of children with ADHD respond to Ritalin. C. not useful since the effects of Ritalin are similar for children with and without ADHD. D. useful, especially in cases where the child's symptoms are highly variable.

not useful since the effects of Ritalin are similar for children with and without ADHD.

The purpose of "rotation" in factor analysis is to: Select one: A. obtain a pattern of factor loadings that is easier to interpret. B. reduce the impact of measurement error on the factor loadings. C. reduce the magnitude of the communalities. D. obtain a clearer pattern of communalities.

obtain a pattern of factor loadings that is easier to interpret. --The pattern of factor loadings in the initial factor matrix is often difficult to interpret, so the factors are rotated to obtain a pattern that's easier to interpret.

A licensed psychologist who is working in an isolated rural community finds that some of his clients have problems that are beyond his training and expertise. The psychologist should: Select one: A. continue seeing the clients only if there is no other psychologist available. B. use only those interventions he feels competent to use in treating the clients. C. obtain appropriate consultation by phone. D. refuse to see the clients until he receives adequate training.

obtain appropriate consultation by phone.

Dr. X has been asked to administer a psychological test to an individual who requires special accommodations because of a physical disability. The test was not developed or normed for individuals with this disability. The APA's Guidelines for Test User Qualifications recommend that, in this situation, psychologists: Select one: A. refuse to administer the test. B. refuse to administer the test unless there are no alternative methods of assessment. C. exercise "special care" when administering and scoring the test and interpreting its results. D. obtain legal advice on relevant legal requirements regarding the administration and use of the test.

obtain legal advice on relevant legal requirements regarding the administration and use of the test --In their summary, Turner et al. note that there may be legal requirements that apply to the administration and use of tests when an individual requires special accommodations. To ensure that these requirements are met, psychologists should consider them and, when appropriate, "obtain legal advice on legal and regulatory requirements regarding appropriate administration of tests and use of test data when assessing individuals with disabilities" (p. 1103). Note that Standard 10.2 of the Standards for Educational and Psychological Testing also addresses this issue and requires test users who do not have adequate knowledge or experience in modifying tests for individuals with disabilities to seek guidance from a consultant or supervisor who has expertise.

Research on the universality of the "Big Five" personality traits has generally found that the traits are cross-culturally valid. However, of the five traits, empirical evidence for the universality of __________ is least compelling. Select one: A. openness to experience B. neuroticism C. conscientiousness D. extraversion

openness to experience --The best general conclusion about the Big Five is that they are universal. However, factor analyses have not been entirely consistent, with evidence for openness to experience showing the weakest replicability in some Asian and other non-Western countries.

An examinee's score on which of the following Rorschach variables is most useful for estimating his/her level of intelligence? Select one: A. organizational activity B. perceptual thinking C. WSum6 D. S-CON

organizational activity --Organizational activity reflects the extent to which the examinee organizes the various elements of the inkblot into a unified image and, of the Rorschach variables listed in the answers, correlates most highly with intelligence. Other Rorschach variables that correlate with intelligence include developmental quality, lambda, and F%.

When treating a client with a paraphilia, which of the following techniques would be most useful for replacing the inappropriate stimulus that currently elicits sexual arousal with a more appropriate stimulus? Select one: A. covert sensitization B. flooding (in vivo exposure) C. habit reversal training D. orgasmic reconditioning

orgasmic reconditioning --When using orgasmic reconditioning, the client is "reconditioned" to masturbate to a more appropriate stimulus. This is accomplished by having the client begin to masturbate while fantasizing about the inappropriate stimulus and then switch his fantasy to the more appropriate stimulus.

"I have two feets" and "I runned fast" are examples of the grammatical error known as: Select one: A. telegraphic speech B. holophrastic speech C. overextension D. overregularization

overregularization --Young children make a number of predictable errors when acquiring language. Overregularization occurs when children overextend grammatical rules - e.g., add "s" to feet to form the plural or "ed" to run to form the past tense.

A patient taking a benzodiazepine develops several undesirable symptoms including insomnia, nightmares, hallucinations, and rage reactions. These symptoms are most suggestive of which of the following? Select one: A. overdose B. anticholinergic effects C. paradoxical agitation D. rebound anxiety

paradoxical agitation --Insomnia and the symptoms listed in the question are the symptoms of paradoxical agitation, which is most likely to occur in people with a history of aggressive behavior or unstable emotional behavior.

Research examining the effects of divorce suggests that maladjustment in children following the divorce of their parents is most likely the result of which of the following? Select one: A. the parent's separation B. parental disharmony C. disruption in lifestyle D. diminished parental control

parental disharmony --Conflict (disharmony) has been found to be more critical than the separation. Children of divorced parents show less maladjustment when the conflict between their parents is minimal.

When conducting research with human participants, a psychologist should be aware that: Select one: A. participants must be debriefed promptly after their participation in the study. B. participants must be debriefed after their participation in a study only when the study involved deception. C. participants must be debriefed promptly if possible or, if necessary to delay the debriefing, psychologists must reduce the risk for harm. D. the decision to debrief is left to the psychologist and involves considering the risk for harm to participants if they are not debriefed.

participants must be debriefed promptly if possible or, if necessary to delay the debriefing, psychologists must reduce the risk for harm. --Research participants should be promptly provided with "appropriate information about the nature, results, and conclusions of the research." When there must be a delay in providing such information, "psychologists take reasonable measures to reduce the risk of harm."

Electroconvulsive shock therapy (ECT) is considered an effective intervention for severe depression that has not been responsive to other treatments or is accompanied by a high suicide risk. However, ECT also often produces adverse side effects including: Select one: A. patchy anterograde amnesia. B. temporary retrograde amnesia. C. patchy anterograde amnesia and temporary retrograde amnesia. D. permanent retrograde amnesia.

patchy anterograde amnesia and temporary retrograde amnesia --Memory loss most often involves patchy anterograde amnesia for three to six months post-ECT and retrograde amnesia for events that occurred within several months prior to ECT. In most cases memories of past events eventually return.

According to Berscheid and Walster's (1974) two-factor theory of love, love is the result of: Select one: A. attraction and arousal. B. passion and intimacy. C. mutual attraction and reciprocal reinforcement D. physiological arousal and a label for it.

physiological arousal and a label for it. --Berscheid and Walster conceptualize love as being similar to other emotions. Support for the two-factor theory is provided, for example, by a study showing that couples who participate in an exciting, arousing game together subsequently report stronger positive feelings toward one another than couples who participate in a more mundane activity.

A psychologist working at an alcohol rehabilitation clinic is conducting a study to determine if treatment length is related to relapse. She adds four weeks to the standard 12-week program and randomly assigns one-half of the current clinic clients to the extended 16-week treatment and the other half to the standard 12-week treatment. Relapse is measured as the number of times an individual drinks during the 3 months following the end of his or her treatment program. The appropriate correlation coefficient for the data collected in this study is: Select one: A. Pearson r. B. tetrachoric. C. point biserial. D. Spearman rho.

point biserial --In this situation, there is one continuous variable (relapse rate) and one dichotomous variable (treatment length -- 12 weeks or 16 weeks). The point biserial coefficient is used when one variable is continuous and the other is a true dichotomy. (In this case, treatment length is a true dichotomy because participants receive either a 12-week or a 16-week program.)

From a "control systems" perspective, when established control mechanisms are no longer effective due to radical changes in the environment, new mechanisms must be sought or developed. These new mechanisms will help the system adapt to environmental changes by providing: Select one: A. negative feedback. B. positive feedback. C. formative evaluations. D. summative evaluations.

positive feedback --Although, under normal circumstances, positive feedback induces change and can result in "runaways," when the system must adapt to radical changes in the environment, it is positive feedback that makes adaptation possible.

The use of shaping to establish a complex behavior depends on which of the following? Select one: A. latent learning B. positive reinforcement C. higher-order conditioning D. successive discrimination

positive reinforcement --Shaping involves reinforcing successive approximations to the target (desired) behavior. When using shaping to establish a new behavior, responses that come closer and closer to the desired behavior are successively reinforced.

According to Cross (2001), an African American adult in which of the following stages of identity development is likely to say that racial discrimination is not a contributor to his problems and that he prefers to see a White therapist? Select one: A. disintegration B. pre-encounter C. emersion D. pseudo-independence

pre-encounter --According to Cross, race has low salience for individuals in the pre-encounter phase. People in this stage are likely to deny the impact of racial oppression and prefer a White therapist.

According to Piaget, the key feature of the _______________ stage is the development of the "semiotic function." Select one: A. concrete operational B. preoperational C. sensorimotor D. formal operational

preoperational --The preoperational stage extends from ages 2 to 7. During this stage, thought becomes symbolic in form.

Studies suggest that the ______________ of Obsessive-Compulsive Disorder are similar across cultures. Select one: A. prevalence rates B. prevalence rates and age of onset C. prevalence rates, age of onset, and gender distribution D. prevalence rates, age of onset, gender distribution, and comorbidity

prevalence rates, age of onset, gender distribution, and comorbidity --The DSM-5 notes that prevalence rates, age of onset, gender distribution, comorbidity, and symptom structure of OCD are similar across cultures (APA, 2013, p. 140). However, the content of obsessions and compulsions may be affected by cultural factors.

As defined by Bandura, self-efficacy beliefs have four sources. These are: Select one: A. prior accomplishments, observations of others, verbal persuasion, and emotional and physiological states. B. external reinforcement, internal (self) reinforcement, vicarious reinforcement, and logical verification. C. observation of others, imitation of others, reinforcement by others, and self-reinforcement. D. observation, cognitive mediation, rehearsal, and successful performance.

prior accomplishments, observations of others, verbal persuasion, and emotional and physiological states. --Bandura's (1982, 1986) theory of self-efficacy proposes that people undertake activities they believe they are capable of performing. These are the four sources of self-efficacy that Bandura has identified.

Experts on cross-cultural counseling note that, when working with African American clients, it is important to recognize that their past experiences in a racist society are likely to have fostered feelings of powerlessness. Consequently, a good strategy for a therapist is to adopt a: Select one: A. non-directive, less active approach. B. problem-solving, time-limited approach. C. color-blind approach. D. neutral, educative approach.

problem-solving, time-limited approach

Which of the following techniques would be most useful for investigating a person's cognitive processes while working on a complex task? Select one: A. production interview B. protocol analysis C. behavioral systems assessment D. event coding

protocol analysis --Protocol analysis involves having the individual think aloud while solving a problem and then evaluating the protocol (transcript) to clarify the individual's problem-solving process.

You receive a phone call from Hermann H., age 28, who says he is "totally miserable" because of the recent breakup with his girlfriend and that he would like to begin therapy with you. During the first session with Hermann, you find out that his political views are completely repugnant to you, and you feel that you would not enjoy working with him. As an ethical psychologist, you should: Select one: A. consult with another psychologist during the course of treatment to make sure your feelings don't interfere with your objectivity. B. discuss the difference in political views with Hermann only if they become relevant to the psychotherapy process. C. see Hermann in therapy until his current crisis is over and then make a referral if necessary. D. provide Hermann with appropriate referrals.

provide Hermann with appropriate referrals. --Psychologists are not required to see every client who seeks their services - and, to do so, would be unethical if the client's characteristics interfere with the provision of adequate services. It's evident objectivity is already impaired as the case scenario indicates the psychologist doesn't think he'll like working with this client, therefore a referral is the best option based on the information provided. Of the responses given, answer D describes the action that is in the best interests of both the client and you (the psychologist).

With regard to the termination of professional services, the Ethics Code requires psychologists to: Select one: A. offer to help the client locate alternative services. B. take actions that protect the best interests of the client. C. provide pretermination counseling and suggest alternative services. D. take action that is consistent with the reason(s) for the termination.

provide pretermination counseling and suggest alternative services.

A legal guardian has been appointed by the court for a 9-year-old child after his biological parents are found guilty of child abuse. The legal guardian brings the child to you for treatment. You should: Select one: A. obtain permission from both of the child's biological parents before providing treatment. B. obtain permission from one of the child's biological relatives before providing treatment. C. obtain permission from the court before providing treatment. D. provide the treatment to the child as requested by the guardian.

provide the treatment to the child as requested by the guardian. --The child's legal representative in this case, the court-appointed guardian, has the right to request treatment for the child. Approval from the court of the child's biological parents or relatives is not necessary.

Research on the effects of divorce on the parent-child relationship has generally shown that, during the initial period following divorce, the relationship between the custodial parent and his/her child often changes. Specifically, during this period, the custodial mother most often: Select one: A. provides harsher but more inconsistent punishment. B. becomes overindulgent and overpermissive. C. spends more time with her child. D. is more concerned about the effectiveness of her parenting skills.

provides harsher but more inconsistent punishment.

Professor Sharp argues against raising the legal drinking age from 18 to 21 on the ground that doing so will only encourage 18-, 19-, and 20-year olds to drink. Apparently, Sharp is familiar with which of the following theories? Select one: A. individuation B. psychological reactance C. inoculation D. expectancy-value

psychological reactance --Sharp's argument is most consistent with the predictions of psychological reactance theory, which proposes that, when people feel their freedom is being threatened, they will do something to restore that freedom (e.g., engage in the legally-prohibited behavior).

Major Neurocognitive Disorder due to HIV infection has been linked to abnormalities in which of the following structures of the brain? Select one: A. putamen and ventral striatum B. suprachiasmatic nucleus C. superior and inferior colliculi D. entorhinal cortex

putamen and ventral striatum --Wang et al. found that Major Neurocognitive Disorder due to HIV infection is associated with decreased dopamine transporters in the putamen and ventral striatum. These structures are part of the basal ganglia, which is involved in voluntary motor activity.

According to classical test theory, X = T + E, where E refers to: Select one: A. random error B. equivalency C. predictive error D. estimated consistency

random error --According to classical test theory, an examinees obtained test score (X) is composed of two components -- T and E. "T" refers to true score variability, while "E" refers to random (measurement) error.

To determine if cigarette smoking is better controlled through aversion therapy, self-hypnosis, or cognitive self-control, a researcher randomly assigns smokers of at least two packs of cigarettes a day to one of the treatment groups and determines the average number of cigarettes each participant smokes each day during the 30 days following administration of the appropriate intervention. The scale of measurement of the dependent variable in this study is which of the following? Select one: A. ordinal B. ratio C. interval D. nominal

ratio --A ratio scale has the properties of order, equal intervals, and an absolute zero point -- and number of cigarettes has all three properties.

Systematic desensitization was originally designed by Wolpe as an application of which of the following? Select one: A. aversive counterconditioning B. reciprocal inhibition C. habituation D. operant extinction

reciprocal inhibition --Use of reciprocal inhibition (counterconditioning) to reduce an undesirable behavior is based on the assumption that a response can be eliminated by replacing it with an incompatible response; e.g., replacing anxiety with relaxation.

As a treatment for alcohol dependence, naltrexone: Select one: A. causes the person to feel sick when taken in combination with alcohol. B. reduces the person's craving for alcohol. C. eliminates symptoms of alcohol withdrawal. D. is used to produce rapid detoxification.

reduces the person's craving for alcohol. --Naltrexone (ReVia) is an opioid receptor antagonist. Naltrexone blocks the reinforcing effects and craving (desire) for alcohol.

The Bonferroni test helps control the experimentwise error rate by: Select one: A. controlling the total number of comparisons that can be made. B. reducing the level of significance for each comparison. C. permitting individual comparisons only after the omnibus test has produced significant results. D. requiring that all comparisons be conducted as two-tailed tests.

reducing the level of significance for each comparison. --The greater the number of planned comparisons, the greater the likelihood that a Type I error will be made. The Bonferroni test controls this possibility by reducing the level of alpha for each comparison.

A client says she is upset because her doctor has suggested she undergo tests for a possible hyperactive thyroid. She says she believes that doctors are always looking for excuses to perform unnecessary procedures in order to make more money. You should: Select one: A. read up on hyperthyroidism so that you can give her an informed opinion. B. have her sign a release of information and contact her physician. C. refer her to an endocrinologist to discuss the purpose of the tests. D. explore the source of her hostility toward doctors.

refer her to an endocrinologist to discuss the purpose of the tests.

A psychology intern considers her supervisor to be likeable, admirable, and accepting. As a consequence, the intern is open and responsive to her supervisor's comments and recommendations. In this situation, the intern is responding to her supervisor's: Select one: A. referent power. B. reward power. C. expert power. D. legitimate power.

referent power --French and Raven's (1959) six bases of social power are described in the Social Psychology chapter of the written study materials. In this situation, the supervisor has become a "significant reference person" for the intern -- i.e., the supervisor's influence stems from the intern's desire to identify with (or "be like") the influential person.

A Type I error occurs when, on the basis of a study's results, a researcher: Select one: A. retains a false null hypothesis. B. rejects a false null hypothesis. C. retains a true null hypothesis. D. rejects a true null hypothesis.

rejects a true null hypothesis.

You receive a letter from the attorney of a former client requesting that you send test information to her from the client's file. The requested information will be used in a court case and includes the test questions, the client's responses to those questions, and the client's test scores. The attorney's letter is accompanied by a signed release from the client. To be consistent with ethical requirements, you should: Select one: A. release all test information as requested. B. release only a summary of the client's test information. C. release the clients responses and test scores but not the test questions. D. refuse to release any information to anyone other than a mental health professional.

release the clients responses and test scores but not the test questions. --Standards 9.04 and 9.11 of the APA's Ethics Code distinguish between test data and test material and state that test data may be provided to an attorney or other person listed in a client's release but that test material should not be released because doing so may constitute a breach of test integrity and security. As defined in Standard 9.11, test material includes "manuals, instruments, protocols, and test questions or stimuli." Of the answers given, this one is most consistent with the provisions of the APA's Ethics Code and with Paragraph IV.11 of the Canadian Code of Ethics for Psychologists.

In the context of test construction, __________ refers to the extent to which test items contribute to achieving the stated goals or purpose of testing. Select one: A. relevance B. reliability C. significance D. verification

relevance --Test items are relevant when they are germane to the purpose of testing. For example, some educators use a "test relevance index" to determine the extent to which test items on achievement tests are relevant to the goals of the curriculum.

Howard et al.'s (1986) phase model proposes that a client's progress in therapy occurs in three predictable phases. These are: Select one: A. unfreezing, changing, and refreezing. B. remoralization, remediation, and rehabilitation. C. engagement, exploration, and evaluation. D. contemplation, action, and termination.

remoralization, remediation, and rehabilitation --As described by Howard et al., remoralization is an improvement in the subjective sense of well-being, remediation is a reduction in symptoms, and rehabilitation involves improvements in overall functioning.

A psychologist conducting a study to determine the effectiveness of backward conditioning with human participants will use which of the following procedures? Select one: A. Repeatedly present the CS before the US. B. Repeatedly present the US before the CS. C. Apply an intermittent schedule of reinforcement after a continuous schedule has been used to establish a response. D. Apply a continuous schedule of reinforcement after an intermittent schedule has been used to establish a response.

repeatedly present the US before the CS. --Backward conditioning is just the opposite of forward conditioning and involves presenting the US prior to the CS. Backward conditioning is not usually effective.

In adults, the early motor symptoms of Huntington's disease most often involve: Select one: A. tingling, numbness, and weakness in one limb. B. repetitive movements in the extremities and face. C. slowness and poverty of movement. D. difficulty swallowing and speaking.

repetitive movements in the extremities and face. --For many adults, early motor symptoms include rapid, repetitive, involuntary movements in the extremities and face - e.g., "piano-playing" movements of the fingers and facial grimaces.

Freud (1984) introduced the concept of "defense" in his description of hysteria and attributed hysteria to which of the following defense mechanisms? Select one: A. projection B. reaction formation C. sublimation D. repression

repression --Freud's initial explanation for hysteria was that it represents an attempt to eliminate memories related to early psychic trauma from consciousness. Repressions is considered the most basic defense mechanism because it underlies all others. It was the initial defense mechanism identified by Freud, and he used it to explain the etiology of hysteria.

From the perspective of Freudian psychoanalytic theory, the defense mechanism of _______________ is viewed as underlying all other defense mechanisms. Select one: A. regression B. repression C. sublimation D. projection

repression --Repression involves pushing unacceptable id impulses into the unconscious. Repression can be viewed as the most "basic" of the defense mechanisms since the goal of all defense mechanisms is to make unwanted impulses unconscious.

Eric Erikson proposed that an adolescent who doesn't successfully resolve the psychosocial conflict of identity versus identity confusion due to uncertainty about his/her sexual identity is likely to exhibit which of the following maladaptive responses? Select one: A. compulsion B. inertia C. disdain D. repudiation

repudiation --Repudiation is the maladaptive outcome for inadequate resolution of the identity vs. identity confusion conflict.

Pavlov produced "experimental neurosis" in experimental dogs by using which of the following procedures? Select one: A. applying negative reinforcement each time the dogs ate B. pairing eating with an aversive tone C. making it impossible for the dogs to escape electrical shock D. requiring the dogs to make difficult discriminations

requiring the dogs to make difficult discriminations --Stimulus discrimination refers to an organism's ability to discriminate between stimuli and thereby respond only to the conditioned stimulus with a conditioned response. While conducting studies on stimulus discrimination, Pavlov discovered that extremely difficult discriminations elicited behaviors characteristic of human neurosis.

In family therapy, the adolescent daughter complains that she doesn't think she can change her attitudes toward school and feels that doing what the therapist has requested is pointless. The therapist responds by saying, "Well, in that case, I think you shouldn't even try." The therapist's response is an example of which of the following? Select one: A. reactance B. restraining C. positioning D. prescription

restraining --This technique (telling a client not to change) is referred to as restraining.

Higher scores on the Gender Role Conflict Scale (GRCS) have been linked to a number of negative outcomes including lower self-esteem, more symptoms of depression, and more somatic complaints. For example, a study by Good et al. (1995) found that, for male college students, scores on which of the following subscales of the GRSC are most predictive of scores on a measure of psychological distress? Select one: A. conflicts between work and family relations B. restrictive affectionate behavior between men C. success, power, and competition D. restrictive emotionality

restrictive emotionality --The GRCS was designed as a measure of men's gender role conflict and provides scores on the four subscales listed in the answers to this question. For their sample of male undergraduates, Good et al. found that scores on the restrictive emotionality subscale were most predictive of scores on the Global Severity Index of the Symptom Checklist-90-Revised. Based on these results, the researchers conclude that men may experience psychological distress as a result of "their beliefs about the need for restrictive emotionality as part of male gender role expectations" (p. 8).

Which of the following contributes LEAST to depth perception for objects at great (versus short) distances? Select one: A. interposition B. linear perspective C. retinal disparity D. motion parallax

retinal disparity --Retinal disparity is a binocular cue and is also known as stereopsis. It refers to the fact that our two eyes see objects in the world from two different views, and, the closer an object, the greater the disparity of the two images. Depth perception depends on a combination of binocular and monocular cues. Binocular cues are responsible for depth perception of objects at relatively short distances, while monocular cues contribute to depth perception for objects at greater distances.

The loss of memory for events that occurred during the weeks prior to a head injury is referred to as: Select one: A. anterograde amnesia. B. retrograde amnesia. C. proactive interference. D. retroactive interference.

retrograde amnesia --Retrograde amnesia refers to amnesia for events "backward in time."

Ebbinghaus was one of the first investigators to systematically study memory. In his studies, Ebbinghaus used himself as a subject and memorized lists of nonsense syllables. Results of his research indicated that: Select one: A. syllables in the middle of a list are better remembered than those at the beginning and end of the list. B. when memorizing syllables, new learning tends to interfere with previous learning. C. overlearning improves memory for syllables up to a point but thereafter has no effect. D. rote learning of syllables tends to lead to rapid forgetting.

rote learning of syllables tends to lead to rapid forgetting. --Ebbinghaus memorized syllables using rote learning (verbatim learning through repetition) and found that rote learning of nonsense syllables leads to rapid forgetting. Ebbinghaus was interested in assessing the effects of time on memory and, based on the results of his studies, constructed a "curve of forgetting," which shows that 50% of memorized syllables are forgotten within less than one hour.

Two rats are being conditioned to push a button in order to receive a food pellet. During the acquisition of this behavior, both rats were reinforced using a continuous schedule of reinforcement. After their response rates peaked and then declined, one rat began to receive two food pellets after each button push, whereas the second rat continued to received a single pellet after each response. Despite this difference in the amount of reinforcement, the response rate (i.e., the number of button pushes) for both rats remained the same. This is most likely attributable to which of the following? Select one: A. extinction B. habituation C. satiation D. inhibition

satiation --Satiation occurs when a reinforcer loses its reinforcing value. In this experiment, there is evidence that both rats have become satiated because their responses had declined. So an extra food pellet will not have any more impact than a single food pellet.

As defined by Piaget, a __________ circular reaction occurs when a baby's action gets a pleasurable or interesting response from an object or other person, which then leads the baby to repeat the action. Select one: A. reflexive B. primary Incorrect C. secondary D. tertiary

secondary --According to Piaget, much of the cognitive development that occurs during the sensorimotor stage is the result of circular reactions, in which the child learns to reproduce a pleasurable or interesting experience that originally occurred by chance. Piaget distinguished between three types of circular reactions - primary, secondary, and tertiary. Secondary circular reactions predominate from ages 4 to 8 months and are actions involving other people or objects. For example, at this stage, an infant may learn that grabbing a rattle with his/her hand and shaking it produces an interesting noise.

First-and second-grade children who have been identified as being at risk for academic underachievement are provided with a special after-school program. This is an example of: Select one: A. primary prevention. B. secondary prevention. C. tertiary prevention. D. crisis intervention.

secondary prevention --Since the school program is being given to identified individuals (children who have been identified as being at risk for academic problems), this qualifies as a form of secondary prevention.

Which of the following information-processing strategies is characteristic of the reintegration status of Helms's (1995) White Racial Identity Development Model? Select one: A. suppression of information and ambivalence B. obliviousness and denial C. flexibility and complexity D. selective perception and negative out-group distortion

selective perception and negative out-group distortion --The combination of selective perception and negative out-group distortion is characteristic of the reintegration status. A person in this stage attempts to resolve moral conflicts about racism by idealizing White society and denigrating members of minority groups.

Anosognosia is best described as an impairment in: Select one: A. coordination and balance. B. speech. C. self-awareness. D. visual perception.

self-awareness --Anosognosia is defined as a deficit in self-awareness and, more specifically, awareness of one's own symptoms.

Which of the following systems has/have been linked to Social Phobias? Select one: A. serotonergic only B. serotonergic and dopaminergic C. cholinergic only D. cholinergic and glutamatergic

serotonergic and dopaminergic --Several neurotransmitter abnormalities have been linked to the anxiety disorders, but serotonin and dopamine have been most closely associated with Social Phobia.

Research by Kaye, Gendall, and Strober (1998) suggests that food restriction associated with Anorexia Nervosa reduces __________ levels. Select one: A. serotonin B. acetylcholine C. dopamine D. glutamate

serotonin --Kaye proposes that high levels of serotonin cause anxiety and that starvation reduces tryptophan, which then reduces brain levels of serotonin and temporarily relieves anxiety.

A person taking an SSRI in conjunction with an MAOI may develop ____________, which is characterized by agitation, confusion, tremor, unsteady gait, diarrhea, sweating, and chills. Select one: A. a hypertensive crisis B. a cholinergic crisis C. serotonin syndrome D. sympathomimetic syndrome

serotonin syndrome --Serotonin syndrome is associated with a number of cognitive, neuromuscular, and autonomic symptoms including the ones listed in this answer. Without treatment, it can lead to coma and death.

Second generation (atypical) antipsychotic drugs such as clozapine (Clozaril) and risperidone (Risperdal) are: Select one: A. serotonin-dopamine antagonists. B. serotonin-dopamine agonists. C. serotonin agonists/dopamine antagonists. D. serotonin antagonists/dopamine agonists.

serotonin-dopamine antagonists. --To choose the correct response to this question, you also need to know that an antagonist is a drug that blocks the effects of a neurotransmitter and that an agonist is a drug that increases or mimics a neurotransmitter's effects. Many of the second generation antipsychotics exert their beneficial effects on the positive and negative symptoms of Schizophrenia primarily by blocking the effects of dopamine and serotonin.

The National Campaign on Teen Pregnancy (Kirby, 2001) concluded that which of the following has the strongest evidence of success for reducing teen pregnancy rates? Select one: A. abstinence-only programs B. service learning programs C. school condom distribution programs D. community-wide programs and initiatives

service learning programs

The function of the state licensing boards is probably best described as: Select one: A. ensuring that psychologists provide clients with effective services. B. ensuring that only competent psychologists become and remain licensed. C. setting entry-level qualifications for licensure and monitoring the conduct of licensed psychologists. D. defining the services that can be legitimately provided by licensed psychologists.

setting entry-level qualifications for licensure and monitoring the conduct of licensed psychologists.

A psychologist is hired by a defendant's attorney to evaluate the defendant for a possible insanity defense. Prior to conducting the evaluation, the psychologist: Select one: A. should inform the defendant that all information provided during the evaluation is confidential and will be revealed only to his attorney. B. should inform the defendant that information will be provided only to his attorney unless the defendant and his attorney decide to use the results of the evaluation to support an insanity defense. C. should inform the defendant that information related to his mental state at the time he committed the crime is not confidential and will be provided to the court and the opposing attorney. D. should inform the defendant that any information he provides during the course of the evaluation will be revealed not only to his attorney but also to the court and the opposing attorney.

should inform the defendant that information will be provided only to his attorney unless the defendant and his attorney decide to use the results of the evaluation to support an insanity defense. --In this situation, the client should be advised that information will be shared only with his attorney unless the client and his attorney decide to use it to support an insanity or similar defense.

Marlatt and Gordon's (1985) relapse prevention model focuses on: Select one: A. situations antecedent to relapse. B. the perceived consequences of relapse. C. controlled drinking. D. contingency management.

situations antecedent to relapse --According to Marlatt and Gordon, relapse is a failure to maintain a behavior change after treatment and is best avoided by identifying and dealing with its antecedents.

Social comparison theory's predictions about behavior are particularly applicable to: Select one: A. situations involving uncertainty. B. inequitable situations. C. people who are working on a difficult task. Incorrect D. people who belong to a cohesive group.

situations involving uncertainty --According to social comparison theory, people evaluate their own attitudes, abilities, and emotions by comparing themselves with similar others. Social comparison theory applies to a variety of situations but, as noted in the Social Psychology chapter of the written study materials, seems to be particularly applicable to situations that lack objective standards (i.e., to uncertain situations).

For gay and lesbian adolescents, the primary presenting problem at social service agencies is: Select one: A. suicidal ideation. B. drug or alcohol abuse. C. social and emotional isolation. D. academic problems.

social and emotional isolation --Isolation is frequently cited as a primary problem for gay and lesbian adolescents, and one study found it to be the single-most frequent presenting problem

The primary target of "advocacy consultation" is best described as: Select one: A. worker well-being. B. the child's best interests. C. interpersonal conflicts. D. social change.

social change --As noted by J. C. Conoley, a distinctive characteristic of advocacy consultation is that it is based on an "explicit value orientation that targets social change in the direction of power equalization"

__________ predicts that the decision to remain or leave a relationship depends on the relative costs and rewards of that relationship. Select one: A. Social comparison theory B. Self-verification theory C. Social exchange theory D. Gain-loss theory

social exchange theory --Social exchange theory predicts that we're more likely to stay in a relationship when the rewards of the relationship exceed its costs.

Gerald Patterson and his colleagues have developed a model to explain the origins of adolescent delinquency. Their approach is most consistent with which of the following? Select one: A. object relations theory B. social learning theory C. cognitive developmental theory D. social comparison theory

social learning theory --Patterson's model emphasizes the impact of modeling/imitation and reinforcement and, therefore, is consistent with a social learning approach.

In terms of sexual orientation, most men who receive a diagnosis of Transvestic Disorder identify themselves as: Select one: A. solely or predominantly heterosexual. B. solely or predominantly homosexual. C. unequivocally bisexual. D. unequivocally asexual.

solely or predominantly heterosexual. --According to the DSM-5 the majority of men with Transvestic Disorder identify themselves as heterosexual, although some have occasional sexual relations with other men, especially when cross-dressed.

StartofPracticeExam7Questions: Epidemiological studies on rates of various mental disorders in urban and rural areas in the United States have found that: Select one: A. for most mental disorders, the rates are significantly higher in urban areas. B. for most mental disorders, the rates are significantly higher in rural areas. C. some disorders are more common in rural or in urban areas, but the differences in rates are usually not statistically significant. D. some disorders are more common in rural or in urban areas, and the differences in rates are usually statistically significant.

some disorders are more common in rural or in urban areas, but the differences in rates are usually not statistically significant. --Overall, the best conclusion that can be drawn is that there are few consistent (or statistically significant) differences in the rates of mental disorders in urban and rural areas. For example, a recent Healthcare for Communities (HCC) survey of 9,585 individuals living in rural and urban areas found no significant differences in the prevalence of the major types of mental illness . This lack of significant differences between rates in rural and urban areas is consistent with the findings of the Epidemiologic Catchment Area Study and the National Comorbidity Survey.

When discussing a new screening test that you've developed with several psychologists, one of the psychologists says, "Let's say that I know a particular patient does not have the disorder that is assessed by the screening test. What is the chance that the test results will show that my patient does not have the disorder?" This psychologist is interested in your test's: Select one: A. sensitivity. Incorrect B. specificity. C. positive predictive value. D. negative predictive value.

specificity --A test's specificity refers to the proportion of individuals without the disorder who are correctly identified by the test as not having the disorder.

The internal consistency of test items can be assessed with which of the following? Select one: A. kappa statistic B. split-half reliability C. alternate forms reliability D. coefficient of stability

split-half reliability --Internal consistency (homogeneity of test items) is used as an index of reliability. Split-half reliability is a type of internal consistency reliability and provides information on the consistency of items in two halves of the test.

As defined by Aaron Beck, automatic thoughts are: Select one: A. more accessible and less stable than voluntary thoughts. B. spontaneously triggered by a specific circumstance and accompanied by an emotional reaction. C. systematic errors in reasoning that are evident during times of psychological distress. D. always irrational.

spontaneously triggered by a specific circumstance and accompanied by an emotional reaction.

The primary advantage of manual-guided treatment is that the use of manuals: Select one: A. standardizes treatment delivery so that, in research, treatment effects can be more clearly ascertained. B. substantially improves treatment outcomes across different clients, diagnoses, and contexts. C. reduces the legal liability of clinicians for "treatment errors." D. ensures that the procedures used to evaluate treatment effectiveness are valid.

standardizes treatment delivery so that, in research, treatment effects can be more clearly ascertained. --Manual-guided treatments provide manuals that specify the theoretical underpinnings of the treatment and treatment goals and present specific therapeutic guidelines and strategies. Treatment manuals were initially developed to standardize treatments so that their effects could be empirically evaluated and to provide training guidelines for therapists.

Secondary reinforcers are: Select one: A. stimuli that increase a behavior through their accidental pairing with that behavior. B. stimuli that gain their reinforcing value through their association with an unconditioned reinforcer. C. high-frequency behaviors that are used to reinforce low-frequency behaviors. D. reinforcers that are removed (rather than applied) following a behavior.

stimuli that gain their reinforcing value through their association with an unconditioned reinforcer. --Primary (unconditioned) reinforcers are inherently reinforcing, while secondary (conditioned) reinforcers acquire their reinforcing value through their association with one or more primary reinforcers. Money is an example of a secondary reinforcer.

Your new client, Jay J., age 31, complains that he's had trouble falling asleep at night for a number of years but that it's become a more significant problem since he got married last year because his sleep problems are now keeping his wife awake. You instruct Jay to go to bed only when he feels tired; to get out of bed, go to another room, and engage in a relaxing activity if he doesn't fall asleep within about 20 minutes after going to bed; to use the bed only for sleep and sexual activity; to get up in the morning at about the same time regardless of how much sleep he's had; and to avoid napping. You are using which of the following techniques? Select one: A. habit reversal B. Premack principle C. stimulus control D. overcorrection

stimulus control --Stimulus control involves increasing the links between certain cues and a behavior in order to increase the frequency of that behavior. It was originally applied to the treatment of sleep problems by R. Bootzin

The aspect of implicit memory that is referred to as procedural memory is mediated primarily by the: Select one: A. hippocampus. B. thalamus. C. medulla. D. striatum.

striatum --The striatum (also referred to as the corpus striatum) is part of the basal ganglia and consists of the caudate nucleus and putamen. It plays an important role in procedural memory. Implicit memories are retrieved or recalled without conscious awareness. Several types of implicit memories are distinguished including procedural memories (memories for skills and procedures) and associative memories (memories that result from classical conditioning). Implicit memory has been linked to several areas of the brain including the striatum, amygdala, and cerebellum.

As defined by Alfred Adler, __________ refers to a person's goals and the ways in which he or she attempts to achieve those goals. Select one: A. homeostasis B. success identity C. self-actualization D. style of life

style of life --Adler believed that each person has a unique set of goals and ways for achieving goals, which he referred to as the person's style of life. According to Adler, a person's style of life is fairly well established by 4 or 5 years of age.

Virginia Satir's approach to family therapy emphasizes: Select one: A. styles of communication. B. family projection processes. C. boundary disturbances. D. transferences between family members.

styles of communication --Satir distinguishes between five communication styles--placater, blamer, super-reasonable, irrelevant, and congruent.

Which of the following best describes the requirements of the ethics codes published by the American and Canadian Psychological Associations? Select one: A. Subjects do not have to be debriefed about the nature of a research study following their participation unless doing so is required by the institutional review board. B. Subjects do not have to be debriefed about the nature of a research study following their participation unless the study involved the use of deception. C. Subjects must be debriefed about the nature of a research study following their participation in a way that minimizes harm. D. Subjects must always be debriefed about the nature of a research study immediately following their participation.

subjects must be debriefed about the nature of the research study following their participation in a way that minimizes harm --This answer is most consistent with the requirements of the American and Canadian codes: Standard 8.08(c) of the APA's Ethics Code states that, "when psychologists become aware [during the debriefing] that research procedures have harmed a participant, they take reasonable steps to minimize the harm." And Principle II.44 of the Canadian Code of Ethics states that psychologists must "debrief research subjects in such a way that any harm caused can be discerned, and act to correct any resultant harm."

A patient experiencing chronic pain tells you that her physician has prescribed an opioid analgesic. She also says she doesn't want to take the drug because her brother is an ex-drug addict and she has had problems with alcohol in the past. You should: Select one: A. continue to explore her feelings about drug addiction in subsequent therapy sessions. B. support her decision and explore alternative methods of pain relief such as biofeedback and relaxation training. C. suggest that you discuss the issue with her physician and obtain a waiver of confidentiality so that you can do so. D. reassure her that the use of an opioid for pain is unlikely to result in addiction as long as the drug is taken as prescribed.

suggest that you discuss the issue with her physician and obtain a waiver of confidentiality so that you can do so.

Research on "motor imagination" indicates that which area of the brain is most likely to be active when a person is imagining that he/she is engaging in a motor activity? Select one: A. posterior occipital lobe B. mammillary bodies C. supplementary motor area D. ascending reticular activating system

supplementary motor area --The specific areas that are activated when a person imagines he/she is engaging in a motor activity depend on the specific nature of the activity. However, the areas most likely to be involved include the supplementary motor area, cerebellum, basal ganglia, and parietal lobe (especially the somatosensory cortex).

The most effective intervention for cigarette smoking combines nicotine replacement therapy with: Select one: A. antidepressants and bibliotherapy. B. support from a clinician and skills training. C. covert sensitization and relapse prevention. D. stimulus control and habit reversal training.

support from a clinician and skills training. --There is evidence that NRT is most effective in terms of long-term effects when it is combined with guidance and support from a mental health professional and skills training that focuses on ways for avoiding and dealing with relapse.

HIV infection is often described in the literature in terms of three stages. The second stage is characterized by a strong immune system response and: Select one: A. swollen glands but other few symptoms. B. flu-like symptoms. C. the onset of numerous opportunistic infections. D. impaired concentration, attention, and motor activity.

swollen glands but other few symptoms --The second (middle) stage lasts for about eight to ten years and involves few, if any, clinical symptoms. During this asymptomatic stage, the immune system in fighting the virus.

A couple comes to therapy complaining that their conversations always end up as arguments. In talking with the couple, you realize that a common pattern is for one partner to make an offensive comment to the other and for the other partner to respond with an even more offensive remark. This pattern of communication is referred to as: Select one: A. symmetrical. B. complementary. C. mystification. D. pseudohostility.

symmetrical --In this situation, the husband and wife are mirroring each other's behavior in what is sometimes referred to as a "one-upsmanship" game. Symmetrical communication is characterized by equality between the partners. It can lead to conflict and competition (symmetrical escalation) when each partner tries to "one-up" the other.

You conduct a research study to assess the effects of T.V. violence on aggressive behavior. You plan to observe sixteen children during recess for three days and calculate their average number of aggressive acts. You will then have all children observe aggressive T.V. programs for three hours and again observe the children during recess for three days and calculate their average number of aggressive acts. To analyze the data you collect, you will use which of the following? Select one: A. t-test for a single sample Incorrect B. t-test for correlated samples C. one-way ANOVA D. chi-square for a single sample

t-test for correlated samples --This study has one dependent variable that is measured on a ratio scale (number of aggressive acts) and one independent variable that has two levels (before film and after film). The analysis will involve comparing two means obtained from the same group of participants. Therefore, the means are related and the t-test for related (correlated) samples is the appropriate test.

Item response theory is considered a useful technique for constructing all of the following types of tests except: Select one: A. criterion-keyed tests. B. tailored (computerized) tests. C. diagnostic or employment screening tests. D. teacher-made (classroom) achievement tests.

teacher-made (classroom) achievement tests. --Item response theory is theoretically and mathematically very complex, which limits its use. Use of item response theory for test construction requires constructing an "item response curve" for each item. To do so, a large sample of examinees is needed, which would probably not be available for classroom tests. The other types of tests listed have been identified as good candidates for item response theory.

A client you have been seeing in therapy for three months tells you that her former therapist made repeated sexual advances toward her. She tells you that she does not want you to say anything about it to anyone and that she just wants to "get over it." As an ethical psychologist, you should: Select one: A. inform the client that you are ethically obligated to make a report to the state licensing board. B. convince the client that she should make a report to the appropriate authorities. C. tell the client that you will maintain confidentiality. D. tell the client that you must file a complaint with APA but that you will not reveal her name.

tell the client that you will maintain confidentiality.

To determine the degree of association between two continuous variables that have been artificially dichotomized, you would use which of the following correlation coefficients? Select one: A. eta B. biserial C. tetrachoric D. contingency

tetrachoric --The tetrachoric correlation coefficient is used to determine the relationship between two normally distributed continuous variables that have been artificially dichotomized. For example, it would be used to assess the degree of association between treatment outcome and symptom severity, when both variables were originally measured on a continuous scale but were then dichotomized so that outcome is categorized as either successful or unsuccessful and symptom severity is categorized as either mild or severe.

Research on infantile amnesia has shown that: Select one: A. children do not develop memories for autobiographical events until age 3 or 4. B. the loss of early memories is universal and is unrelated to gender, culture, or type of event. C. memories for autobiographical events that occurred prior to age 3 or 4 remain stable over the lifespan. D. the ability to recall memories for autobiographical events that occurred prior to age 3 or 4 varies over the lifespan.

the ability to recall memories for autobiographical events that occurred prior to age 3 or 4 varies over the lifespan. --The studies on infantile amnesia have found that memories for events that occurred prior to age 3 or 4 are limited but not completely absent. In addition, the recall of early memories varies across the lifespan. J. M. Rybash and K. L. Hrubi, for example, found that older adults typically have later-occurring first memories than younger adults do. and J. Kilstrom and J. Harackiewicz found that, in adolescence, reported first memories vary over time.

Research on Helms's White Racial Identity Development Model has shown that White therapists are most effective in cross-cultural counseling situations when they are in which stage of identity development? Select one: A. the stage that matches the client's stage of identity development B. the stage that complements the client's stage of identity development C. the reintegration stage D. the autonomy stage

the autonomy stage --The research on Helms's model has found that, the more advanced the White therapist's identity development, the greater his/her effectiveness when working with clients from culturally diverse groups.

Studies comparing the age at which gays and lesbians first disclose their sexual orientation to a friend, family member, or other person have found that: Select one: A. the average age of disclosure for gay men is 2 to 3 years earlier than the average age of disclosure for lesbians. B. the average age of disclosure for gay men is 5 to 6 years earlier than the average age of disclosure for lesbians. C. the average age of disclosure for lesbians is 3 to 4 years earlier than the average age of disclosure for gay men. D. the average ages of disclosure for gay men and lesbians are not significantly different.

the average ages of disclosure for gay men and lesbians are not significantly different --In a recent study, R. C. Savin-Williams and L. M. Diamond compared the sexual identity milestones for gay males and lesbians and found that, in terms of disclosure, the average age was not substantially different. They also report the results of previous research, which is generally consistent with their findings. Note that Savin-Williams and Diamond did find significant differences for initial same-sex sexual contact and first self-labeling as gay or lesbian, with the average age for both milestones being lower for gay men than for lesbians.

According to Kinder and Sears (1981), symbolic racism is rooted in: Select one: A. authoritarianism. B. realistic self-interest. C. the belief that members of minority groups violate traditional American values. D. limited social contact with members of minority groups or contact only under aversive conditions.

the belief that members of minority groups violate traditional American values. --Kinder and Sears (1981) proposed that symbolic racism has largely replaced overt (old-fashioned) racism. According to Kinder and Sears, symbolic racism is characterized by a combination of negative affect toward minorities and the belief that minorities violate traditional American values such as individualism, self-reliance, and the work ethic.

During the fourth session, a therapy client says he "is already feeling much better." Most likely: Select one: A. this is simply a transitory placebo effect. B. this reflects the initial development of insight. C. the client is beginning to feel more hopeful about his situation. D. the client is starting to benefit from newly acquired coping skills.

the client is beginning to feel more hopeful about his situation --According to Howard and his colleagues, during the first few sessions of therapy, a client experiences an increase in hopefulness. This stage is referred to as remoralization.

A child living in the United States is exposed to both English and a second language between the ages of six months and three years and, as result, becomes fluent in both languages. If the child had not been exposed to the second language until after the age of 4 or 5, she would have had more trouble acquiring the second language. This provides evidence for: Select one: A. the concept of critical periods B. the concept of sensitive periods C. the Whorfian hypothesis D. the notion of imprinting

the concept of sensitive periods --Although some authors use the terms "critical period" and "sensitive period" interchangeably, they actually mean two different things. A critical period refers to a time when, if certain developmental events don't happen, later aspects of development will not occur. A sensitive period refers to an optimal period rather than a necessary period. The situation described in this question suggests there is a sensitive period for dual language learning.

From the perspective of Gestalt therapy, an optimal "contact boundary" is semipermeable and coterminous with the actual self. Introjection is one type of boundary disturbance. It occurs when: Select one: A. the contact boundary is located far in the environment. B. the contact boundary is located deep within the self. C. the contact boundary is turned back against the self. D. the contact boundary is extremely rigid.

the contact boundary is located deep within the self. --Introjection occurs when a person incorporates aspects of another person into him/herself. The boundary is located inside one, thereby allowing others to be incorporated into the self.

When factors in a factor matrix are "oblique," this means that: Select one: A. the correlation coefficient for any two factors is equal to zero. B. the correlation coefficient for any two factors is greater than zero. C. the identified factors explain a statistically significant amount of variability in test scores. D. the identified factors do not explain a statistically significant amount of variability in test scores.

the correlation coefficient for any two factors is greater than zero

Research on the serial position effect has provided evidence for which of the following? Select one: A. the presence of hemispheric specialization. B. the distinction between short- and long-term memory. C. the effects of proactive and retroactive interference. D. the effects of latent learning.

the distinction between short- and long-term memory. --The serial position effect refers to the tendency of people to recall words in the beginning and end of a list better than words in the middle of the list when asked to recall the words immediately after reading the list. The ability to recall words from the beginning of the list is referred to as the primacy effect and is attributed to the storage of those words in long-term memory. In contrast, the ability to recall words from the end of the list is referred to as the recency effect and is attributed to the presence of those words in short-term memory.

Two studies are conducted to test the effects of a cognitive-behavioral technique on attention span in children with ADHD. The studies are identical in procedures, number of participants, and so on, but in Study #1, p<.05, while in Study #2, p>.10. Based on this information, you can conclude that: Select one: A. the results of Study #1 indicate an effect opposite of the effect produced in Study #2. B. the effect size in Study #1 was larger than the effect size in Study #2. C. the researcher has made a Type II error in Study #2. D. a one-tailed test was used in Study #1; a two-tailed test in Study #2.

the effect size in Study #1 was larger than the effect size in Study #2. --The difference between the two studies is that one achieved statistical significance (p<.05), while the other did not (p>.10). Significance is achieved when the effect size (effect of the IV) is sufficiently large to be detected by the statistical test. There is enough information given in this question to draw this conclusion.

In the context of operant conditioning, "fading" refers to: Select one: A. a reduction in positive reinforcement. B. the gradual removal of prompts. C. extinguishing a previously reinforced response. D. eliminating stimulus generalization.

the gradual removal of prompts. --Nonverbal and verbal prompts are used to help initiate a response. Because the ultimate goal is for the response to occur independently, prompts are gradually removed once the response is well-established. This procedure is referred to as fading.

When the Kuder-Richardson Formula 20 produces a coefficient of .98, this means that: Select one: A. the items in the test are homogeneous. B. the trait being measured by the test is stable over time. C. the measuring instruments are equivalent. D. the test measures what it was designed to measure.

the items in the test are homogeneous --As a measure of internal consistency, a high KR-20 coefficient indicates a homogeneous test.

A psychotherapist is most likely to say that which of the following is the greatest source of stress in his or her work? Select one: A. isolation B. overwork C. the general passive nature of his/her work D. the lack of therapeutic success

the lack of therapeutic success --The overwhelming majority of respondents (73.7%) said the major source of stress in their work was the lack of therapeutic success.

When conducting a principal components analysis, the components are extracted so that: Select one: A. each component accounts for the same amount of variability. B. the first component accounts for the largest amount of variability. C. the last component accounts for the largest amount of variability. D. the middle component accounts for the largest amount of variability.

the last component accounts for the largest amount of variability. --Principal components analysis is similar to factor analysis and is used to identify a set of variables that accounts for all (or nearly all) of the total variance in a set of test scores. Principal components are extracted so that the first component accounts for the largest amount of variability in test scores; the second component accounts for the second largest amount of variability, etc.

From the perspective of Prochaska and DiClemente's (1984) transtheoretical model of change, the failures of treatment programs for cigarette smoking are often due to the fact that: Select one: A. the majority of smokers are not ready to change. B. cigarette-smoking is self-reinforcing and difficult to extinguish. C. nicotine addiction has a genetic component. D. interventions do not address the environmental cues that trigger relapse.

the majority of smokers are not ready to change. --Research by Prochaska and colleagues has shown that a majority of individuals with addictions are in the precontemplation or contemplation stage and, therefore, are ambivalent about change. For these individuals, most interventions will be unsuccessful.

Studies on concurrent schedules of reinforcement have found that an organism's relative frequency of responding to one alternative corresponds to the relative frequency of reinforcement for responses to that alternative. This phenomenon is referred to as: Select one: A. the law of effect. B. the matching law. C. Rescorla-Wagner theory. D. the Premack principle

the matching law --The matching law states that, when using concurrent schedules of reinforcement, the proportion of responses will match the proportion of reinforcements.

To calculate an effect size (Cohen's d), you need: Select one: A. the standard error of estimate of the criterion. B. the median scores for the pre- and post-tests. C. the means of the experimental and control groups. D. the actual and predicted scores for the outcome measure.

the means of the experimental and control groups --Cohen's d indicates the magnitude of the effect of a treatment in terms of the difference between the means of the experimental (treatment) and control (no treatment) groups. It is calculated by subtracting the mean of the control group from the mean of the experimental group and dividing the result by a pooled standard deviation or by the control group standard deviation.

According to Gerald Patterson and his colleagues, which of the following is a key contributor to Conduct Disorder? Select one: A. the child's temperament B. the child's cognitive skills C. the strength of the parent-child attachment D. the parent's childrearing skills

the parent's childrearing skills --Patterson et al.'s "coercion hypothesis" focuses on interactions between parents and their children and identifies the parents' childrearing skills as a key determinant of the nature of these interactions.

The "FI scallop" refers to: Select one: A. the pause followed by accelerated responding that occurs within each fixed interval. B. the increase in responding that occurs after reinforcement at the end of each fixed interval. C. the gradual pause followed by an essential instantaneous high rate of responding that occurs between each fixed interval. D. the pause in responding that signals satiation after exposure to an extended fixed interval schedule.

the pause followed by accelerated responding that occurs within each fixed interval. --The term "scallop" is ordinarily applied to the response pattern that is observed when an organism is being reinforced on a fixed interval schedule. When using an FI schedule, the delivery of reinforcement is typically followed first by a pause in responding and then an increasing rate of responding until the next reinforcement is delivered. On a cumulative record, this produces a "scallop."

In the context of factor analysis, "specificity" refers to: Select one: A. the proportion of variability in a test that has not been explained by the factor analysis. B. the proportion of variability in a test that has been explained by a single factor. C. the proportion of variability in a test that has been explained by all of the identified factors. D. the proportion of variability in a test that is attributable to measurement error.

the proportion of variability in a test that has not been explained by the factor analysis. --In factor analysis, a test's specificity is the variability that is due to factors that are specific to the test and not measured by any other test included in the analysis -- i.e., variability that is not accounted for by the identified factors. From the perspective of factor analysis, variability in test scores is due to a combination of communality, specificity, and error.

For a client or former client to successfully bring a charge of malpractice against a psychologist, he/she must show that: Select one: A. the psychologist practiced in a way that could have led or did lead to harm. B. the psychologist knowingly or deliberately acted in a way that led to harm. C. the psychologist's actions were the cause of a demonstrable injury. D. the psychologist did not adequately warn the client of the possible negative consequences of the treatment.

the psychologist's actions were the cause of a demonstrable injury. --Some experts describe malpractice in terms of four basic elements: The psychologist must owe a duty of care to the plaintiff; there must have been a breach of that duty; the breach must have led to demonstrable injury; and the psychologist's acts must have been the cause of that injury. This response addresses the third and fourth elements of a malpractice claim.

The split-plot ANOVA is used when: Select one: A. one of the independent variables is an extraneous variable. B. all independent variables are quantitative. C. the researcher has used a "matched-pair" design. D. the researcher has used a "mixed" design.

the researcher has used a "mixed" design. --Knowing that the split-plot ANOVA is also referred to as the mixed ANOVA would have helped you identify this as the correct answer.

When using the split-half method to estimate the reliability of a 100-item speed test: Select one: A. the split of the test into halves should be done in a random way. B. the test should be split so that the first 50 items are grouped together and the second 50 items are grouped together. C. the resulting reliability coefficient will overestimate the reliability of the test if the split is made on the basis of odd- versus even-numbered items. D. the resulting reliability coefficient will underestimate the reliability of the test if the split is made on the basis of odd- versus even-numbered items.

the resulting reliability coefficient will overestimate the reliability of the test if the split is made on the basis of odd- versus even-numbered items. --For the exam, be sure to remember that split-half and other forms of internal consistency reliability overestimate the reliability of a speed test.

Berkowitz (1971) added which of the following to the original frustration-aggression hypothesis? Select one: A. the role of aggressive cues B. the impact of the model's attractiveness C. the role of the individual's level of frustration tolerance D. the likelihood that aggressive behavior will be rewarded or punished

the role of aggressive cues --The frustration-aggression hypothesis was one of the first psychological theories of aggression and proposd that frustration triggers a readiness to act aggressively. Berkowitz extended the original theory by proposing that unjustified frustration leads to anger and that this anger is most likely to elicit aggression in the presence of aggressive cues.

When using second-order conditioning to establish a new conditioned response: Select one: A. the second conditioned response will not be as strong as the first conditioned response. B. the second conditioned response will be difficult to establish if the second conditioned stimulus is not similar in nature to the first conditioned stimulus. C. the second conditioned response will be more difficult to extinguish than the first conditioned response. D. the second conditioned stimulus will produce "experimental neurosis" if it is too similar to the first conditioned stimulus.

the second conditioned response will not be as strong as the first conditioned response. --The further the stimulus is from the original US, the less in magnitude the CR. In other words, the first CR will be less in magnitude than the original UR, and the second CR will, in turn, be less in magnitude than the first CR.

Delirium shares many symptoms with Schizophrenia, Schizophreniform Disorder, and certain other psychotic disorders, such as disordered thinking, hallucinations, and delusions. Thus, the differential diagnosis of these disorders is often difficult. Generally speaking, however, Delirium can be distinguished from the psychotic disorders because: Select one: A. the symptoms of Delirium tend to be random and haphazard while the symptoms of the psychotic disorders are ordinarily systematized. B. the onset of symptoms in Delirium is insidious while the onset of symptoms in psychotic disorders is usually rapid. C. clouding of consciousness is rare in Delirium but common in the psychotic disorders. D. the psychotic disorders involve hallucinations and delusions but these symptoms are never present in Delirium.

the symptoms of Delirium tend to be random and haphazard while the symptoms of the psychotic disorders are ordinarily systematized. --A difference between Delirium and the psychotic disorders is that the symptoms of Delirium tend to be random and haphazard while the symptoms of the psychotic disorders are often systematized.

In structural family therapy, "joining" depends on: Select one: A. the motivation of family members to change. B. the permeability of the boundaries between family members. C. the therapist's ability to remain neutral. D. the therapist's ability to adapt to the family.

the therapist's ability to adapt to the family. --In order to join the family, the therapist must be able to adapt or accommodate to its style.

When conducting a factor analysis, a researcher would choose an orthogonal (rather than oblique) rotation when: Select one: A. the variables included in the analysis are correlated. B. the variables included in the analysis are uncorrelated. C. the effects of one or more variables have been removed from X only. D. the effects of one or more variables have been removed from X and Y.

the variables included in the analysis are uncorrelated --An orthogonal rotation is used when the variables included in the analysis are believed to be uncorrelated. For example, if you conduct a factor analysis on 50 questionnaire items designed to measure a leader's task- or person-orientation and you believe that these two orientations are independent (uncorrelated), you would perform an orthogonal rotation.

As defined by Gerald Caplan, the target of consultee-centered case consultation is the consultee's skills, knowledge, abilities, and/or objectivity. Caplan describes which of the following as a potential cause of a consultee's lack of objectivity? Select one: A. projective identification B. diagnostic overshadowing C. parallel process D. theme interference

theme interference --Caplan defines theme interference as a type of transference that occurs when a consultee's unresolved conflict related to a particular type of client or situation interferes with his/her current performance with similar clients or in similar situations.

When working with older adults, it is important to remember that: Select one: A. psychotherapy, especially insight-oriented therapy, is generally ineffective. B. there is greater variability among older people than younger people on a range of characteristics. C. a therapeutic alliance is particularly difficult to establish, especially when the therapist is young. D. all of the above.

there is greater variability among older people than younger people on a range of characteristics. --When working with older adults, it is important to remember that there is greater variability among older people than younger people on a range of characteristics (physical health, cognitive skills, income, etc.).

For the treatment of migraine headaches: Select one: A. autogenic training is more effective than thermal biofeedback. B. thermal biofeedback is more effective than autogenic training. C. thermal biofeedback plus autogenic training is more effective than either treatment alone. D. thermal biofeedback plus autogenic training is no more effective than either treatment alone.

thermal biofeedback plus autogenic training is more effective than either treatment alone. --Several studies have found that this combination of treatments is the best approach for migraine headaches.

When using protocol analysis to assess problem-solving in adolescents, a psychologist will ask each adolescent to: Select one: A. assemble a representative set of task assignments that the psychologist will review. B. report the first answer he/she thinks of when presented with a problem. C. describe how he/she determined the solution after working on a problem. D. think aloud while solving a problem.

think aloud while solving a problem --Protocol analysis involves analyzing the transcript (protocol) of a person's verbalizations while solving a problem, making a decision, etc. Having a person think-aloud while solving a problem, recording the person's verbalizations, and then analyzing the transcript of that recording are the procedures used in protocol analysis.

Dr. Chang evaluates the relationship between a child's behavior and rejection of the child by peers by observing the behavior of children previously classified as rejected or non-rejected in free-play situations over several days. During each observation period, Dr. Chang observes the first child on a list for 10 seconds and then records the child's behavior, observes the next child on the list for 10 seconds and records his/her behavior, and so on until all children have been observed. Dr. Chang is using which of the following strategies? Select one: A. time sampling B. time series C. sequential act coding D. multiple baseline

time sampling --Time sampling involves observing an individual or group of individuals during prespecified periods of time and, at the end of each observation, recording whether or not the target behavior(s) occurred. Time sampling is usually categorized as a type of interval recording.

In working with a client who has received a diagnosis of Dissociative Identity Disorder, a cognitive therapist's initial goal would be: Select one: A. to collaborate with the client to identify specific treatment goals. B. to make a "cognitive diagnosis" for the purpose of developing a concrete treatment plan. C. to identify the cognitive antecedents and consequences that are sustaining the client's multiple personalities. D. to establish a working alliance that provides the client with a sense of safety.

to establish a working alliance that provides the client with a sense of safety.

StartofPracticeExam5Questions:Historically, which of the following has (have) been considered the "hallmark" of addiction? Select one: A. a "craving" for the substance B. denial C. impairments in social and occupational functioning D. tolerance and withdrawal

tolerance and withdrawal --The repeated administration of certain drugs can result in physical dependence (addiction), which is characterized by tolerance and withdrawal symptoms when drug use is abruptly stopped.

Youngsters begin to describe themselves in terms of personality attributes - e.g., "I'm smart," "I'm friendly," "I'm shy": Select one: A. in early childhood (ages 2-6). B. at the beginning of middle childhood (ages 7-8). C. toward the end of middle childhood (ages 10-12). D. in early adolescence (ages 13-14).

toward the end of middle childhood (ages 10-12) --Toward the end of middle childhood, a child's self-concept becomes increasingly based on personality traits.

To determine if there is a statistically significant pattern in the effect of time on memory for a list of nonsense syllables, you have subjects memorize a list of 15 syllables and then test their memory at 15-minute intervals for the next two hours. The best technique for analyzing the data you have collected is: Select one: A. multiple regression. B. factorial ANOVA. C. orthogonal analysis. D. trend analysis.

trend analysis --In this situation, you want to determine if there is a pattern (or "trend") in forgetting the list of nonsense syllables. Of the techniques listed, only trend analysis (a type of analysis of variance that is used when the IV is quantitative) would be useful for determining if there is a pattern or trend in subjects' memory for the list of nonsense syllables over time.

A practitioner of Minuchin's structural family therapy uses which of the following to alter the hierarchical relationships within a family system or subsystem? Select one: A. unbalancing B. tracking C. reframing D. mimesis

unbalancing --Unbalancing is a restructuring technique that is used by structural family therapists to change hierarchical relationships between family members. It may involve affiliating with a family member, ignoring a family member, or entering into a coalition with a family member against another member.

The DSM-5 includes the World Health Organization Disability Assessment Schedule 2.0 (WHODAS) as a means for assessing disability in adults ages 18 years and older. The WHODAS uses a 5-point Likert scale to assess functioning in which the following domains? Select one: A. self-care, life activities, cognitive skills, mood and affect, physical health, and interpersonal relationships B. understanding and communicating, getting around, self-care, getting along with people, life activities, and participation in society C. affect, personality functioning, substance use, cognition, personal relationships, and reality testing D. communication skills, activities of daily living, interpersonal skills, motor skills, cognition, and affect

understanding and communicating, getting around, self-care, getting along with people, life activities, and participation in society

Dr. Weiner, a psychologist, is conducting a research study and obtains subjects for his study by placing advertisements in a local newspaper. Dr. Weiner informs potential subjects about all aspects of the study and tells them that they should not agree to participate unless they are willing to stay for its duration (six sessions over a six-week period), since once they sign up, they will not be permitted to withdraw from the study. According to APA's Ethics Code, the proposed arrangement is: Select one: A. clearly ethical. B. ethical as long as the withdrawal restriction is included in the informed consents that all subjects must sign. C. ethical as long as the withdrawal restriction has been determined to be crucial to the study. D. unethical.

unethical --A researcher must respect the participants' right to withdraw at any time for any reason in the course of a study. a. Incorrect A withdrawal restriction such as the one described in the question is unethical.

The only time a client is able to see you is during her lunch hour, which requires her session to be limited to 30 minutes because of the distance she must travel to get to your office. When you bill her insurance company, however, you bill for the entire hour. This is: Select one: A. ethical since the shorter sessions are being provided at the client's request. B. ethical since your policy is to always bill clients for an entire hour. C. unethical. D. a legal, but not an ethical, issue.

unethical --This is the best answer of those given. Providing any inaccurate information to the insurance company would represent insurance fraud.

Akathisia is characterized by: Select one: A. spasms and contortions. B. motor and psychic hypoactivity. C. unpleasant feelings of physical restlessness. D. rigidity and coarse tremors.

unpleasant feelings of physical restlessness --Akathisia is an extrapyramidal side effect of the neuroleptic drugs and a symptom of Parkinson's disease and involves an unpleasant feeling of restlessness and need to move.

Older people are especially sensitive to both the therapeutic and toxic effects of psychoactive drugs. For example, for older (vs. younger) adults, benzodiazepine use is more likely to produce: Select one: A. unsteadiness, confusion, wandering, and memory loss. B. bizarre or hostile behavior, paranoia, and confusion. C. extrapyramidal symptoms and cardiovascular effects. D. tremor, weight gain, restlessness, and fatigue.

unsteadiness, confusion, wandering, and memory loss.

Research on the use of cognitive-behavioral therapy with older adults has found that it is: Select one: A. less effective than cognitive-behavioral therapy for younger adults and, consequently, is generally contraindicated. B. usually most effective when it is conducted at a slower pace and at higher levels of abstraction. C. usually most effective when it is short- (versus long) term and conducted at lower levels of abstraction. D. usually most effective when it focuses on replacing negative cognitions with positive ones and on the present rather than on the past.

usually most effective when it is conducted at a slower pace and at higher levels of abstraction. --Knight and Satre note that older adults generally benefit most from cognitive-behavior therapies when therapy is presented at a slower pace and is conducted at an abstract level (i.e., at a level that focuses on schemas and relationship scripts rather than on simple cognitions).

StartofPracticeExam8Questions: A DSM-5 diagnosis of Intermittent Explosive Disorder requires the presence of: Select one: A. verbal aggression or physical aggression toward property, animals, or other people that does not cause injury or destruction or behavioral outbursts that do cause injury or destruction. B. physical aggression that is injurious to animals or people or destructive to property. C. physically aggressive acts that are precipitated by an irritable mood and that cause injury to people or animals or destruction to property. D. aggressive acts that cause injury to people or animals or destruction to property and are accompanied by a lack of remorse or guilt. Incorrect

verbal aggression or physical aggression toward property, animals, or other people that does not cause injury or destruction or behavioral outbursts that do cause injury or destruction. --The DSM-5 diagnosis of Intermittent Explosive Disorder requires recurrent behavioral outbursts that reflect a failure to control aggressive impulses as manifested by verbal aggression or physical aggression toward property, animals, or other people that does not result in destruction or injury or behavioral outbursts that do result in destruction of property or injury of people or animals.

In his research, a cognitive psychologist uses paired-associate and serial learning tasks. Apparently this psychologist is investigating: Select one: A. verbal learning. B. selective attention. C. procedural memory. D. iconic memory.

verbal learning --A paired associate task requires the individual to respond with one member of a pair when presented with the other member. A serial learning task requires the individual to learn and recall a list of words in a particular order. These tasks are frequently used in the study of verbal learning.

A father wants to increase his son's studying and violin practice and, therefore, reinforces each behavior whenever it occurs with a token that can be exchanged later for a desired privilege. After a period of time, the father realizes that his son will never become a violin virtuoso and stops reinforcing the boy's violin practice. The father can expect which of the following? Select one: A. violin practice and studying will both decrease B. violin practice and studying will both increase C. violin practice will decrease but studying will increase D. violin practice will decrease and studying will stay the same

violin practice will decrease but studying will increase --Removal of reinforcement from a previously reinforced behavior typically results in an increase in another reinforced behavior. This phenomenon is referred to as "behavioral contrast."

The James-Lange theory of emotions emphasizes the role of: Select one: A. visceral and muscular reactions. B. thalamic stimulation of the cortex. C. limbic system structures. D. past experiences with emotion-arousing stimuli.

visceral and muscular reactions --The James-Lange theory is classified as a peripheralist theory of emotion. According to the James-Lange theory, "we are afraid because we run." In other words, emotions are perceptions of bodily reactions.

Visual agnosia is best conceptualized as: Select one: A. visual "not caring." B. visual "not knowing." C. visual repression. D. visual blindness

visual "not knowing" --Visual agnosia is the loss of the ability to recognize visual stimuli even though visual acuity is intact.

Dr. Bernard is designing a longitudinal study on drug use in high schools. Since the study is longitudinal, it will be necessary to maintain records containing participants' identities. A colleague of Dr. Bernard's reminds him of a similar situation in which the investigator was subpoenaed to appear in court to testify against a participant who had been involved in illegal activities. The investigator refused to testify in order to maintain the participant's confidentiality and was imprisoned. What should Dr. Bernard do in order to avoid a similar dilemma: Select one: A. if subpoenaed by the court, testify, since legal requirements override ethical considerations. B. if subpoenaed by the court, testify only after obtaining the consent of the participant. C. warn participants prior to participation in the study about the possibility of a breach of confidentiality and obtain informed consents. D. design the study so that the names of participants are maintained in a separate file by a third party.

warn participants prior to participation in the study about the possibility of a breach of confidentiality and obtain informed consents. --This is most in accord with the above-cited Standard, and it represents a balance between the psychologist's legal and ethical obligations. If he informs participants of the limits of confidentiality, they will give their consent to participate with full awareness of the possibility that the psychologist or his files might be subpoenaed.

According to Bem's (1972) self-perception theory: Select one: A. our attitudes and behaviors are unrelated. B. our emotions shape (cause) our attitudes and behaviors. C. we infer our attitudes from our behaviors. D. we rely on our attitudes to choose a behavior

we infer our attitudes from our behaviors. --According to Bem, people infer their own attitudes and emotions from their behaviors.

For a person with Bipolar Disorder, the risk for suicide is greatest: Select one: A. when depressive symptoms are most severe. B. when depressive symptoms have become less severe. C. shortly after the onset of a depressive episode. D. shortly after the onset of a manic episode.

when depressive symptoms have become less severe. --According to some experts, the risk for suicide is greatest when depressive symptoms begin to lift. Apparently, this is because, at this time, suicidal ideation has not subsided and the individual has the increased energy and planning ability required to carry out a suicide plan.

You conduct a research study to compare the effects of four weight-loss programs. To compare the average weight lost by participants in the four groups, you could conduct either a series of t-tests or a one-way ANOVA. You decide to use the one-way ANOVA because its use: Select one: A. will decrease the likelihood that you will make a Type I error. B. will decrease the likelihood that you will make a Type II error. C. will allow you to control the effects of an extraneous variable. D. will allow you to assess both main and interaction effects.

will decrease the likelihood that you will make a Type I error. --If you conduct separate t-tests in this study, you will have to make many comparisons Treatment 1 vs. Treatment 2, Treatment 1 vs. Treatment 3, Treatment 1 versus Treatment 4, Treatment 2 vs. Treatment 3, etc. As the number of comparisons in a study increases, so too does the probability of making a Type I error. a. CORRECT An advantage of the one-way ANOVA (or any ANOVA) is that it helps control the "experimentwise error rate" (i.e., the probability of making a Type I error). If alpha is set at .05 for this study, for instance, the probability of making a Type I error would be help at 5%. In contrast, if the individual t-tests were conducted, each at the .05 level, the probability of making a Type I error would be much higher.

Weber's law explains why: Select one: A. you perceive objects that are close together as a single stimulus or unity. B. you can be heard when you whisper in a very quiet room but must shout to be heard in a very loud, noisy room. C. you perform better in the presence of others on simple tasks but worse in the presence of others on difficult or complex tasks. D. you feel less pain after banging your elbow when you vigorously rub your elbow or apply heat to it.

you can be heard when you whisper in a very quiet room but must shout to be heard in a very loud, noisy room. --According to Weber's law, the "just-noticeable difference" in a stimulus is proportional to the magnitude of the original stimulus. It explains why a whisper is audible in a quiet room but not in a very noisy room.


संबंधित स्टडी सेट्स

American Lit: Native Americans and the Colonial Period

View Set

Chapter 6: Domain Controller and Active Directory

View Set

Microbiology Exam 3 (Chapter 10)

View Set

Chapter 6 The Skeletal System: Bone Tissue

View Set

Comparative Government Chapter 7

View Set